[go: up one dir, main page]

0% found this document useful (0 votes)
81 views103 pages

Surgery Preprof and Prof 39 To 47

Surgery

Uploaded by

Ajay
Copyright
© © All Rights Reserved
We take content rights seriously. If you suspect this is your content, claim it here.
Available Formats
Download as PDF, TXT or read online on Scribd
0% found this document useful (0 votes)
81 views103 pages

Surgery Preprof and Prof 39 To 47

Surgery

Uploaded by

Ajay
Copyright
© © All Rights Reserved
We take content rights seriously. If you suspect this is your content, claim it here.
Available Formats
Download as PDF, TXT or read online on Scribd
You are on page 1/ 103

47 Preprof 6.

A 25 years old shopkeeper presented with dilated


tortuous veins in right leg below knee with patch of
1. A 42 years female having a non healing ulcerated hyperpigmented skin at medial malleolus. The clinical
wound over her left leg. Past history of burn. O/E there diagnosis of this condition can best be described as:
is a wide scar over the previous burns area and A. C1
Irregular ulcer with raised and indurated base. What is B. C2
the most likely diagnosis? C. C3
A. Basal cell carcinoma D. C4
B. Marjelin's ulcer E. C5
C. Malignant melanoma
D. Varicose ulcer 7. Regarding intermittent claudication:
E Tubercular ulcer A. It is present on taking first step
B. It is usually relieved only when resting in the sitting
2. A patient developed acute blood transfusion reaction. position
The immediate step in management is: C. It is typically relieved when resting for more than 5
A. Recheck the cross match report minutes
B. Rehydrate the patient D. It is caused by anaerobic muscle metabolism
C. Stop blood transfusion E. It is due to chronic venous disease
D. Catheterize the patient
E. Give oxygen by mask 8. A young patient received in emergency having the
history of being caught in fire due to a cylinder explosion
3. Preferred treatment for cyclical mastalgia in 42 years and having multiple 3rd degree burns in his arms, chest.
old lady is: Upon arrival his haemodynamic status is as follows:
A. Piroxicam Pulse 130 b/m, BP 90/60 mmHg, RR 23 B/m, Temp
B. Firm bra during the day 97°F. What will be the choice of fluid in this patient?
C. Caffeine A. Blood transfusion
D. Evening primrose oil B. Dextran solution
E. Danazol C. Hartman solution
D. 5% dextrose saline
4. The lymphedema in a 25 years old male of 10 years' E. Sterofundin
duration is best classified as:
A. Primary 9. Preferred method of pleurodesis is by:
B. Juvenile A. Open surgery
C. Congenital B. Video-assisted thoracoscopic surgery (VATS)
D. Praecox C. Radiofrequency ablation
E. Tarda D. Talc
E. Laser ablation
5. Following tumor is associated with MEN 1 associated
hyperparathyroidism: 10. A 23 years old married girl presented with sudden
A. Medullary carcinoma thyroid pain in lower abdomen for 24 hours duration. On
B. Pheochromocytoma examination her pulse is 110 beats/minute, BP 100/60
C. Parathyroid tumor mmHg. Abdominal examination revealed bluish
D. Pituitary tumor discoloration around umbilicus. The most likely
E. Paraganglioma diagnosis is:
A. Acute pancreatitis
B. Ruptured ectopic pregnancy
C. Acute appendicitis
D. Pelvic inflammatory disease
E. Ovarian torsion

11. The hernia where only part of the bowel wall enters
the hernial sac is called as:
A. Spigelian hernia 16. A 14 years old boy presented to emergency with
B. Maydal's hernia painful groin and retention of urine, 02 hours after
C. Richter's hernia playing football with his friends. O/E his right testis is
D. Amyand's hernia painful and slightly pulled highs up left testis seems to
E. Litter's hernia be normal. What is mostly likely diagnosis?
A. Hydrocele
12. Regarding carcinoma of stomach: B. Epididymitis
A. Diffuse gastric carcinoma can be classified as per C. Orchitis
Japanese classification D. Undescended testis
B. Early gastric cancer is defined if not involving the E. Torsion of testis
adjacent lymph nodes
C. Advanced gastric cancer can be classified by Lauren 17. A young girl has been diagnosed with the Grave's
classification Disease. The key feature that differentiate It from the
D. Advanced gastric carcinoma can be classified by hyperthyroidism is:
Bormann classification A. Tachycardia
E. Advanced gastric cancer can be defined into two B. Exophthalmos
varieties C. Tremors
D. Weight loss
13. The functional lobes of liver can best be defined by: E. Delayed ankle jerks
A. Couinaud's plane
B. Middle hepatic vein 18. A 30 years male is having fever & tender
C. Cantlie's line hepatomegaly. U/s abdomen suggests a large (15×15
D. Falciform ligament cm) liver abscess. Which of the following is the most
E. Caudate line likely causative agent for this condition?
A. Echinococcus granulosus
14. Following condition is mostly benefited by B. Entamoeba Histolytica
splenectomy: C. E. Coli
A. Erythroblastosis fetalis D. Klebsiella
B. Hodgkin's lymphoma E. Pseudomonas
C. Splenic abscess
D. Idiopathic thrombocytopenic purpura 19. The commonest complication of chronic
E. Tropical splenomegaly osteomyelitis is
A. Amyloidosis
15. Regarding developmental dysplasia of hip, what is B. Pathological fracture
the most commonly used screening method for high risk C. Acute exacerbation
babies at birth? D. Sinuses discharging pus or piece of bone
A. Arthroscopy E. Deformity
B. Clinical assessment
C. MRI 20. Carcinoma of rectum is present within 7 cm of anal
D. Ultrasound verge, and the best management option is:
E. X-ray A. Abdominoperineal resection of rectum
B. Radiotherapy
C. Extended left hemicolectomy
D. Anterior resection
E. Chemotherapy
21. In a diagnosed case of gastric outlet obstruction, A patient after a car accident comes to the emergency
The expected metabolic abnormality can be corrected unable to breathe, air entry on auscultation is absent:
by administration of? What is the diagnosis?
A. Hypertonic saline or Glucose Ans: Tension Pneumothorax
B. Normal saline with added potassium
C. Plasma expanders 26. A 13 years old boy complains of pain in lower right
D. Ringer's lactate Hartman solution hand side of abdomen. He has high grade fever and
E. Total parenteral nutrition suffered from sore throat last week. On examination he
is tender in right iliac fossa and the tenderness shifts?
22. 60 years old male with 02 years history of passage What is the likely diagnosis?
of urine from umbilicus best signifies? A. Mesenteric adenitis
A. Acute umbilical infection B. Acute appendicitis
B. Chronic umbilical infection C. Henoch schonlein purpura
C. Patent urachus D. Tuberculous adenitis
D. Umbilical malignancy E. Right sided lobar pneumonia
E. Vitelo intestinal fistula
27. Breast conservative surgery is indicated in:
23. A 70 year old man presents with 6 years history of A. Central breast lump less than 10 cm
pruritus ani, the Itching and discomfort has Increased B. Central breast lump less than 05 cm
during last few months. There is neither rectal bleeding C. Central breast lump less than 2.5 cm
nor pain Physical signs suggest chronic dermatitis, and D. Peripheral breast lump more than 7cm
an irregular ulcer on the left anal margin.what is the best E. Peripheral breast lump less than 3 cm.
next step In management:
A. Wide local excision 28. 20 years old woman has reducible hernia below
B. Steroid ointment inguinal ligament. Which of the following is appropriate
C. Nitroglycerine ointment management?
D. Internal sphincterotomy A. Observation & follow-up in surgical clinic for 6
E. Biopsy months
B. Observation & follow-up in surgical clinic of she
24. A 48 years female presented with obstructive develop symptoms
jaundice. U/s abdomen revealed dilated CBD with C. Elective surgical repair
presence of single stone in it. What is the most D. Emergent surgical repair
preferred technique for Its removal E.Emergent surgical repair with exploration of small
A. ERCP & removal of stone bowel.
B. Laparoscopic CBD exploration
C. Percutaneous choledochoscopy 29. A middle aged obese male complaining of
D. Percutaneous transhepatic cholangiography retrosternal burning, epigastric pain radiating to back
E. Surgery and choledochotomy and regurgitation. Which of the fowling is gold standard
to diagnose the problem. Or GERD gold standard for
25. A young male was driving his car when he sustained investigation
head on collision with a truck: when received in the ER, A. Barium swallow
he was in a state of shock & having difficulty in B. Chest X-ray
breathing but his airway was intact. His neck veins were C. Manometry
distended. The most probable diagnosis is? D. 24 hour ph monitoring
A. Tension pneumothorax. E. Upper G-I endoscopy
B. Open pneumothorax
C. Pericardial tamponade 30. A pt of crohn's disease after surgery prolonged
D. Extradural bleeding. recovery time, preferred route of nutrition
E. Subdural hemorrhage Ans Parental
or
31. Wound healing is delayed in diabetic patient
Ans Peripheral neuropathy
32. Patient with Pancreatitis Carcinoma have pain what 45. Swelling of mouth, tender and red tongue, with
epidural opioid is good for pain relief tongue displaced upward: What is the diagnosis?
Ans: Ludwig’s Angina
Opioid in epidural analgesia / anesthesia
Ans Fentanyl or morphine? 46. Common organism in primary peritonitis
Ans E.coli
33. General anesthesia is termed as
Ans Amnesia + analgesia + MR 47. Dance Sign in Gastric outlet obstruction
Ans Suspicious splash
34. Preoperative cardiac risk assessment can be done
through which of the following? 48. Post Ercp-Pancreatitis prophylaxis
Ans Revised Cardiac Risk Index Ans Metronidazole

35. Single nodule in thyroid gland which is hard in 49. Ischemic orchitis after inguinal hernia Repair
consistency: How much (%) risk of malignancy? Ans 2 tight ligation or end artery ligation?
Ans Up to 15% (5-15%)
50. Liver Abscess cyst have Eosinophilia level
36. Regarding wound grading according to the Ans Level III?
Southampton wound system, in a patient with
hemorrhagic-serous discharge coming from one wound,
what is the grading?
Ans Grade III

37. Brain injury is suspected in a patient of RTA. What is


the initial step?
Ans Secure the Airway

38. Platelets are stored at what temperature?


Ans 20-24°C

39. Ebb phase of injury is associated with?


Ans: Decreased BMR

40. Joint space narrowing and osteophytes are seen in?


Ans: Osteoarthritis

41. History of hot, swollen, tender knee joint: Patient


might be suffering from?
Ans: Septic Arthritis

42. Another name of tennis elbow?


Ans: Lateral Epicondylitis

43. Thoracic rupture T10 to T12 movement most


common malignancy followed
Ans: Breast Cancer

44. Which technique is used in vacuum-assisted closure


of wounds?
Ans: Negative Pressure Wound Therapy
47 Prof (SURGERY 1) 6. A 55 year old lady underwent a left sided modified
radical mastectomy (MRM) and axillary clearance for
1. A 30 year old male presented to a surgical breast cancer. After the operation she was found to
emergency with H/O fall from bike. O/E the right have a winging scapula. She is otherwise well. What is
shoulder joint is tender and the patient is unable to the most likely cause of this condition?
perform movements at the shoulder joint. Which A. Patient has suffered a TIA
following types of shoulder dislocation is most common? B. Damage to the thoracodorsal nerve during the
A. Anterior surgery
B. Posterior C. Damage to the long thoracic nerve during the surgery
C. Superior D. Damage to the serratus anterior muscle during the
D. Lateral surgery
E. Medial E. Dislocated scapula

2. Lymphedema of lower limb most common cause 7. A patient underwent open reduction and internal
A. Deep venous thrombosis fixation of fracture femur, complaining of severe limb
B. Filariasis pain that is felt even worse on passive stretching of
C. Fungal infection muscles. Limb found swollen and tense with altered
D. Primary lymphatic malignancy sensations distally. Most likely diagnosis is:
E. Superficial thrombophlebitis A. Femoral artery occlusion
B. Compartment syndrome
3. The most common cause of increased serum calcium C. Development of localized hematoma
levels in hospitalized patients is: D. Missed muscle injuries
A. Primary hyperparathyroidism. E. Prosthesis displacement
B. Immobilization.
C. Malignancy. 8. Intermittent claudication is a cramp-like pain felt in the
D. Vitamin D excess. limb muscles, which of the following statements is
E. Myeloma. appropriate?
A. The pain felt in the lower limb at rest
4. During submandibular salivary gland surgery which of B. The pain felt on taking the first step
the following nerves may likely be at risk C. The pain brought on by walking and relieved by
A. Facial nerve standing still
B. Glossopharyngeal nerve D. The foot is often cold and paralyzed
C. Lingual nerve E. Ulceration and gangrene of toes are usually
D. Trigeminal nerve associated with even mild cases
E. Vagus nerve
9. Buerger's disease is also called as:
5. Which of the following is most frequent site of Bed A. Thromboangiitis obliterans
sores (decubitus ulcers) B. Thrombophlebitis
A. Sacrum C. Disease of large veins
B. Greater trochanter D. More common in middle aged females
C. Ischium E. Common in diabetics
D. Heal
E. Occiput 10. Common cause of oral cancer
A. Tobacco smoking
B. Bettie nut chewing
C. Alcohol
D. Dental ulcer
E- Gingivitis
11. Varicose veins 17. A 45 years old lady presents to OPD with multiple
A. Are the tortuous dilated pampiniform plexus of veins painless swellings in neck. On examination multiple
B. Commonest site is scrotum lymph nodes are palpable in submental and
C. Pregnancy & pelvic tumors are common factors submandibular region of the neck. While discussing final
D. Stockings are contraindicated in old patients with year students during ward round about anatomical
varicose veins levels of neck lymph nodes, these lymph nodes lies at
E. Trendelenburg’s operation has no role which level?
A. Level I
12. The diagnosis of primary hyperparathyroidism is B. Level II
most strongly suggest C. Level III
A. Increased serum acid phosphatase D. Level IV
B. Increased serum calcium E. Level V
C. Decreased urinary calcium
D. Increased alkaline phosphatase 18. A 50 years grossly obese female was planning
E. Increased parathyroid hormone level surgery for extra abdominal mass, which may take two
hours. Which of the following pro encountered in obese
13. A young boy of 23 year age sustained head injury patients undergoing
by road traffic accident by bike he should be evaluated A. Deep venous thrombosis(DVT)
by B. Obstruction and Sepsis
A. X-ray skull C. Poor wound healing
B. Lumbar puncture D. Hypothermia
C. Observation by Glasgow coma scale E. Risk of renal failure
D. C.T scan brain
E. Carotid angiography 19. Extradural haematoma is a neurosurgical
emergency Typically this condition occurs due to
14. Concealed haemorrhage may become revealed in: damage to the
A. Ruptured spleen A. Posterior cerebral artery
B. Ruptured liver B. Middle meningeal artery
C. Ruptured Kidney C. Cortical vessels
D. Ruptured ectopic gestation/pregnancy D. Anterior cerebral artery
E. Cerebral haemorrhage E. Posterior cerebellar artery

15. Which one of the following methods of sterilization Is 20. The best investigation of choice for assessment of
Ideal for surgical instruments blunt abdominal trauma in unstable patient is:
A. Boiling A. CT scan abdomen
B. Spirit B. Diagnostic peritoneal lavage (DPL)
C. Pyodine pollution C. MRI
D. Sterilization by autoclave D. Plain X-Ray abdomen
E. Hot air oven E. U/S abdomen (focused assessment with sonography
in trauma FAST)
16 . A 42 years female having a non healing ulcerated
wound over her left leg. Past history of burn. O/E there 21 . A 40 years old woman is found to have a 2 cm
is a wide scar over the previous burns area and slightly tender cystic mass in her left breast. Her axilla is
Irregular ulcer with raised and indurated base. What is clear. What course would you follow?
the most likely diagnosis? A. Reassurance and reexamination in immediate
A. Basal cell carcinoma post-menstrual period.
B. Marjelin's ulcer B. Immediate excisional biopsy.
C. Malignant melanoma C. Aspiration of mass for cytological analysis.
D. Varicose ulcer D. Fluoroscopically guided needle biopsy.
E Tubercular ulcer E. Mammography and re-evaluation.
22. A patient with trauma + hospital prolonged route for (SURGERY 2)
nutrition
Ans Total parenteral nutrition 1. A glandular metaplasia of the lower oesophageal
mucosa resulting from chronic gastroesophageal reflux
23. Girl present with 3cm solitary nodule investigation is known as
Ans FNAC A. Tylosis
B. Leukoplakia
24. Intertrochanteric fracture of femur C. Achalasia
Ans Reduction and internal fixation D. Barrett s oesophagus
E. Plummer vinson syndrome
25. Risk of breast cancer
Ans Strong family history 2. Tumor marker in hepatocellular carcinoma is:
A. Carcinoembryonic antigen (CEA)
26. Early sign of sepsis B. Alkaline phosphatase
Ans Tachycardia C. Alpha fetoprotein
D. B-HCG
27. Delayed wound healing E PSA
Ans Infection
3. A 22 years female has recurrent attacks of severe
28. Female with thyroid nodule for 20 years cramp like pain in rectum at night, pain subsides
Ans Total thyroidectomy spontaneously within few minutes. What is probable
Diagnosis:
29. COPD patient going for elective surgery what A. Hypertrophied anal papilla
preoperative measures are taken B. Proctalgia fugax
Ans stop smoking 4-6 weeks C. Perianal abscess
D. Fissure in ano
30. Nutritional status of patient at 10th day of surgery E. Worm infestation
Ans Serum albumin
4. A 52-year-old female presents with vague right
31. Breast stage 2B Rx inguinal pain. On examination, a mass is palpable in the
Ans Conservative inguinal region. It is painless and well-defined. What is
the most likely diagnosis?
32. Regarding echinococcus granulosus A. Appendicular lump
Ans Rupture of cyst cause anaphylactic shock B. A mesenteric cyst
C. Tubo-ovarian mass
33. Necrotizing fasciitis D. Right inguinal abscess
Ans Its polymicrobial and synergetic infection E. Enlarged inguinal lymph node

34. Regarding Frey’s syndrome 5. A middle aged female presents with history of weight
Ans Starch iodine test loss, abdominal mass and enlargement of left
supraclavicular node. You teach accompanying interns
35. Regarding massive blood transfusion and residents about Virchow's node that it is a
Ans Coagulopathy prominent lymph node in the left supraclavicular fossa in
response to malignant disease spread from some
primary lesions. Which of the following primary lesion
have association with Virchow’s node?
A. Adrenal tumour
B. Colo-rectal carcinoma
C. Gastric carcinoma
D. Renal cell carcinoma
E. Uterine carcinoma
6. Cutaneous ecchymosis is important clinical finding In 11. A 30 years male is having fever & tender
case of acute hemorrhagic pancreatitis seen in 1% of hepatomegaly. U/s abdomen suggests a large (10x7cm)
cases, Cullen's sign is discoloration of skin Involving liver abscess. Which is the most likely causative agent?
which area of body A. Echinococcus granulosus
A. Around umbilicus B. Entamoeba Histolytica
B. Flanks C. Staphylococcus
C. Hypogastrium D. Streptococcus
D. Legs E. Pseudomonas
E. Upper thighs
12. Meckel's diverticulum is Seen in
7. Commonest site for colorectal cancer. A. 2% of patient
A. Caecum B. 2 cm in length
B. Descending colon C. Located 2 inches proximal to ileocecal junction
C. Transverse colon D. Harmless
D. Recto sigmoid junction E. Main cause of appendicitis
E. Rectum
13. A 22 years old lady presented in OPD with the C/O
8. A 35 years old male patient farmer by occupation bleeding per rectum after defecation and something
presented to surgical OPD with painless swelling right coming out of anus which reduced spontaneously.
hypochondrium. U/S shows a 7x10cm hydatid cyst in Determine the degree of naemorrhelés?
the right lobe of the liver. What is the preferred A. 1 degree
treatment option for this patient? B. 2 degree
A. Immunotherapy C. 3 degree
B. Medical treatment with albendazole D. 4 degree
C. PAIR therapy E. 5 degree
D. Serial ultrasound examination
E. Surgical excision 14. Following condition is mostly benefited by
splenectomy:
9. A middle aged patient c/o severe abdominal pain with A. Erythroblastosis fetalis
past history of dyspepsia. His x-ray chest demonstrates B. Hodgkin's lymphoma
free air under right dome of diaphragm. Which of the C. Splenic abscess
following Is the most likely possibility D. Idiopathic thrombocytopenic purpura
A. Acute cholecystitis E. Tropical splenomegaly
B. Acute pancreatitis
C. Perforated appendicitis 15. The best conservative treatment of anal fissure is
D. Perforated duodenal ulcer A. High fiber diet
E. Ruptured liver abscess B. Stool softness
C. 5% xylocaine cream tropically
10. A 25-year-old male suffered blunt trauma to the D. Chemical sphincterotomy
chest. In the ER, he presented with shortness of breath E. lateral internal sphincterotomy
and cyanosis. On examination, no breath sounds were
heard on the right side, and the percussion note was
resonant. The most likely diagnosis is:
A. Spontaneous pneumothorax.
B. Cardiac tamponade.
C. Cardiac contusion.
D. Tension pneumothorax.
E. Pericardial effusion.
16. A 19 years old football player while playing a match 21. Oesophageal varices usually present with the acute
developed sudden pain in scrotum. He presented to onset of a large volume haematemesis, caused by
emergency after 4 hours of onset of pain. O/E, the portal hypertension In liver cirrhosis. After resuscitation
scrotum is well developed. Right testis is not palpable at the best method of arresting bleeding is:
the base of scrotum but a palpable mass just medial to A. Blood transfusion
the groin crease. What is a provisional diagnosis? B. Endoscopic band ligation combined with
A. Idiophatic scrotal edema vasoconstrictors.
B. Testicular torsion C. Splanchnic vasoconstrictors by vasopressin
C. Hematocele D. Transjugular intra hepatic portosystemic stent shunts
D. Epididymo-orchitis (TIPSS)
E. Undescended testis E. Vitamin-K 10 mg intravenously

17. An 50 years old male presented with deep 22. A 52 years obese woman presents with colicky with
Jaundice, cachexic, and pale, ERCP and MR abdominal pain and vomiting. She has recurrent history
suggestive of early stage of periampullary carcinoma. of pain in RHC and dyspepsia. X-ray abdomen reveals
What is the best treatment.eption? multiple air-fluid levels and air in biliary tree. What is the
A. Conservative most likely diagnosis?
B. exploration of CBD A. Abdominal adhesions
C. Pancreatectomy B. Abdominal lymphosarcoma
D. Stenting C. Carcinoma of the right colon
E. Whipple's procedure D. Gallstone ileus
E. Ileo-caecal tuberculosis
18. Rectal prolapse.
A. In children it is associated with constipation 23. 60 year male presented to emergency Department
B. In children it is usually full thickness with painful irreducible swelling in Rt inguino scrotal
C. Surgery is necessary for full thickness region associated with vomiting and constipation since
D. In children conservative treatment has no role five day,O/E swelling is tender and cough impulse
E. Abdominal rectopexy may lead to severe diarrhea negative. What is diagnosis?
A. Acute intestinal obstruction
19. Carcinoma of rectum is present within 7 cm of anal B. Epididymoorchitis
verge, and the best management option is: C. Hydrocele
A. Abdominoperineal resection of rectum D. Strangulated inguinal hernia
B. Radiotherapy E. Torsion of testis
C. Extended left hemicolectomy
D. Anterior resection 24. The most common site of intussusception in children
E. Chemotherapy is?
A. Ileocolic
20. A 25 years male presented with attacks of B. Colocolic
abdominal pain and intermittent diarrhea associated C. Ileoileal
with weight loss and evening pyrexia since 2-3 years. D. Coloanal
Clinical examination reveals mild anaemia and a E. Colorectal
nontender mass palpable in RIF. DRE unremarkable.
What Is the most likely diagnosis? 25. The mesenteric cysts occurs in the mesentery of the
A. Appendicular mass small intestine or the colon are of various types Which
B. Actinomycosis of the following is the most common variety, frequently
C. Carcinoma of the caecum occurring in the mesentery of
D. Crohn's disease A. Chylolymphatic cyst
E. Ileo-caecal tuberculosis B. Dermoid cyst
C. Hydatid cyst
D. Enterogenous cyst
E. Urogenital cyst
26. 30 years old lady complains about painless bleeding
per rectum for past one month. On examination she is
severely anaemic with third degree haemorrhoids.
What will be the first step in management in this lady:
A. Band ligation
B. Correction of anaemia
C. Discharge and follow up
D. Emergency surgery
E. T Bandage And bedrest

27. Gallbladder
Ans CA 19-9

28. Prostate
Ans adenocarcinoma

29. Undescended testis


Ans MRI

30. Discoloration around flank


Ans Grey turner sign

31. Gallstone in pregnancy


Ans Dietary modification or Observation?

32. Diagnostic laparoscopy


Ans Stable patient?

33. NSAID peritonitis


Ans Perforated PUD

34. Weight loss attenuated bowel habits


Ans Ca-cecum

35. PSA increased


Ans Prostate carcinoma
46 Preprof 5. A 50 year old female presented with altered bowel
habits since 3 years. Sigmoidoscopy and biopsy
1. A 42 years female having a non healing ulcerated revealed carcinoma at the rectosigmoid junction. What
wound over her left leg. Past history of burn. O/E there is the most frequent organ presumed to be involved in
is a wide scar over the previous burns area and distant metastasis?
Irregular ulcer with raised and indurated base. What is A. Brain
the most likely diagnosis? B. Kidney
A. Basal cell carcinoma C. Liver
B. Marjelin's ulcer D. Lung
C. Malignant melanoma E. Ovary
D. Varicose ulcer
E Tubercular ulcer 6. A 70 years old man presented with retention of urine.
He has recent H/O increased urine frequency, urgency
2. A 50 year old female having swelling in front of neck & incomplete bladder emptying. O/E bladder is
for five years. Recently developed dyspnea & dysphagia palpable. Most common cause of retention in this age
for six months, O/E swelling moving with deglutition, is?
veins of face, neck and anterior chest wall are A. Ureteric stone
engorged, cervical lymph nodes are not palpable. What B. Urethral stone
is most likely diagnosis? C Benign prostate enlargement
A simple multi nodular goiter D. Stricture of urethra
B. Ca: Thyroid E. Neurogenic bladder
C. Simple diffuse goiter
D. Retrosternal goiter 7. A 24 years old man presented with flank pain &
E. Primary toxic goiter vomiting. U/S shows right gross hydronephrosis & 2 cm
renal stone. X-ray IVP shows the right renal stone but
3. A keloid scar is defined as excessive scar tissue that no contrast. Most likely a test to confirm the function of
extends beyond the boundaries of the original incision the right kidney?
or wound. Which of the following treatment options have A. DMSA
high rates of recurrence? B. CT scan KUB
A. Surgical excision C. MRI
B. Intralesional steroid injection D. MAG-3
C. Laser therapy E. CT scan urography
D. Pressure elastic garments
E. Silicone gel sheeting 8. A patient suffers burn injury to the anterior aspect of
abdomen, left upper limb and left lower limb. What is the
4. A 22 years old lady presented in OPD with the C/O total body surface area involved?
bleeding per rectum after defecation and something A. 10%
coming out of anus which reduced spontaneously. B. 27%
Determine the degree of naemorrhelés? C. 36%
A. 1 degree D. 44%
B. 2 degree E. 50
C. 3 degree
D. 4 degree
E. 5 degree
9. A12 year old woman presents to the emergency 13. A 65 years old patient presents with complete rectal
department with a chief complaint of severe left upper prolapse. You are planning surgery for him. "which of
quadrant (LUQ) pain after being punched by her the following procedure is associated with lowest risk of
husband. Her blood pressure is 110/70 mm Hg. puke is recurrence.
100 bpm, and respiration rate is 24 breaths per minute. A. Thiersh procedure
The best means to establish a diagnosis is which of the B. Delorme's procedure
following? C. Altemelers procedure
A. FAST D. Abdominal rectopexy
B. Physical examination E. Starr procedure
C. CT of the abdomen
D. Peritoneal lavage 14. diagnosis of thyroid diseases are among those fine
E. Upper gastrointestinal (GI) series needle aspiration cytology [FNAC) is the investigation of
choice for the diagnosis of ?
10. A female presented with complaints of something A. Diffuse thyroid swelling
coming out of anus which goes inside itself with no B. Discrete thyroid swelling.
manipulation. She also has complaints of painless C. Grave's Disease
per-rectal bleeding. What is management in her case? D. Multi nodular goiter
A. Stapled hemorrhoidectomy E. Hashimoto's Thyroiditis
B. Almond oil injection
C. Banding 15. A young female about 30 years old presented with
D. Ice therapy diffuse goiter In Front of neck. The most important factor
E. Observation in the development of endemic goiter is?
A. Dietary deficiency of iodine
11. An elderly male diabetic patient presents with B. Dyshormonogenesis
perianal discharge on examination there were multiple C. High altitude
perianal fistulas. Which of the following investigations D. High iodine intake
should be performed? E. Use of goitrogens
A. Ultrasound
B. CT scan 16. A 30 years female presented with discrete thes
C. MRI swelling her in lower right lobe since one year Following
D. Fistulogram clinical examination ultrasound and cytological
E. Biopsy assessment the lesion is characterized as malignant.
Which of the following is the most common malignancy
12. A 44 years male presents in OPD with complaint of frequently found in the thyroid gland?
severe anal pain and some bleeding PR that Farts A. Anaplastic carcinoma
during defecation and persists for many minutes B. Lymphoma
afterwards. On examination anal tone is creased and C. Medullary carcinoma
pain on attempt to insert finger nits the examination. D. Follicular carcinoma
What is your diagnosis? E. Papillary carcinoma
A. Anal fissure
B. perianal hematoma 17. A 50 year old male became dehydrated after
C. Pilonidal sinus episodes of persistent vomiting later on as confirmed by
D. Perianal abscess lab tests, He developed hypokalemic metabolic
E. Hemorrhoids alkalosis. Which of the following 1/ fluid replacement is
most appropriate for this condition?
A. Dextrose water 5%
B. Dextrose water 10%
C. 0.9% normal saline with KCl
D. Plasma expander
E. Ringer's lactate
18. A 60 year old male developed fever after laparotomy 22. A thin built 50 years old lady presented with a
within 48 hours. What is the most common cause of painless epigastric mass, which on CT scan appears to
fever during his early post-operative period be arising from the tail of pancreas, having
A. Atelectasis lymphovascular invasion and adjacent fat stranding. No
B. Deep venous thrombosis metastasis in detected. What will be he preferred
C. Superficial thrombophlebitis treatment modality for this patient?
D. Urinary tract infection A. Distal pancreatectomy
E. Wound infection B. Endoscopic deroofing
C. Pancreatectomy
19. After a clinical case presenting on multiple neck D. Percutaneous aspiration
swelling discussion started between residents regarding E. Whipple's procedure
anatomic level of left supraclavicular matnode. This
lymph node is clinically considered Important is staging 23. A 50 years old patient having uncontrolled diabetes
various malignancies. What anatomic level this lymph presented in emergency with right upper quadrant pain
node lies & jaundice. U/S suggestive of acute cholecystitis. What
A. level I will be the best treatment option?
B. level II A. Cholecystectomy
C. level III B. Conservative treatment
D. level IV C. Laparoscopic cholecystectomy
E. level V
24. During laparoscopic cholecystectomy the surgeon
20. A 45 year old male was operated on for colonic wants to identify calot's triangle anatomy in order to
surgery. He developed fever and wound infection after 5 avoid damage to which of the following structures?
days postoperatively despite broad spectrum antibiotics. A. CBD
Most appropriate next step of management of such type B. Cystic duct
of wound infection would be? C. Cystic artery
A. Frequent change of wound dressing D. Cystic vein
B. Send blood culture E. Duodenum
C. Send CBC and start antipyretics
D. Switch over 25. An 8 years old male child is brought to OPD by his
E. Wound should be opened parents with complaint of right testis going in and out of
scrotum. O/E scrotum is well developed. What should
21. An old age lady having prior hospital admission for be the best strategy?
acute pancreatitis. She has recently developed A. Orchiectomy
epigastric mass which is non-tender and her TLC count B. Orchidopexy
is 5000x10' U/L. What could be the possible diagnosis? C. Observation with follow up
A. Duodenal cancer D. Beta-HCG levels
B. Pancreatic abscess E. None of the above
C. Pancreatic pseudocyst
D. Pancreatic necrosis 26. A 19 years old football player while playing a match
E. Pancreatic cancer developed sudden pain in scrotum. He presented to
emergency after 4 hours of onset of pain. O/E, the
scrotum is well developed. Right testis is not palpable at
the base of scrotum but a palpable mass just medial to
the groin crease. What is a provisional diagnosis?
A. Idiophatic scrotal edema
B. Testicular torsion
C. Hematocele
D. Epididymo-orchitis
E. Undescended testis
27. A 30 year old male presented to a surgical 31. A young male patient has been brought to
emergency with H/O fall from bike. O/E the right emergency with history of blunt trauma of left chest. O/E
shoulder joint is tender and the patient is unable to breathing sounds are absent and dullness on
perform movements at the shoulder joint. Which percussion noted over the left side of chest. What is the
following types of shoulder dislocation is most common? most likely diagnosis?
A. Anterior A. Left haemothorax
B. Posterior B. Cardiac tamponade
C. Superior C. Left simple pneumothorax
D. Lateral D. Left diaphragmatic rupture
E. Medial E. Rt: Tension pneumothorax

28. A 50 years old female presented with complain of 32. An elderly female is a diagnosed case of intestinal
scapula becoming more prominent on shoulder obstruction. You advise to put an NG tube and start
protraction. She underwent a modified mastectomy for Intravenous fluid. In which of the following condition NG
left breast carcinoma few years back. Damage to long tube is contraindicated?
thoracic nerve is suspected. Which of the following is A. Gastric perforation
consistent with in long thoracic nerve? B. Diaphragmatic rupture
A. Loss of adduction C. Open depress skull fracture
B. Wrist drop D. Fracture of cervical spine
C. Winging of scapula E. Fracture of cribriform plate
D. Claw hand
33. a middle aged male patient has been brought to the
29. After prolonged exposure to cold temperature in accident & emergency department having significant
Muree a young tourist brought in emergency with chest trauma. Chest tube has been asserted. In which
peripheral cyanosis Diagnosis as frost bite has been of the following conditions chest
made. Which of the following is recommended method tube insertion is indicated?
for initial treatment of Frostbite? A. Pneumothorax
A Vasodilators B. Pneumomediastimum
B. Anticoagulants C. Lung contusion
C. Warm (40°) water D. Diaphragmatic rupture
D. Padding and elevation E. Cardiac tamponade
E. Application of heat from a hair dryer
34. A 44 years old female patient with uncontrolled DM
30. A young man sustains a rifle wound to the mid of presented to surgical OPD with an U/S report that the
abdomen, is brought promptly to the emergency carried out for pain in left lumbar region U/S shows a
department by pre-hospital personnel. His skin is cool single 3 cm stone in gall bladder that was Incidentally
and diaphoretic and his systolic blood pressure is 58 diagnosed. Though she is asymptomatic for gallstones
mmhg. Warmed crystalloid fluids are initiated but no but still she requires cholecystectomy because.
improvement in his vital sign has been observed. Next A. She will develops symptoms later on
most appropriate step is to perform. B. She has more chances of malignancy
A. Laparotomy C. She is prone to develop complications of
B. CT Abdomen Cholecystitis
C. Diagnostic laparoscopy D. Stone may slip to CBC causing obstructive jaundice
D. Abdominal ultrasound E. Gallbladder may perforate later
E. Diagnostic peritoneal lavage
35. 45 year old male patient admitted in surgical ward 39. A 50 years old female presented to surgical OPD
with complaint of painful jaundice since 2 weeks On with C/O painless lump in the upper outer quadrant of
examination tenderness positive in right upper right breast. On examination 5x 6 cm lump palpable that
abdomen, Blood CBC shows raised TLC. U/S abdomen is hard in consistency, mobile and few lymph nodes
shows cholelithiasis, dilated CBD with single stone were palpable in axilla. Next step in management of this
impacted in CBD. Which is the most suitable treatment patient requires.
option in this patient? A. Incisional biopsy
A. Antibiotics B. Mammography
B. Cholecystectomy C. MRI
C. Choledochotomy D. Trucut biopsy
D. ERCP E. MAM
E. Nasobiliary drainage
40. A 35 years old male patient farmer by occupation
36. A young female patient presented to OPD wis presented to surgical OPD with painless swelling right
diffuse thyroid swelling along with signs thyrotoxicosis hypochondrium. U/S shows a 7x10cm hydatid cyst in
Provisional diagnosis of Grave disease made. Indication the right lobe of the liver. What is the preferred
for thyroid scanning thyroid swelling is: treatment option for this patient?
A. Inflammatory swellings A. Immunotherapy
B. Midline neck swelling B. Medical treatment with albendazole
C. Malignant swellings C. PAIR therapy
D. Recurrent swellings D. Serial ultrasound examination
E. Toxicity with nodular swelling E. Surgical excision

37. 20 years old girl came to OPD with complain of 41. A 24 year female patient presented to surgical OPD
painless swelling at outer 1/3 of eyebrow since with pain right upper abdomen & fever. She had a
childhood. On examination it is soft, non tender skin recent history of diarrhea as well. O/E tender
overlying swelling is pinchable. Most diagnosis. hepatomegaly appreciated, blood CBC shows normal
A. Cold abscess TLC. U/S shows cystic swelling in the liver. Most likely
B. Dermoid cyst diagnosis is?
C. Hydatid cyst A. Amoebic liver abscess
D. Lipoma B. Hemangioma liver
E. Sebaceous cyst C. Hydatid cyst
D. Liver abscess
38. A young patient brought to the emergency E. Simple liver cyst
department with dehydration, altered level of
consciousness, vomiting, abdominal distension and 42. 3 weeks old child presents with non-bilious projectile
constipation for 3 days. His pulse was 120 beats per vomiting what diagnosis is suspected keeping in mind
min, BP-90/60 mm Hg and X-ray abdomen shows the age of child:
multiple air fluid levels which are suggestive intestinal A Congenital hypertrophic pyloric stenosis
obstruction Resuscitations started with I/V fluid. Most B. Duodenal atresia
important Indicator of effective resuscitation in this C Acute appendicitis
patient is. D. Intussusception
A. Blood pressure E. Volvulus
B. Pulse rate
C. Regain of consciousness 43. In laparoscopic cholecystectomy the preferred gas
D. S. electrolytes used for creating a pneumoperitoneum is:
E. Urine output A. Carbon dioxide
B. Carbon monoxide
C. Helium
D. Nitrous oxide
E. Oxygen
44. An 50 years old male presented with deep 49. Cutaneous ecchymosis is important clinical finding
Jaundice, cachexic, and pale, ERCP and MR In case of acute hemorrhagic pancreatitis seen in 1% of
suggestive of early stage of periampullary carcinoma. cases, Cullen's sign is discoloration of skin Involving
What is the best treatment.eption? which area of body
A. Conservative A. Around umbilicus
B. exploration of CBD B. Flanks
C. Pancreatectomy C. Hypogastrium
D. Stenting D. Legs
E. Whipple's procedure E. Upper thighs

45. A 30 year old female presented in emergen room 50. A 60 years male presented with H/O weight loss &
with hypovolemic shock after blunt trauma of the dull pain in Right hypochondrium. On clinical
abdomen. An emergency USG of abdomen shows examination he is looking pale & Liver is palpable. U/S
splenic tear. Which of the following is to done? abdomen suggestive of complex mass in right lobe of
A. CT scan of the abdomen liver could be hepatoma. Which of the following is the
B. Chest x-ray tumor marker for hepatocellular carcinoma:
C. Diagnostic lavage of peritoneal cavity before A. Carcinoembryonic antigen (CEA)
proceeding B. Alkaline phosphatase
D. Immediate surgery C. Alpha fetoprotein
E. Monitor patient to assess for progression D. B-HCG
E. PSA
46. A 45 years female presented with severe epigastric
pain, radiating to the back since 24 hours. She has past 51. A young adult man is suffering from severe diarrhea
H/O dyspepsia. Her serum amylase level 2200 U/lit. & vomiting leading to dehydration & shock. Which
She is diagnosed with acute pancreatitis Which of the resuscitation fluid is best for the management of shock.
following is the most common of acute pancreatitis in A. 5% dextrose water
Asian Countries? B. 10% dextrose water
A. Alcohol C. Plasma expander colloid
B. Autoimmune D. Ringer lactate
C. Gallstones E. Whole blood
D. Steroids
E. Trauma 52. Gas gangrene wound infections are associated with
severe local pain and crepitus. Which organisms is the
47. 20 years old boy presented to the emergency most appropriate for development of this
department, he was hit against the motorcycle handle. Conditions.
He was in shock and urgent Investigation of choice for A. B-haemolytic streptococci
blunt trauma at in unstable patient is: B. CL Perfringens
A. CT scan C. CL. Tetanus
B. Diagnostic peritoneal lavage D. E-coli
C. MRI E. Klebsiella
D. USG
E. X-ray abdomen 53. In a diagnosed case of gastric outlet obstruction,
The expected metabolic abnormality can be corrected
48. Endocrine organ 20-40% the exocrine part. by administration of?
Functional unit al endocrine part in islets of langerhans. A. Hypertonic saline or Glucose
Which endocrine cells are responsible for production of B. Normal saline with added potassium
Insulin? C. Plasma expanders
A. A cells D. Ringer's lactate Hartman solution
B. B cells E. Total parenteral nutrition
C. D cells
D. E cells
E. Pcells
54. A 50 years grossly obese female was planning 59. Intermittent claudication is a cramp-like pain feel in
surgery for extra abdominal mass, which may take two the limb muscles, which of the following statement is
hours. Which of the following pro encountered in obese appropriate
patients undergoing A. The pain felt in lower limb at rest
A. Deep venous thrombosis(DVT) B. The pain felt on taking the first step
B. Obstruction and Sepsis C. The pain brought on by walking and relieved by
C. Poor wound healing standing still
D. Hypothermia D. The foot is often cold and paralyzed
E. Risk of renal failure E. Ulceration and gangrene of toes are usually
associated with even mild cases
55. 22 years college student notices bulge in right grain,
which is accentuated by coughing, but easily reducible. 60. A middle aged patient c/o severe abdominal pain
Which of the following herr follows the path of spermatic with past history of dyspepsia. His x-ray chest
cord with cremasteric muscle? demonstrates free air under right dome of diaphragm.
A. Femoral Which of the following is the most likely possibility
B. Direct Inguinal A. Acute cholecystitis
C. Indirect inguinal B. Acute pancreatitis
D. Spigellion C. Perforated appendicitis
E. Para umbilical D. Perforated duodenal ulcer
E. Ruptured liver abscess
56. 70 years old female with past history of mid
emergency laparotomy followed by postoperative 61. A 30 years male is having fever & tender
wound infection has developed expansile swelling over hepatomegaly. U/s abdomen suggests a large (15×15
operative site. What is the most probable diagnosis? cm) liver abscess. Which of the following is the most
A. Recurrent hernia likely causative agent for this condition?
B. Incisional hernia A. Echinococcus granulosus
C. Abdominal hernia B. Entamoeba Histolytica
D. Dorsal hernia C. E. Coli
E. Spigellion hernia D. Klebsiella
E. Pseudomonas
57. A 65 years male C/O dull pain in upper abdomen
associated with weight loss. On CT scan abdomen 62. A 23 years male is having splenomegaly and
diagnosed as CA head of pancreas. The commonest tendency to bleed upon minor injury. Which of the
clinical feature of carcinoma head of pancreas is: following disorders are indicated for splenectomy:
A. Epigastric pain A. Acute leukemia
B. Anorexia and malaise B. Chronic myeloid leukemia
C. Thrombophlebitis C. Idiopathic thrombocytopenic purpura
D. Obstructive jaundice D. Polycythaemia
E. Mass in the abdomen E. Pernicious anemia

58. During submandibular salivary gland surgery which 63. 45 years female is c/o pain RHC, U/s abdomen
of the following nerves may likely be at risk revealed cholelithiasis. Which is the best procedure of
A. Facial nerve choice for this condition
B. Glossopharyngeal nerve A. ERCP
C. Lingual nerve B. Lithotripsy
D. Trigeminal nerve C. Laparoscopic cholecystectomy
E. Vagus nerve D. Open cholecystectomy
E. Use of stone dissolving agents
64. Young man about 35 years brought in emergency 68. The compartment syndrome of lower limb occurs
with severe haematemesis O/E he is looking pale in a after an injury or burn, resulting in blood or inflammatory
state of shock. The commonest cause of the upper fluid accumulated in the compartment causing pain,
gastrointestinal hemorrhage is: pallor and paresthesia. What is the best urgent
A. Avascular malformation treatment for this condition?
B. Mallory- Weiss tear A. Amputation
C. Peptic ulcer B. Elevation of leg
D. Esophageal varices C. Fasciotomy
E. Gastro duodenal erosion D. Hyperbaric oxygen
E. Observation, antibiotics and analgesics
65. Fine Needle Aspiration Cytology (FNAC) is the d of
investigation for the diagnosis of most thyroid lesions, 69. Barrett's esophagus is metaplastic change in the
which pathological lesion is not by FNAC? living mucosa of the lower esophagus usually occurs in
A. Colloid goiter response to:
B. Follicular carcinoma A. Achalasia
C. Nodular carcinoma B. Chronic gastro-oesophageal reflux
D. Nodular goiter C. Corrosive injury
E. Papillary carcinoma D. Drug induced
E. Excessive intake of beverages
66. A. 70 years old man presents with 6 years history of
pruritus ani, the itching and discomfort has increased 70. A 55 years old female presented with painless lump
during last few months. There is neither rectal bleeding in her right breast since three years on clinical
nor pain. Physical signs suggest chronic dermatitis, and examination a lump is palpable in upper and outer
an irregular ulcer on the left anal margin. What is the quadrant of right breast about 5x5 cm with
best next step in management: peude'orange appearance and few mobile right axillary
A. Wide local excision lymph nodes were also detected FNAC is suggestive of
B. Steroid ointment carcinoma. Which of the following is the most common
C. Nitroglycerine ointment variant of carcinoma of Breast?
D. Internal sphincterotomy A. Ductal carcinoma
E. Biopsy B. Lobular carcinoma
C. Medullary carcinoma
67. A 45 years male was diagnosed with rectal cancer D. Mucinous carcinoma
after Biopsy report. This case was discussed in a
multidisciplinary tumor board and was advised
neo-adjuvant therapy. Neo-adjuvent therapy is
described as:
A. A process of periodic testing individuals for detecting
a potential malignancy in someone
B. A kind of therapy to ablate the disease completely
C. In incurable disease, measures can still be taken to
ease the symptoms of the patients
D. Radiochemotherapy before planning for surgical
resection of tumor
E. Radiochemotherapy after surgical resection of Tumor
46 Prof (SURGERY 1) 5. Extradural haematoma is a neurosurgical emergency
Typically this condition occurs due to damage to the
1. A 42 years female having a non healing ulcerated A. Posterior cerebral artery
wound over her left leg. Past history of burn. O/E there B. Middle meningeal artery
is a wide scar over the previous burns area and C. Cortical vessels
Irregular ulcer with raised and indurated base. What is D. Anterior cerebral artery
the most likely diagnosis? E. Posterior cerebellar artery
A. Basal cell carcinoma
B. Marjelin's ulcer 6. Which of the following is the most common variety of
C. Malignant melanoma thyroid cancer?
D. Varicose ulcer A. Papillary carcinoma
E Tubercular ulcer B. Follicular carcinoma
C. Medullary carcinoma
2. 50 years female having swelling in front of neck for D. Anaplastic carcinoma
five years associated with dyspnea, dysphagia for six E. Lymphoma
months, O/E swelling moving with deglutition, veins of
face, neck and anterior chest wall are engorged, 7. A 60 years male developed fever after elective
cervical lymph nodes are not palpable. What is the most laparotomy within 48 hours. What is the most common
likely diagnosis? cause of fever during his early post-operative period?
A. Simple multi nodular goiter A. Atelectasis
B. Ca thyroid B. Deep Venous Thrombosis
C. Simple diffuse goiter C. Superficial thrombophlebitis
D. Retrosternal goiter D. Urinary Tract Infection
E. Primary toxic goiter E. Wound Infection

3. A keloid scar is defined as excessive scar tissue that 8. A 45 years old lady presents to OPD with multiple
extends beyond the boundaries of the original incision painless swellings in neck. On examination multiple
or wound. Which of the following treatment options have lymph nodes are palpable in submental and
high rates of recurrence? submandibular region of the neck. While discussing final
A. Surgical excision year students during ward round about anatomical
B. Intralesional steroid injection levels of neck lymph nodes, these lymph nodes lies at
C. Laser which level?
D. Pressure elasticated garments A. Level I
E. Silicone gel sheeting B. Level II
C. Level III
4. What is the percentage of burns in a patient who D. Level IV
suffers burns over the whole left lower limb E. Level V
(circumferential) plus anterior trunk?
A. 18%. 9. A 30 year old male presented to a surgical
B. 27%. emergency with H/O fall from bike. O/E the right
C. 36%. shoulder joint is tender and the patient is unable to
D. 45%. perform movements at the shoulder joint. Which
E. 54%. following types of shoulder dislocation is most common?
A. Anterior
B. Posterior
C. Superior
D. Lateral
E. Medial
10. A young female patient presented to OPD wis 14. A 50 years old female presented to surgical OPD
diffuse thyroid swelling along with signs thyrotoxicosis with C/O painless lump in the upper outer quadrant of
Provisional diagnosis of Grave disease made. Indication right breast. On examination 5x 6 cm lump palpable that
for thyroid scanning thyroid swelling is: is hard in consistency, mobile and few lymph nodes
A. Inflammatory swellings were palpable in axilla. Next step in management of this
B. Midline neck swelling patient requires.
C. Malignant swellings A. Incisional biopsy
D. Recurrent swellings B. Mammography
E. Toxicity with nodular swelling C. MRI
D. Trucut biopsy
11. 20 years old girl came to OPD with complain of E. MAM
painless swelling at outer 1/3 of eyebrow since
childhood. On examination it is soft, non tender skin 15. Gas gangrene wound infections are associated with
overlying swelling is pinchable. Most diagnosis. severe local pain and crepitus. Which organisms is the
A. Cold abscess most appropriate for development of this
B. Dermoid cyst Conditions.
C. Hydatid cyst A. B-haemolytic streptococci
D. Lipoma B. CL Perfringens
E. Sebaceous cyst C. CL. Tetanus
D. E-coli
12. A young patient brought to the emergency E. Klebsiella
department with dehydration, altered level of
consciousness, vomiting, abdominal distension and 16. Which of the following problems can be encountered
constipation for 3 days. His pulse was 120 beats per in obese patients postoperatively, after prolonged
min, BP-90/60 mm Hg and X-ray abdomen shows abdominal surgery lasting for 4 hours?
multiple air fluid levels which are suggestive intestinal A. Deep venous thrombosis
obstruction Resuscitations started with I/V fluid. Most B. Hypothermia
important Indicator of effective resuscitation in this C. Intestinal obstruction
patient is. D. Poor wound healing
A. Blood pressure E. Risk of renal failure
B. Pulse rate
C. Regain of consciousness 17. During submandibular salivary gland surgery which
D. S. electrolytes of the following nerves may likely be at risk
E. Urine output A. Vagus nerve
B. Facial nerve
13. In dehydrated patient which of the following is C. Trigeminal nerve
the best parameter for assessing hydration after D. Glossopharyngeal nerve
fluid replacement E. Lingual nerve
A. Respiratory rate
B. Pulse 18. The compartment syndrome of lower limb occurs
C. BP after an injury or burn, resulting blood or inflammatory
D. Urine output fluid accumulated In the compartment causing pain,
E. Temperature pallor, paresthesia. What is the be?urgent treatment for
this condition?
A. Amputation
B. Elevation of leg
C. Fasciotomy
D. Hyperbaric oxygen
E. Observation, antibiotics and analgesics
19. A 45 years female presented with severe epigastric 24. Careful clinical examination will be diagnostic in the
pain, radiating to the back since 24 hours. She has past vast majority Of patients complaining of anal symptoms.
H/O dyspepsia. Her serum amylase level 2200 U/lit. Which is the most common position for examination of
She is diagnosed with acute pancreatitis Which of the anal pathology?
following is the most common of acute pancreatitis in A. Jack-knife position
Asian Countries? B. Knee-elbow position
A. Alcohol C. Lithotomy position
B. Autoimmune D. Left lateral (sims) position
C. Gallstones E. Right lateral position
D. Steroids
E. Trauma 25. In Amoebiasis the majority of abscesses occur in
which of the following organs?
20. A 20 years old man receives multiple blood A. Spleen
transfusions for abdominal gunshot wound. He B. Right lobe of liver
complains of numbness around his mouth and displays C. Lungs
carpopedal spasm and the positive Chvostek sign. The D. Sub diaphragmatic
treatment required is: E. Left lobe of liver
A. Intravenous bicarbonate
B. Intravenous potassium 26. Which of the following is most frequent site of Bed
C. Intravenous calcium sores (decubitus ulcers)
D. Intravenous digoxin A. Sacrum
E. Intravenous parathormone B. Greater trochanter
C. Ischium
21. Branchial cyst is best differentiated from cold D. Heal
abscess by: E. Occiput
A. Fluctuant.
B. Trans illumination . 27. A 35 years male is brought in emergency
C. Contains cholesterol crystal department following a car accident. He has facial
D. Contains sulphur granules laceration and displaced tibia fracture. Which of the
E. Contains sulphur granules following is the first priority in the management?
A. Insert an intravenous line
22. A 60 years old lady having multi nodular swelling in B. Insert foleys catheter
front of neck since 20 years. Her swelling remarkably C. Perform a thorough neurologic examination
increased in size during last 2 months , associated with D. Maintain the airway
weight loss and reduced appetite. What is most E. Check her blood pressure
probable diagnosis?
A. Grave's disease 28. An elderly woman presents with a unilateral red,
B. Lymphoma eczematous lesion of the nipple, associated with
C. Thyroid carcinoma underlying carcinoma of the breast. The most likely this
D. Simple multi nodular goiter condition is called as:
E. Toxic multi nodular goiter A. Actinomycosis
B. Bacterial mastitis
23. A patient underwent open reduction and internal C. Duct ectasia
fixation of fracture femur, complaining of severe limb D. Mondor’s disease
pain that is felt even worse on passive stretching of E. Paget's disease of the nipple
muscles. Limb found swollen and tense with altered
sensations distally. Most likely diagnosis is:
A. Femoral artery occlusion
B. Compartment syndrome
C. Development of localized hematoma
D. Missed muscle injuries
E. Prosthesis displacement
29. Which of the following surgical procedures is (SURGERY 2)
associated with high risk of development of DVT
postoperatively? 1. A 65 years male presented with progressive
A. Inguinal hernia surgery dysphagia and weight loss since 3 years On
B. Neurosurgery esophagoscopy and biopsy he is diagnosed as
C. Maxillofacial surgery esophageal cancer in middle 3rd of oesophagus What
D. Total knee and hip replacement is the most common histopathological variety of
E. Urological surgery oesophageal cancer?
A. Carcinoid tumour
30. 1- A 30 years male bus conductor by occupation B. Adenocarcinoma
complaining of heaviness and in leg increased on C. Mucoepidermoid carcinoma
prolonged standing and relieved by compression and D. Anaplastic carcinoma
leg elevation O/E dilated, tortuous veins on medial side E. Squamous cell carcinoma
of leg. What is your diagnosis
Ans varicose veins 2. What is the most common histopathologic type
of gastric carcinoma?
31. A patient develops tetany after thyroid surgery. A. Squamous cell carcinoma
Cause B. Carcinoid tumour
Ans parathyroid insufficiency C. Adenocarcinoma
D. Anaplastic carcinoma
32. A patient with history of betel chewing presents with E. Sarcoma
ulcer on tongue. Whats the next management
Ans diagnostic biopsy 3. A 48 years female presented with obstructive
jaundice. U/s abdomen revealed dilated CBD with
33. Obstructive shock occurs in presence of single stone in it. What is the most
Ans tension pneumothorax preferred technique for Its removal
A. ERCP & removal of stone
34. A burn patient with TBS 40%, weight 50 kg needs B. Laparoscopic CBD exploration
how much fluid in 24 hours C. Percutaneous choledochoscopy
Ans 8L D. Percutaneous transhepatic cholangiography
Formula: Fluids in 24 hours=4×body weight (kg)×total E. Surgery and choledochotomy
body surface area burned (TBSA %)
4. A 35 years old male patient farmer by occupation
35. A female patient presents with history of neck presented to surgical OPD with painless swelling right
swelling presents with palpitations and fine tremors. hypochondrium. U/S shows a 7x10cm hydatid cyst in
Which test is performed to know functional status of the right lobe of the liver. What is the preferred
patient treatment option for this patient?
Ans ye sawal sabko grebi me diya jata he, sab apna A. Immunotherapy
apna theek kre. Thank you. B. Medical treatment with albendazole
C. PAIR therapy
D. Serial ultrasound examination
E. Surgical excision
5. A 70 year old man presents with 6 years history of 10. A 22 years old lady presented in OPD with the C/O
pruritus ani, the Itching and discomfort has Increased bleeding per rectum after defecation and something
during last few months. There is neither rectal bleeding coming out of anus which reduced spontaneously.
nor pain Physical signs suggest chronic dermatitis, and Determine the degree of naemorrhelés?
an irregular ulcer on the left anal margin.what is the best A. 1 degree
next step In management: B. 2 degree
A. Wide local excision C. 3 degree
B. Steroid ointment D. 4 degree
C. Nitroglycerine ointment E. 5 degree
D. Internal sphincterotomy
E. Biopsy 11. Tumor marker in hepatocellular carcinoma is:
A. Carcinoembryonic antigen (CEA)
6. What the most common organ injured in blunt B. Alkaline phosphatase
abdominal trauma: C. Alpha fetoprotein
A. Diaphragm D. B-HCG
B. Liver E PSA
C. Mesentery
D. Spleen 12. Which of the following tumour marker is associated
E. Stomach with colonic cancer?
A. CA 15-3
7. A 30 year old woman comes with hypovolemic shock B. CEA
after blunt trauma of abdomen An emergency USG of C. AFP
abdomen shows splenic tear. Which one of the following D. CA 27-29
will be next step in management: E. CA 19-9
A CT scan of the abdomen
B. Diagnostic lavage of peritoneum before proceeding 13. In which part of the large, bowel do most colorectal
C. Monitor patient to assess for progression cancer occur?
D. Immediate surgery A. Caecum
E. Chest X ray B. Transverse colon
C. Descending colon
8. 3 weeks old child presents with non-bilious projectile D. Ascending colon
vomiting what diagnosis is suspected keeping in mind E. Sigmoid colon and rectum
the age of child:
A Congenital hypertrophic pyloric stenosis 14. A 50 year old female presented with altered bowel
B. Duodenal atresia habits since 3 years. Sigmoidoscopy and biopsy
C Acute appendicitis revealed carcinoma at the rectosigmoid junction. What
D. Intussusception is the most frequent organ presumed to be involved in
E. Volvulus distant metastasis?
A. Brain
9. Injection sclerotherapy is the non surgical B. Kidney
modality of treatment for certain haemorrhoids C. Liver
What is the best indication of this treatment? D. Lung
A. 3rd degree haemorrhoids E. Ovary
B. Intemo external haemorrhoids
C. 4th degree haemorrhoids
D. 1st and 2nd degree haemorrhoids
E. Thrombosed haemorrhoids
15. A 70 years old man presented with retention of 20. Young man about 35 years brought in emergency
urine. He has recent H/O increased urine frequency, with severe haematemesis O/E he is looking pale in a
urgency & incomplete bladder emptying. O/E bladder is state of shock. The commonest cause of the upper
palpable. Most common cause of retention in this age gastrointestinal hemorrhage is:
is? A. Avascular malformation
A. Ureteric stone B. Mallory- Weiss tear
B. Urethral stone C. Peptic ulcer
C Benign prostate enlargement D. Esophageal varices
D. Stricture of urethra E. Gastro duodenal erosion
E. Neurogenic bladder
21. Cutaneous ecchymosis is important clinical finding
16. An old age lady having prior hospital admission for In case of acute hemorrhagic pancreatitis seen in 1% of
acute pancreatitis. She has recently developed cases, Cullen's sign is discoloration of skin Involving
epigastric mass which is non-tender and her TLC count which area of body
is 5000x10' U/L. What could be the possible diagnosis? A. Around umbilicus
A. Duodenal cancer B. Flanks
B. Pancreatic abscess C. Hypogastrium
C. Pancreatic pseudocyst D. Legs
D. Pancreatic necrosis E. Upper thighs
E. Pancreatic cancer
22. In a diagnosed case of gastric outlet obstruction,
17. An elderly male diabetic patient presents with The expected metabolic abnormality can be corrected
perianal discharge on examination there were multiple by administration of?
perianal fistulas. Which of the following investigations A. Hypertonic saline or Glucose
should be performed? B. Normal saline with added potassium
A. Ultrasound C. Plasma expanders
B. CT scan D. Ringer's lactate Hartman solution
C. MRI E. Total parenteral nutrition
D. Fistulogram
E. Biopsy 23. 70 years old female with past history of open
cholecystectomy followed by postoperative wound
18. Which gas is used for the creation of infection has developed expansile swelling near
pneumoperitoneum in laparoscopic cholecystectomy? operative site. Name most probable diagnosis?
A. CO2 A. Recurrent hernia
B. O2 B. Incisional hernia
C. NO2 C. Abdominal hernia
D. Halothane D. Dorsal hernia
E. Carbon Monoxide E. Spigellion hernia

19. A 25-year-old male suffered blunt trauma to the 24. A 50 year female presented with a mass in right iliac
chest. In the ER, he presented with shortness of breath fossa, he was diagnosed as carcinoma caecum, which
and cyanosis. On examination, no breath sounds were of the following surgical procedure is appropriate for this
heard on the right side, and the percussion note was condition?
resonant. The most likely diagnosis is: A. Right hemicolectomy
A. Spontaneous pneumothorax. B. Total colectomy
B. Cardiac tamponade. C. Hartmann's procedure
C. Cardiac contusion. D. Paul mikulicz procedure
D. Tension pneumothorax. E. Extended right hemicolectomy
E. Pericardial effusion.
25. A middle aged patient c/o severe abdominal pain C. Transvesical prostatectomy
with past history of dyspepsia. His x-ray chest D. Transperitoneal Prostatectomy
demonstrates free air under right dome of diaphragm. E. Transurethral prostatectomy
Which of the following Is the most likely possibility
A. Acute cholecystitis 31. The most common site of distant metastasis by
B. Acute pancreatitis carcinoma of prostate is
C. Perforated appendicitis A. Brain
D. Perforated duodenal ulcer B. Liver
E. Ruptured liver abscess C. Pelvic bones
D. Kidneys
26. A final year MBBS student was asked to read an E. Lungs
X.ray abdomen, which revealed air under the right dome
of diaphragm. What is the likely diagnosis? 32. What is the single best test to establish the
A. Intestinal obstruction diagnosis of small bowel obstruction?
B. Intestinal perforation A. Ultrasound abdomen
C. Uterine perforation B. X-Ray abdomen Erect and supine
D. Ruptured liver abscess C. Barium meal follow through
E. Intestinal volvulus D. CT scan abdomen
E. MRI
27. The pseudomyxoma peritonei is a rare condition,
occurs more frequently in women, in which the 33. Investigation of choice for Urinary tract Stone
abdomen is filled with large quantities of yellow jelly Ans CT KUB
material. This condition is usually associated with
A. Desmoid tumour of peritoneum 34. Female with anorexia, tender hepatomegaly,
B. Mucinous adenocarcinoma of colon eosinophilia
C. Mucinous adenocarcinoma of stomach Ans Hydatid cyst.
D. Primary mucinous appendiceal tumour
E. Tuberculous peritonitis 35. left sided colon Ca often presents with
Ans Change in bowel habit.
28. A volvulus is a twisting or axial rotation of a portion
of bowel about its mesentery The most common site of
its occurrence is
A. Caecum
B. Transverse colon
C. Small intestine
D. Sigmoid colon
E. Stomach

29. A young patient admitted in ward with history of pain


in Rt iliac fossa for 4 days. On examination a tender
mass is palpable in right iliac fossa associated with
temperature 100 F. U/S suggests appendicular mass.
What is the management option?
A. Appendicectomy on next available OT list
B. Colonoscopy and biopsy from mass
C. Discharge and advise interval appendicectomy
D. Laparotomy and Rt hemicolectomy
E. Admit and start Ochsner Sherren Regimen
30. The best surgical approach to the prostate for BPH
is:
A. Perineal prostatectomy
B. Retropubic prostatectomy
45 Preprof 5. A. 45 years male was operated for colonic surgery.
He developed fever and wound infection after 5 days
1. Which of the following features is consistent with a operatively despite broad spectrum antibiotics. Most
medullary carcinoma? appropriate next step for managing such type of wound
A. Diagnosis of MEN I syndrome infection would be?
B. Localized amyloid deposits A. Frequent change of wound dressing
C. Grossly soft and tender mass B. Send blood culture
D. High levels of calcitonin C. Send CBC and start antipyretics
E. Haematologic metastatic spread D. Switch over to high dose of broad spectrum
antibiotics
2. A 50 years male patient underwent surgery for E. Wound should be opened
common bile duct stone. Choledocholithotomy was
performed and T-tube placed. On 10th postoperative 6. Tumor marker in hepatocellular carcinoma is:
day T-tube Cholangiogram performed that shows free A. Carcinoembryonic antigen (CEA)
flow of dye into the duodenum. What will be your next B. Alkaline phosphatase
step of management? C. Alpha fetoprotein
A. Block T-tubc and observe for few days D. B-HCG
B. Keep T-tube in place for 1 week more E PSA
C. Perform CT scan followed by T-tube removal
D. Remove T-tube 7. During a lecture of pathology, Teacher discusses
E. Repeat T-tube cholangiogram about tumour markers as blood chemicals produced by
the different malignant cells and every tumour has its
3. A 38 years female is diagnosed with right breast own characteristic marker associated with ilWhich of the
cancer. Clinical examination and imaging identifies a following tumour marker is associated with carcinoma
lump of 3x3 cm in upper outer quadrant with benign prostate?
axillary nodes. Patient is interested in conserving breast A. AFP
and has been suggested for neo adjuvant B. Beta-hCG
chemotherapy.Regarding management of malignant C. BTA
tumor, neo adjuvant therapy is described as: D. LDH
A. The process of periodic testing Individuals for E. PSA
detecting a potential malignancy in someone.
B. A kind of therapy to ablate the disease completely. 8. A 35 years old male patient presented with painful,
C. In Incurable disease, measures can still be taken to irreducible swelling in right inguinal region for 2 days
ease the symptoms of the patient. associated with acute intestinal obstruction. Surgery
D. Chemotherapy before planning for surgical resection performed in emergency and after exploration of the
of tumour swelling, gut was found bluish-black in color. Most likely
E. Radiochemotherapy after surgical resection of diagnosis is:
tumour A. Incarcerated hernia
B. Ischemic bowel disease
4. During a lecture of surgery , different postoperative C. Obstructed inguinal hernia
complications after splenectomy are being discussed. D. Obstructed femoral hernia
Splenectomy predisposes the patient, especially a child E. Strangulated inguinal hernia
to infection Which of the following organisms are
Involved in such infectious complications?
A. E.coli
B. Klebsiella
C. Pseudomonas
D. Pneumococci
E. Staphylococci
9. A 45 years old lady presents to OPD with multiple 13. A 55 years old lady with long standing history of
painless swellings in neck. On examination multiple thyroid swelling presents to OPD with recent history
lymph nodes are palpable in submental and discomfort, weight loss and hoarseness of voice.
submandibular region of the neck. While discussing final Clinical examination reveals hard fixed thyroid swelling
year students during ward round about anatomical .FNAC shows malignant cells. Which of the following is
levels of neck lymph nodes, these lymph nodes lies at the most common variety of thyroid cancer?
which level? A. Anaplastic carcinoma
A. Level I B. Follicular carcinoma
B. Level II C. Lymphoma
C. Level III D. Medullary carcinoma
D. Level IV E. Papillary carcinoma
E. Level V
14. A 60 years lady is known case of Gall bladder
10. A 55 years female has been diagnosed with Right disease. Now she presents with symptoms of small
Breast carcinoma (Invasive Ductal Carcinoma. GradeII), bowel obstruction for last one day. Her plain X Ray
having clinical stage (cT2 N1).Metastatic workup is abdomen reveals multiple air-fluid levels and pr air in
negative for any distant mets.You are planning right the biliary tree (pneumobilia). What is the most likely
sided modified radical mastectomy (MRM) in this diagnosis?
patient. Which of the following structures are excised in A. Acute cholecystitis infected with gas forming
MRM? organisms.
A. Breast tissue along with nipple and areola B. Cholecysto-enteric fistula leading to gallstone ileus C.
B. Breast tissue along with Nipple and areola + axillary Duodenal perforation
clearance D. Gangrene of gallbladder
C. Breast tissue along with nipple and areola + axillary E. Rupture of gallbladder
clearance+pectoralis muscles
D. Breast tissue along with Nipple and areola + axillary 15. Colorectal cancer is the second most common
sampling cause of cancer death in the west. Regarding
E. Breast tissue along with Nipple and areola + distribution of colorectal cancer, which is the most
pectoralis muscles frequent site of involvement
A. Ascending colon
11. A 50 years obese female c/o severe epigastric pain B. Caecum
radiating to the back diagnosed as a case of acute C. Descending colon
pancreatitis. What could be the most common cause of D. Rectosigmoid
acute pancreatitis? E. Transverse colon
A. Drugs
B. CBD stone 16. A 50 year old female presented with altered bowel
C. Idiopathic habits since 3 years. Sigmoidoscopy and biopsy
D. Hyperlipidemia revealed carcinoma at the rectosigmoid junction. What
E. Alcohol is the most frequent organ presumed to be involved in
distant metastasis?
12. Ascending cholangitis is an infection of the biliary A. Brain
tract usually associated with obstruction, and patient B. Kidney
presents with pain, fever and jaundice. The most C. Liver
probable cause of this condition is: D. Lung
A. As a complication of ERCP E. Ovary
B. CBD stone
C. Cholangiocarcinoma
D. Primary biliary cirrhosis
E. Primary sclerosing cholangitis
17. Which Of the following is the best surgical 22. The compartment syndrome of lower limb occurs
approach for performing prostatic surgery: after an injury or burn, resulting blood or inflammatory
A. Retropubic fluid accumulated In the compartment causing pain,
B. Transperineal pallor, paresthesia. What is the be?urgent treatment for
C. Transperitoneal this condition?
D. Transvesical A. Amputation
E. Transurethral B. Elevation of leg
C. Fasciotomy
18. Over the last 30 years . helicobacter pylori has D. Hyperbaric oxygen
proved to be an important etiological factor for common E. Observation, antibiotics and analgesics
gastrointestinal disease The sufficient epidemiological
data support the association of H pylori with 23. A 45 years male, after 8 hours of undergoing
A. Type A gastritis surgical management for open femoral fracture develop
B. Type B gastritis severe pain In the operated limb.On examination patient
C. Erosive gastritis is anxious and even the passive movements provoke
D. Reflux gastritis pain.Limb appears swollen and tense. Sensations are
E, Stress gastritis reduced distally. The most likely clinical diagnosis is:
A. Compartment syndrome
19. An elderly woman presents with a unilateral red, B. Compression of common peroneal nerve
eczematous lesion of the nipple, associated with C. Femoral artery occlusion
underlying carcinoma of the breast. The most likely this D. Localized hematoma
condition is called as: E. Muscle injury
A. Actinomycosis
B. Bacterial mastitis 24. A 60 years male smoker has history of progressive
C. Duct ectasia dysphagia and weight loss. Oesophagosycopy and
D. Mondor’s disease biopsy has been performed to evaluate the cause which
E. Paget's disease of the nipple shows oesophageal carcinoma. The most frequent site
of involvement of oesophageal cancer is:
20. Lung cancer is one of the most common cancers A. Lower third of oesophagus
throughout the world and is the most common causes B. Middle third of oesophagus
cancer death. The occurrence of most common type of C. Oesophagogastric junction
lung cancer is: D. Pharyngo oesophageal junction
A. Adenocarcinoma E. Upper third of oesophagus
B. Bronchoalveolar carcinoma
C. Large cell undifferentiated carcinoma 25. The incidence of congenital umbilical hernia occurs
D. Small cell lung cancer upto 10% of infants. Which of the following treatment
E. Squamous cell carcinoma options is indicated in asymptomatic hernia under the
age of two years?
21. A glandular metaplasia of the lower esophageal A. Conservative treatment and reassurance to parents
mucosa resulting from chronic gastroesophageal reflux B. Laparoscopic hernia repair
is known as: C. Open surgery through sub umbilical incision and
A. Tylosis defect is closed with Interrupted suture.
B. Leukoplakia D. Open surgery with umbilical excision and defect is
C. Achalasia closed with interrupted suture
D. Barrett's esophagus E. Open surgery with mesh repair
E. Plummer-vinson syndrome
26. The mesenteric cysts occurs in the mesentery of the 31. 70 year old male who is known case of peptic ulcer
small intestine or the colon are of various types Which disease for past 30 years, presented with H/O anorexia,
of the following is the most common variety, frequently progressive vomiting, with significant weight loss. He
occurring in the mesentery of looks grossly anaemic. Abdominal examination reveals
A. Chylolymphatic cyst a mass in epigastric region. Best investigation to
B. Dermoid cyst confirm the diagnosis will be:
C. Hydatid cyst A. Barium meal
D. Enterogenous cyst B. CT scan abdomen
E. Urogenital cyst C. Endoluminal ultrasound
D. Endoscopy and biopsy
27. Oesophageal varices usually present with the acute E. PET scan
onset of a large volume haematemesis, caused by
portal hypertension In liver cirrhosis. After resuscitation 32. Epigastric hernia arises through a defect in the linea
the best method of arresting bleeding is: alba anywhere from xiphoid process to the umbilicus. It
A. Blood transfusion commonly contains:
B. Endoscopic band ligation combined with A. Extraperitoneal fat
vasoconstrictors. B. Falciform ligament
C. Splanchnic vasoconstrictors by vasopressin C. Intestinal loop
D. Transjugular intra hepatic portosystemic stent shunts D. Omentum
(TIPSS) E. Part of greater curvature of the stomach
E. Vitamin-K lO mg intravenously
33. The pseudomyxoma peritonei is a rare condition,
28. A 65 years male C/0 painless swelling below right occurs more frequently in women, in which the
eye for two years, O/E rounded, non tender, ulcerated abdomen is filled with large quantities of yellow jelly
nodule with rolled edges and indurated base. What is material. This condition is usually associated with
the probable diagnosis? A. Desmoid tumour of peritoneum
A. Basal cell carcinoma B. Mucinous adenocarcinoma of colon
B. Cutaneous horn C. Mucinous adenocarcinoma of stomach
C. Malignant Melanoma D. Primary mucinous appendiceal tumour
D. Squamous cell carcinoma E. Tuberculous peritonitis
E. Solar keratosis
34. Typhoid fever is caused by salmonella typhi and
29. The best investigation of choice for assessment of patient usually presents with fever and abdo The most
blunt abdominal trauma in unstable patient is: common and frequent surgical complication is:
A. CT scan abdomen A. Cholecystitis
B. Diagnostic peritoneal lavage (DPL) B. Intestinal haemorrhage
C. MRI C. Ileal perforation
D. Plain X-Ray abdomen D. Intestinal obstruction
E. U/S abdomen (focused assessment with sonography E. Paralytic ileus
in trauma FAST)
35. A 25 years male presented with attacks of
30. A 50 year female presented with a mass in right iliac abdominal pain and intermittent diarrhea associated
fossa, he was diagnosed as carcinoma caecum, which with weight loss and evening pyrexia since 2-3 years.
of the following surgical procedure is appropriate for this Clinical examination reveals mild anaemia and a
condition? nontender mass palpable in RIF. DRE unremarkable.
A. Right hemicolectomy What Is the most likely diagnosis?
B. Total colectomy A. Appendicular mass
C. Hartmann's procedure B. Actinomycosis
D. Paul mikulicz procedure C. Carcinoma of the caecum
E. Extended right hemicolectomy D. Crohn's disease
E. Ileo-caecal tuberculosis
36. A 35 years male is brought in emergency 41. A 50 years male became dehydrated after episodes
department following a car accident. He has facial of persistent vomiting, later on as confirmed by I;tests,
laceration and displaced tibia fracture. Which of the he developed hypokalemic metabolic alkalosis. Which
following is the first priority in the management? of the following l/V fluid replacement is most appropriate
A. Insert an intravenous line for this condition?
B. Insert foleys catheter A. Dextrose water 5%
C. Perform a thorough neurologic examination B. Dextrose water 10%
D. Maintain the airway C. 0.9% Normal saline with KCI
E. Check her blood pressure D. Plasma expander
E. Ringer's lactate
37. In Barium swallow, which radiologic finding is
diagnostic in case of Achalasia? 42. 30 years old lady complains about painless bleeding
A. Coffee Bean sign per rectum for past one month. On examination she is
B. Bird Beak appearance severely anaemic with third degree haemorrhoids.
C. Claw sign What will be the first step in management in this lady:
D. Double Bubble A. Band ligation
E. Air-fluid level B. Correction of anaemia
C. Discharge and follow up
38. A 42 years female having non healing ulcerated D. Emergency surgery
wound over her left leg O/E there is a vertical scar of E. Tbandageand bedrest
previous surgery and irregular ulcer with raised and
indurated base. What is the most likely diagnosis? 43. In a round, the resident presents a case of 55 years
A. Basal cell carcinoma old male with obstructive jaundice. When you talk about
B. Marjolin's ulcer the possible causes, his knee jerk response is
C. Malignant Melanoma choledocholithiasis. On examination, Glad palpable.
D. Varicose ulcer You ask resident about Courvoisier's law. Regarding
E. Tubercular ulcer Courvoisier's law.
A. If infection of the common bile duct supervenes , the
39. While discussing various investigation tools to jaundice and pain are complicated by hi grade fever
evaluate thyroid conditions you explain the role of FNAC B. If in the presence of jaundice gallbladder is palpable
Most of the thyroid conditions may be diagnosed by then the jaundice Is unlikely to be due CBD stone
FNAC Which of the following conditions that cannot be C. If the non tender gallbladder Is palpable without the
diagnosed by FNAC: presence of jaundice
A. Anaplastic carcinoma D. If the tender gallbladder is palpable associated with
B. Follicular carcinoma high grade fever and severe toxaemia, but no jaundice
C. Medullary carcinoma E. Stone is impacted in Hartmann’s pouch causing
D. Papillary carcinoma jaundice due to compression of the common bile duct
E. Thyroiditis
44. A young patient admitted in ward with history of pain
40. A 60 years male developed fever after elective in Rt iliac fossa for 4 days. On examination a tender
laparotomy within 48 hours. What is the most common mass is palpable in right iliac fossa associated with
cause of fever during his early post-operative period? temperature 100 F. U/S suggests appendicular mass.
A. Atelectasis What is the management option?
B. Deep Venous Thrombosis A. Appendicectomy on next available OT list
C. Superficial thrombophlebitis B. Colonoscopy and biopsy from mass
D. Urinary Tract Infection C. Discharge and advise interval appendicectomy
E. Wound Infection D. Laparotomy and Rt hemicolectomy
E. Admit and start Ochsner Sherren Regimen
45. A middle aged female presents with history of
weight loss, abdominal mass and enlargement of left
supraclavicular node. You teach accompanying interns
and residents about Virchow's node that it is a
prominent lymph node in the left supraclavicular fossa in
response to malignant disease spread from some
primary lesions. Which of the following primary lesion
have association with Virchow’s node?
A. Adrenal tumour
B. Colo-rectal carcinoma
C. Gastric carcinoma
D. Renal cell carcinoma
E. Uterine carcinoma

46. In a final year posting class, surgeon is discussing


about importance of surgical drains which allow
drainage of fluid or air that may otherwise collect at an
operation site. Under water seal drain is special type of
drain used after which of the following surgical
procedure?
A. Laparotomy
B. Mastectomy
C. Parotidectomy
D. Thoracotomy
E. Thyroidectomy

47. While discussing with your trainees about


management of patients presenting with breast lumps,
you attempt to test the baseline knowledge of trainees
in this context, by asking about term "Triple
Assessment". What is the most accurate explanation of
triple assessment?
A. Clinical assessment, U/S breast & mammography
B. Clinical assessment, U/S breast & CT scan chest
C Clinical assessment, U/S breast & tissue sampling
taken for either cytological or histological analysis
D. U/S breast, CT scan chest & FNAC
E. U/S breast, MRI & Trucut needle biopsy

48. Renal calculi are the most frequent cause of renal


pain in our country. A staghorn calculus is smooth
walled and present in renal pelvis. It is usually
composed of:
A. Blood pigment
B, calcium oxalate monohydrate
C. Calcium, ammonium and magnesium phosphate
(Trippie phosphate)
D. Cystine
E. Uric acid
45 Prof (SURGERY 1) 5. What is the most common cause of lymphoedema
(non-pitting oedema) worldwide?
1. A middle aged male having bilateral lymphatic A. Iatrogenic -radiotherapy
oedema, hydrocele and inguinal lymphadenitis. Blood B. Filariasis
smear shows micro filarae and eosinophilia: What is C. Iatrogenic - lymph node resection surgery
diagnosis: D. Leptospirosis
A. Filariasis E. Chagas disease
B. Lymphatic malignancy
C. Deep vein thrombosis 6. A 30-year-old man presents to the Emergency
D. Superficial thrombo phlebitis Department following a high speed motor vehicle
E. Fungal infection accident. He has marked abdominal distension, a pulse
rate of 130 and a blood pressure of 80/50 mmHg. The
2. A 50 years female having painless lump in Rt breast most appropriate initial investigation would be
with blood stained nipple discharge. O/E a single 3 x 3 A. Abdominal angiogram
cm non tender, irregular hard, lump is palpable in B. Abdominal paracentesis
central part of Rt breast, ipsilateral axillary lymph nodes C. CT scan of the abdomen
palpable and mobile. What investigations will be helpful D. plain X-ray of the abdomen
to reach final diagnosis: E. FAST (focused abdominal sonography for trauma)
A. Trucut needle biopsy scan
B. Doppler ultrasound
C. Ultrasound breast 7. You were called to the emergency department to
D. FNAC review a 30 year old male patient who was in a major
E.Mammography car accident. His vital signs revealed heart rate of 140
beats/min and blood pressure of 60/40. Physical
3. A 35 female complaining of painless swelling in front examination and radiograph revealed open-book pelvic
of neck for a year, no pressure symptoms. O/E discrete fracture. There was no other major injury identified. The
swelling in rt lobe of thyroid. Thyroid scan shows cold diagnosis of this patient is:
nodule in rt lobe of thyroid. FNAC revealed nodular A. Hypovolemic shock.
goiter. How will you treat this condition? B. Septic shock.
A. Wait and watch C. Neurogenic shock.
B. Rt lobectomy and isthmusectomy D. Cardiogenic shock.
C. Radioiodine therapy E. Spinal shock.
D. Sub total thyroidectomy
E. Total thyroidectomy 8. A 20 years old man receives multiple blood
transfusions for abdominal gunshot wound. He
5. In tongue cancer, the site least affected is: complains of numbness around his mouth and displays
A. Posterior portion carpopedal spasm and the positive Chvostek sign. The
B. Lateral margin treatment required is:
C. Dorsal surface A. Intravenous bicarbonate
D. Tip B. Intravenous potassium
E. Ventral surface C. Intravenous calcium
D. Intravenous digoxin
4. The most common fracture of face is that of: E. Intravenous parathormone
A. Zygoma
B. Nasal bone
C. Orbital bone
D. Mandible
E. Maxilla
9. A 35 years male is brought in emergency department 15. During submandibular salivary gland surgery which
following a car accident. He has facial laceration and of the following nerves may likely be at risk
displaced tibia fracture. Which of the following is the first A. Vagus nerve
priority in the management? B. Facial nerve
A. Insert an intravenous line C. Trigeminal nerve
B. Insert foleys catheter D. Glossopharyngeal nerve
C. Perform a thorough neurologic examination E. Lingual nerve
D. Maintain the airway
E. Check her blood pressure 16. Intermittent claudication is a cramp-like pain felt in
the limb muscles, which of the following statements is
10. The use of prophylactic antibiotics are usually appropriate?
recommended A. The pain felt in the lower limb at rest
A. Two hours after surgery B. The pain felt on taking the first step
B. Two hours before surgery C. The pain brought on by walking and relieved by
C. One hour before surgery standing still
D. At the time of induction of anesthesia D. The foot is often cold and paralyzed
E. One hour after surgery E. Ulceration and gangrene of toes are usually
associated with even mild cases
11. What is the percentage of burns in a patient who
suffers burns over the whole left lower limb 17. A 65 years male C/0 painless swelling below right
(circumferential) plus anterior trunk? eye for two years, O/E rounded, non tender, ulcerated
A. 18%. nodule with rolled edges and indurated base. What is
B. 27%. the probable diagnosis?
C. 36%. A. Basal cell carcinoma
D. 45%. B. Cutaneous horn
E. 54%. C. Malignant Melanoma
D. Squamous cell carcinoma
12. Which of the follow ing maybe clinical features of E. Solar keratosis
severe shock
A. Normal conscious level 18. Fluid containing highest amount of K(Potassium) is:
B. Decreased Respiratory rate A. Ringer lactate
C. Decreased Pulse rate B. Plabolyte M
D. Increased urine output C. 10% D/water
E. Decreased urine output D. N/Saline
E. 5% D/saline
13. Which of the following factors decrease the risk of
wound infection 19. A 60 years old lady having multi nodular swelling
A. Chemotherapy infront of neck since 20 years. Her swelling remarkably
B. Good surgical technique increased in size during last 2 months , associated with
C. Obesity weight loss and reduced appetite. What is most
D. Radiotherapy probable diagnosis?
E. Steroids A. Grave's disease
B. Lymphoma
14. In Amoebiasis the majority of abscesses occur in C. Thyroid carcinoma
which of the following organs? D. Simple multi nodular goiter
A. Spleen E. Toxic multi nodular goiter
B. Right lobe of liver
C. Lungs
D. Sub diaphragmatic
E. Left lobe of liver
20. During a lecture of surgery different post-operative 23. A 45 years male, after 8 hours of undergoing
complications after splenectomy are being discussed. surgical management for open femoral fracture develop
Splenectomy predisposes the patient, especially a child severe pain In the operated limb.On examination patient
to infection Which of the following organisms are is anxious and even the passive movements provoke
Involved in such infectious complications? pain.Limb appears swollen and tense. Sensations are
A. E.coli reduced distally. The most likely clinical diagnosis is:
B. Klebsiella A. Compartment syndrome
C. Pseudomonas B. Compression of common peroneal nerve
D. Pneumococci C. Femoral artery occlusion
E. Staphylococci D. Localized hematoma
E. Muscle injury
21. A 5O years female presents to outpatient
department with result of core biopsy from her right 24. A 60 years male developed fever after elective
breast lump done a week ago. Report confirms Invasive laparotomy within 48 hours What is the most common
ductal carcinoma Grade II, ER/PR (positive), (negative). cause of fever during his early post operative period?
Clinically tumor is 5.5*3 cm located in upper outer A. Wound infection
quadrant. Axilla is unremarkable metastatic workup is B. UH
negative (cT3N0M0).You refer the patient to Medical C. Superficial thrombophlebitis
oncologist for Neo Chemotherapy (NAC). The main D. Atelectasis
purpose for NAC in this patient will be: E. DVT
A. For palliative treatment
B. To avoid future radiotherapy 25. While discussing various investigation tools to
C. To downstage disease evaluate thyroid conditions you explain the role of FNAC
D. To prevent local spread Most of the thyroid conditions may be diagnosed by
E. To reduce post-surgical complications FNAC Which of the following conditions that cannot be
diagnosed by FNAC:
22. A 55 years female has been diagnosed with Right A. Anaplastic carcinoma
Breast carcinoma (Invasive Ductal Carcinoma. GradeII), B. Follicular carcinoma
having clinical stage (cT2 N1).Metastatic workup is C. Medullary carcinoma
negative for any distant mets.You are planning right D. Papillary carcinoma
sided modified radical mastectomy (MRM) in this E. Thyroiditis
patient. Which of the following structures are excised in
MRM?
A. Breast tissue along with nipple and areola 26. A 45 years old lady presents to OPD with multiple
B. Breast tissue along with Nipple and areola + axillary painless swellings in neck. On examination multiple
clearance lymph nodes are palpable in submental and
C. Breast tissue along with nipple and areola + axillary submandibular region of the neck. While discussing final
clearance+pectoralis muscles year students during ward round about anatomical
D. Breast tissue along with Nipple and areola + axillary levels of neck lymph nodes, these lymph nodes lies at
sampling which level?
E. Breast tissue along with Nipple and areola + A. Level I
pectoralis muscles B. Level II
C. Level III
D. Level IV
E. Level V
27. A 30 years male bus conductor by occupation (SURGERY 2)
complaining of heaviness and in leg increased on
prolonged standing and relieved by compression and 1. Tumor marker of Yolk Sac Tumor
leg elevation O/E dilated, tortuous veins on medial side Ans AFP
of leg. What is your diagnosis
Ans Varicose veins 2- Regarding Wilms tumor which one is correct?
Ans. On examination stromal and epithelial appearance
28. Which of the following tumors give osteosclerotic
lesion in comparison to osteolytic lesion by rest of Remaining Question from Preproff & Past Papers
others
Ans Prostate

29. Regarding development of postoperative DVT,


which of the following surgical procedures is associated
with high risk of DVT
Ans Pelvic bone

30.

31. Functional state of thyroid gland is done by


Ans Thyroid scan / Thyroid profile

32. Empty sella turcica Scenario


Ans Pituitary Tumor

33. RTA with severe bleeding Scenario


Ans Arrest hemorrhage

34. Tetany postoperatively scenario


Ans Parathyroid Insufficiency

35. Pan Chewing + ulcer on right margin of tongue with


Everted margin and obdurated base
Ans Biopsy Of Tongue to confirm diagnosis or Right
hemiglossectomy??
44 Preprof (SURGERY 1) 7. A gross lymphoedema of the lower limbs is often
called elephantiasis is consequent lymphatic obstruction
1. Lymphoedema are classified into primary and caused by parasitic infection of the lymphatics Which of
secondary. Among the secondary lymphoedema . what the following parasites are responsible for this
is the most common cause of world wide, particularly condition?
prevalent in Africa and india? A. Filaria
A. Filariasis B. Trichomonas
B. Lymphoma C. Trypnosomas
C. Radiotherapy to lymph node D. Giardia
D. Surgical excision of lymph nodes E. Leishmania
E. Lymphnode
8 ??
2. Enter Roll No. of Final Year MBBS Pre-Prof.
Online Test Session 2015-16 (MEDICINE - II): 9. In Amoebiasis the majority of abscesses occur in
which of the following organs?
3. The most common direction of dislocation of shoulder A. Spleen
joint is: B. Right lobe of liver
A. Anterosuperior C. Lungs
B. Superior D. Sub diaphragmatic
C. Posterior E. Left lobe of liver
D. Antero inferior
E. Inferoposterior 10. During the catabolic phase of the stress response
muscle wasting occurs as a result of an increase in
4. What is the minimal adequate postoperative urine muscle protein degradation The major site of protein
output for excretion of waste products? loss occurs in
A. 1ml/kg/hr A. Respiratory muscles
B. 0.5 ml/kg/hr B. Cardiac muscles
C. 3ml/kg/hr C. Smooth muscles of gut
D. 15ml/kg/hr D. Central truncal skeletal muscles
E. 2ml/kg/hr E. Peripheral skeletal muscles

5. During parotid gland excision which of the following 11. Compartment syndromes typically occur in closed
nerves may likely be at risk? lower limb injuries it is best managed by
A. Trigeminal nerve A. Late fasciotomy
B. Accessory nerve B. Early fasciotomy
C. Lingual nerve C. Elevation of the affected limb close observation
D. Hypoglossal nerve D. Intravenous antibiotics
E. Facial nerve E. Intravenous diuretics

6. Which of the following thyroid operations is also 12. Which of the following surgical procedures is
called Dunhill procedure? associated with high risk of development of DVT
A. Lobectomy ♦ isthmusectomy postoperatively?
B. Near total thyroidectomy A. Inguinal hernia surgery
C. Total thyroidectomy B. Neurosurgery
D. Isthmusectomy C. Maxillofacial surgery
E. Subtotal thyroidectomy D. Total knee and hip replacement
E. Urological surgery
13. Which of the following features is consistent with a 19. A patient developed acute blood transfusion
medullary carcinoma? reaction The immediate step in management is
A. Diagnosis of MEN I syndrome A. Give oxygen by mask
B. Localized amyloid deposits B. Dehydrate the patient
C. Grossly soft and tender mass C. Stop blood transfusion
D. High levels of calcitonin D. Catheterize the patient
E. Haematologic metastatic spread E. Uecheck the cross match report

14. Which or the following Kcal/g will be released by 20. A 45 years male was operated for colonic surgery
providing 1 gram of fat? developed fever and wound infection after 5 days post
A. 10 Kcal/g operatively How is his wound infection treated?
B. 6 Kcal/g A. Wound should be opened and completely drained
C. 8 Kcal/g B. Advise frequent change of wound dressing
D. 9 Kcal/g C. Send Wood culture
E. 4 Kcal/g D. Send CBC and start antipyretics
E. Switch over to high dose ol broad spectrum
15. In dehydrated patient which of the following is antibiotics
the best parameter for assessing hydration after
fluid replacement 21. Which of the following is the mainstay of
A. Respiratory rate investigation for the management of varicose veins?
B. Pulse A. Magnetic resonance venography
C. BP B. Duplex ultrasound imaging
D. Urine output C. Tourniquet test
E. Temperature D. Vericography
E. Descending intravenous venography
16. The most common cause of increased serum
calcium level in hospitalized patient is 22. Which of the following is the most common variety
A. Primary hyperparathyroidism of thyroid cancer?
B. Vitamin D excess A. Papillary carcinoma
C. Malignancy B. Follicular carcinoma
D. Myeloma C. Medullary carcinoma
E. Immobilization D. Anaplastic carcinoma
E. Lymphoma
17. The commonest complication of chronic
osteomyelitis is 23. In shock as tissue perfusion is reduced cells are
A. Amyloidosis deprived of oxygen and must switch from aerobic to
B. Pathological fracture anaerobic metabolism The product of anaerobic
C. Acute exacerbation respiration is
D. Sinuses discharging pus or piece of bone A. Carbon dioxide (co2)
E. Deformity B. Amonia
C. Uric acid
18. 25 year old male suffered blunt trauma to the chest, D. Urea
in the ER he presented with shortness of breath and E. Lactic acid
cyanosis On Examination no breath sounds in the right
side and percussion note resonant The most likely 24. Following massive blood transfusion which of the
diagnosis is following complications may occur
A. Cardiac contusion A. Hypoglycaemia
B. Spontaneous pneumothorax B. Hypercalcaemia
C. Cardiac tamponade C. Hypoglycaemia
D. Pericardial effusion D. Hyperlipidaemia
E. Tension pneumothorax E. Hypocalcaemia
25. A 60 years male developed fever after elective 31. The compartment syndrome of lower limb occurs
laparotomy within 48 hours What is the most common after an injury or bum resulting blood or inflammatory
cause of fever during his early post operative period? fluid accumulated in the compartment causing pain
A. Wound infection pallor paresthesia What is the best urgent treatment for
B. UH this condition?
C. Superficial thrombophlebitis A. Elevation of leg
D. Atelectasis B. Observation antibiotics and analgesics
E. DVT C. Amputation
D. Hyperbaric oxygen
26. Although any portion of the breast may be involved E. Fasciotomy
with breast tumour which of the following sites is the
most frequent for development of breast cancer? 32. Name of Institution:
A. Lower inner quadrant A. Chandka Medical College Larkana
B. Lower outer quadrant B. Ghulam Muhammad Mahar Medical College Sukkur
C. Upper inner quadrant
D. Nipple and areola 33. Tumour markers are blood chemicals produced by
E. Upper outer quadrant the indignant cells Some tumours have characteristic
marker associated with them Which of the following is
27. Which of the following investigation does not has the tumour marker associated with hepatocellular
radiation hazard? carcinoma?
A. IVP A. AFP
B. Thyroid scan B. Beta-HCG
C. X-ray C. CA 19-9
D. CT scan D. CA 125
E. MRI E. CEA

28. During submandibular salivary gland surgery which 34. Which of the following is most frequent site of Bed
of the following nerves may likely be at risk sores (decubitus ulcers)
A. Vagus nerve A. Sacrum
B. Facial nerve B. Greater trochanter
C. Trigeminal nerve C. Ischium
D. Glossopharyngeal nerve D. Heal
E. Lingual nerve E. Occiput

29. The most important factor in the development of 35. Which of the following tumour marker is associated
endemic goiter is: with colonic cancer?
A. Dyshormonogenesis A. CA 15-3
B. Iodine deficiency B. CEA
C. Iatrogenic C. AFP
D. Goitrogens D. CA 27-29
E. Secondary to pituitary or hypothalamic disease E. CA 19-9

30. Which of the following parasite is responsible for 36. Chronic arterial stenosis or occlusion is commonly
filariasis and spread by the mosquito? caused by atheroma Which of the following is the early
A. Dracunculus medinensis feature of chronic arterial occlusion in the leg?
B. Schistosoma mansoni A. Gangrene of toes
C. Toxoplasma gondii B. Ulceration
D. Trypanosoma brucei C. Absent arterial pulsations
E. Wuchereria bancrofti D. Breast pain
E. Intermittent claudication
37. Regarding Carpal Tunnel syndrome 43 ??
A. Patient usually complaining of tingling and numbness 44 ??
in the middle one and half fingers
B. Occurs due to compression of median nerve 45. In which of the following conditions Total
C. Wasting of hypothenar eminence is visible Parenteral Nutrition (TPN) is indicated?
D. Occurs due to compression of radial nerve A. Intestinal fistula
E. Occurs due to compression of ulnar nerve B. Recto vaginal fistula
C. Fistula in ano
38. A 25 year old man presents to surgical OPD with a 4 D. Vesico vaginal fistula
cm mobile anterior neck mass RSAC reveals cells with E. Thyroglossal fistula
orphan Annie eye nuclei and psammoma bodies What
is the most likely diagnosis? 46. The breast cancer is more commonly found in
A. Medullary carcinoma the region of
B. Thyroglossal cyst A. Lower inner quadrant
C. Follicular carcinoma B. Upper inner quadrant
D. Benign thyroid nodule C. Axillary tail
E. Papillary carcinoma D. Upper outer quadrant
E. Lower outer quadrant
39. The treatment options for toxic goiter are Antithyroid
drugs radioiodine and surgery What is the absolute 47. The use of prophylactic antibiotics are usually
contraindication of radioiodine therapy? recommended
A. Renal failure A. Two hours after surgery
B. Pregnancy B. Two hours before surgery
C. Morbid obesity C. One hour before surgery
D. Bronchial asthma D. At the time of induction of anesthesia
E. Cardiac failure E. One hour after surgery

40. Primary hyperparathyroidism is commonly a 48. Which of the following is the causative factor of
sporadic disorder associated with hypercalcaemia and septic shock
inappropriately raised serum PTH Which of the following A. Acute pancreatitis
is the most probable cause of this condition? B. Severe haemorrhage
A. Vitamin D deficient rickets C. Severe diarrhea
B. Chronic renal failure D. Peritonitis following gut perforation
C. Pseudohypoparathyroidism E. Excessive vomiting
D. Parathyroid adenoma
E. Malabsorption 49. For burn patients which of the following fluids is
appropriate for resuscitation?
41. Acute arterial occlusion is usually caused by A. 5% Dextrose water
A. Fat embolism B. Hypertonic saline
B. Atherosclerosis C. Fresh frozen plasma
C. Thromboembolism D. Ringers lactate
D. Air embolism E. 10% Dextrose water
E. Embolism
50. Deep vein thrombosis is commonly diagnosed by
42. Among all the ulcers in lower leg which of the A. Duplex ultrasound image
following is the most common etiologic al type of this B. D Dimer test
non healing ulcer? C. MUI
A. Neoplastic ulcer D. Ascending venography
B. Arterial ischaemic ulcer E. Trendelenburg test
C. Traumatic ulcer
D. Venous disease ulcer 51. ?
E. Neuropathic ulcer
52. Extradural haematoma is a neurosurgical 5. What is best treatment for option for U bladder stone
emergency Typically this condition occurs due to size, upto 2cm?
damage to the A. Percutaneous suprapubic litholapaxy
A. Posterior cerebral artery B. Periurethral litholapaxy
B. Middle meningeal artery C. Open cystolithotomy
C. Cortical vessels D. Extra corporeal shock wave lithotripsy (ESW|)
D. Anterior cerebral artery E. Intra corporeal shockwave lithotripsy
E. Posterior cerebellar artery
6. In which part of the large, bowel do most colorectal
(SURGERY 2) cancer occur?
A. Caecum
1. During decent of testis it passes through deep ring B. Transverse colon
travel into inguinal canal emerges through superficial C. Descending colon
inguinal ring descends into the scrotum The superficial D. Ascending colon
inguinal ring is a v-shaped defect in the? E. Sigmoid colon and rectum
A. Fascia transversalis
B. Rectus abdominis fascia 7. The most common site of distant metastasis by
C. Transversus abdominis carcinoma of prostate is
D. External oblique aponeurosis A. Brain
E. Internal oblique aponeurosis B. Liver
C. Pelvic bones
2. The term odynophagia refers to D. Kidneys
A. Regurgitation E. Lungs
B. Dysphagia for liquids
C. Gastro esophageal reflux 8. After haemorrhoidectomy which of the following may
D. Dysphagia for solids be the early complication noticed
E. Pain on swallowing A. Secondary haemorrhage
B. Faecal Incontinence
3. The pilonidal sinus is a condition lined by granulating C. Anal stricture
tissue containing hair lying loosely within the lumen The D. Anal fissure
common site of its occurrence is E. Reactionary haemorrhage
A. Axilla
B. Inguinal region 9. Hydatid cyst in the liver is a common condition can
C. Interdigital cleft occur after handling with dogs The causative agent is
D. Umbilicus A. Entamoeba histolytica
E. Natal cleft overlying the coccyx B. Ascaris lumbricoides
C. Enterobius vermicularis
4. The most common mode of spread or carcinoma of D. Liver fluke
the stomach is through transperitoneal route when the E. Echinococcus granulosus
tumour reached the serosa The term krukenberg's
tumour is described as when the gastric carcinoma 10. The most common type of liver tumour is
spread through peritoneum and involve the A. Liposarcoma
A. Transverse colon B. Intrahepatic cholangiocarcinoma
B. Ovaries C. Hepatocellular carcinoma
C. Liver D. Hemangiosarcoma
D. Spleen E. Hepatoblastoma
E. Greater omentum
11. A 65 years male presented with progressive 16. Gastro oesophageal reflux disease is commonly
dysphagia and weight loss since 3 years On caused by
esophagoscopy and biopsy he is diagnosed as A. Barret s oesophagus
esophageal cancer in middle 3rd of oesophagus What B. Loss of competence of the lower oesophageal
is the most common histopathological variety of sphincter (LOS)
oesophageal cancer? C. Para oesophageal hiatus hernia (Rolling)
A. Carcinoid tumour D. Achalasia
B. Adenocarcinoma E. Corrosive
C. Mucoepidermoid carcinoma
D. Anaplastic carcinoma 17. Primary intestinal tuberculosis is hyperplastic variety
E. Squamous cell carcinoma usually involves which part of GIT?
A. jejunum
12. An anal Fissure (fissure-in-ano) is a longitudinal split B. Ileo-caecal region
in the anal mucosa of the distal anal canal What is the C. Stomach
most frequent site of its occurrence D. Duodenum
A. Posterior midline at 6 o'clock position E. Rectosigmoid region
B. Left lateral at 3 o'clock position
C. Dual sites anterior and posterior midline 18. For most of the hernias, which of the following
D. Anterior midline at 12 o'clock position method of investigation is required?
E. Rt lateral at 0 o'clock position A. Ultrasound
B. Clinical examination
13. What is the most common post operative C. Laparoscopy
complication following appendicectomy after 4th post D. CT scan
operative day E. Barium contrast study
A. Basal atelectasis
B. Deep venous thrombosis 19. A 50 years obese female c/o severe epigastric pain
C. Wound infection radiating to the back diagnosed as a case of acute
D. Adhesive intestinal obstruction pancreatitis What could be the most common cause of
E. Portal pyaemia acute pancreatitis?
A. Drugs
14. Lung cancer is one of the most common cancers B. CBD stone
throughout the world and is the most common cause of C. Idiopathic
cancer death The occurrence of most common type of D. Hyperlipidemia
lung cancer is E. Alcohol
A. Small cell lung cancer
B. Squamous cell carcinoma 20. Unilateral hydronephrosis is commonly caused by
C. Brone halva olar carcinoma A. BPH
D. Adenocarcinoma B. Urethral stricture
E. Large cell undifferentiated carcinoma C. Pregnancy
D. Ureteric stone
15. A 55 year lady presenting with pain in right E. Idiopathic retroperitoneal fibrosis
hypochondrium and dyspepsia What is the primary
method of diagnosing gall stones? 21. The principal function of the colon is
A. Ultrasound upper abdomen A. Absorption of bile
B. LFT B. Absorption of vitamin B 12
C. Plain X Ray abdomen C. Absorption of fat protein and carbohydrates
D. MR CP D. Absorption of iron and folate
E. C.T scan abdomen E. Absorption of water
22. Injection sclerotherapy is the non surgical 27. What is the most common histopathologic type
modality of treatment for certain haemorrhoids of gastric carcinoma?
What is the best indication of this treatment? A. Squamous cell carcinoma
A. 3rd degree haemorrhoids B. Carcinoid tumour
B. Intemo external haemorrhoids C. Adenocarcinoma
C. 4th degree haemorrhoids D. Anaplastic carcinoma
D. 1st and 2nd degree haemorrhoids E. Sarcoma
E. Thrombosed haemorrhoids
28 The commonest urinary tract stones are
23. Carcinoma of the prostate is the most common A. Cystine
malignant tumour in men over the age of 65 years It B. Calcium phosphate
usually originates in the C. Magnesium ammonium phosphate
A. Membranous part of urethra D. Calcium oxalate
B. Peripheral zone of the prostate E. Uric acid
C. Transitional zone of the prostate
D. Central zone of the prostate 29 A hydrocoele is an abnormal collection of serous ??
E. Prostatic urethra Ans Tunica vaginalis

24 Endoscopic retrograde cholangio pancreatography 30 A glandular metaplasia of the lower oesophageal


(EliCP) is the method of investigation for the mucosa resulting from chronic gastroesophageal reflux
assessment of hepatobiliary disorders This technique is is known as
used for A. Tylosis
A. Cannot assess the level of bile duct obstruction B. Leukoplakia
B. Only diagnostic, assessment in obstructive jaundice C. Achalasia
C. Diagnostic and therapeutic modalities in biliary D. Barrett s oesophagus
obstruction E. Plummer vinson syndrome
D. Only therapeutic intervention in obstruction jaundice
E. Cannot identity the stones in CBD 31. A volvulus is a twisting or axial rotation of a portion
of bowel about its mesentery The most common site of
25. Which of the following open surgical procedure is its occurrence is
most commonly performed as tension free repair for A. Caecum
inguinal hernia? B. Transverse colon
A. Lichtenstein repair C. Small intestine
B. Shouldice repair D. Sigmoid colon
C. Bassini repair E. Stomach
D. Darn s repair
E. Bcsarda repair 32. A 60 years male presented with yellow
discolouration of sclera pruritus and weight loss since 2
26. What is the single best test to establish the 3 years On examination he has deep jaundice and
diagnosis of small bowel obstruction? gallbladder is palpable What is the most probable
A. Ultrasound abdomen clinical diagnosis?
B. X-Ray abdomen Erect and supine A. Hepatocellular carcinoma
C. Barium meal follow through B. Porcelain gallbladder
D. CT scan abdomen C. Mucocele of the gallbladder
E. MRI D. Carcinoma at head of the pancreas
E. Carcinoma of the gallbladder

33. Lower urinary tract infection and cystitis is


commonly occurs through
E. Haematogenous spread
34. A Meckel's diverticulum is a persistent remnant of 40. The most common (90%) histological type of urinary
the vitellointestinal duct and present in about 2 percent bladder carcinoma occurring is
of the population It is found in the A. Transitional cell carcinoma
A. Ileum B. Adenocarcinoma
B. jejunum C. Squamous cell carcinoma
C. Sigmoid colon D. Papillary carcinoma
D. Caecum E. Clear cell carcinoma
E. Duodenum
41. Peritonitis is simply defined as inflammation of
35. The most frequent site for blood borne metastases peritoneum Acute bacterial peritonitis is most commonly
in colorectal cancer is caused by
A. Lungs A. Ischaemic bowel disease
B. Liver B. Female genital tract infection
C. Adrenals C. Gastro intestinal perforation
D. Bones D. Primary peritonitis
E. Brain E. Pancreatitis

36. Typhoid fever is caused by salmonella typhi and 42. A middle aged male presented with haematuria in
patient usually presents with fever and abdominal pain association with left groin pain and a palpable mass in
The most common and frequent surgical complication is left groin With this classic triad of symptoms and signs
A. Cholecystitis the most probable diagnosis is
B. Ileal perforation A. Pyelonephritis
C. Paralytic ileus B. Hydronephrosis
D. Intestinal haemorrhage C. Pyonephrosis
E. Intestinal obstruction D. Psoas abscess
E. Renal tumour
37. Renal cell carcinoma is among the 10 most common
cancers worldwide Which of the following type of renal 43. A middle aged woman presented with H/0
tumour is most common? dysphagia associated with iron deficiency anaemia
A. Transitional cell carcinoma glossitis and koilonychia Her most probable diagnosis
B. Chromophobe renal cell carcinoma would be
C. Papillary renal cell carcinoma A. Plummer vinson syndrome
D. Renal cell carcinoma B. Oesophagitis due to Candida albicans
E. Wilms tumour (nephroblastoma) C. Crohn s oesophagitis
D. Carcinoma oesophagus
38. The most common site of intussusception in children E. Achalasia
is?
A. Ileocolic 44. The most frequent site for peptic ulcers is
B. Colocolic A. 1st Part duodenum
C. Ileoileal B. 2nd part of the duodenum
D. Coloanal C. 3rd part of the duodenum
E. Colorectal D. Greater curvature of the stomach
E. Fundus of the stomach
39. The most frequent cause of acute urinary retention
in elderly is 45. In hepatobiliary disorder which of the following
A. BPH statement is most appropriate regarding
B. Neurogenic bladder
C. Acute urethritis or prostatitis 46. Name of Institution:
D. Urethral calculus A. Chandka Medical College Larkana
E. Urethral stricture B. Ghulam Muhammad Mahar Medical College Sukkur
47 The spleen must be enlarged to about three limes its
normal size before it becomes clinically palpable which
of the following causes of splenomegaly needs
splenectomy?
A. Lymphoma
B. Myelofibrosis
C. Chronic myeloid leukemia
D. Idiopathic thrombocytopenic purpura
E. Polycythaemia

48. Enter Roll No. of Final Year MBBS Pre-Prof.


Online Test Session 2015-16 (SURGERY - II):

49. Appendix is a blind muscular tube its base is


attached to the caecum near the terminal What is the
common anatomical position of the tip of the appendix?
A. Pelvic
B. Rectocaecal
C. Paracaecal
D. Post ileal
E. Pre ileal

50. Bile is produced by the liver and stored in the


gallbladder from which it is released in to the duodenum
About 95 percent of bile salts are reabsorbed in which
part of GIT called as enterohepatic circulation?
A. Terminal ileum
B. Mid ileum
C. jejunum
D. Duodenum
E. Ascending colon

51. Carcinoid tumour is one of the variety of


neuroendocrine tumour occurs throughout the
gastrointestinal tract The most common site of its
occurrence is?
A. Duodenum
B. Ileum
C. Colon
D. Appendix
E. Rectum

52. For the diagnosis of upper gastrointestinal tract


pathological lesions which of the following is considered
as gold standard investigation?
A. Ultrasound upper abdomen
B. Endoscopy
C. MRI
D. Barium contrast radiology
E. CT scan upper abdomen
44 Prof (SURGERY 1) 7. Trendelenburg position is useful for management in
which type of shock?
1. In hypothyroidism, which of the following is the most A. Hypovolemic shock.
appropriate clinical sign of this condition B. Cardiogenic shock.
A. Clubbing C. Septic shock.
B. Delayed relaxation phase of ankle jerks D. Neurogenic shock.
C. Tachycardia E. Electric shock.
D. Tremors
E. Warm extremities 8. The use of prophylactic antibiotics are usually
recommended
2. A 50 years female having painless lump in Rt breast A. Two hours after surgery
with blood stained nipple discharge. O/E a single 3 x 3 B. Two hours before surgery
cm non tender, irregular hard, lump is palpable in C. One hour before surgery
central part of Rt breast, ipsilateral axillary lymph nodes D. At the time of induction of anesthesia
palpable and mobile. What investigations will be helpful E. One hour after surgery
to reach final diagnosis:
A. Tru cut needle biopsy 9. Breast conservative surgery is indicated in:
B. Doppler ultrasound A. Central breast lump less than 10 cm
C. Ultrasound breast B. Central breast lump less than 05 cm
D. FNAC C. Central breast lump less than 2.5 cm
E. Mammography D. Peripheral breast lump more than 7cm
E. Peripheral breast lump less than 3 cm
3. Which form of actinomycosis is most common:
A. Facicocervical 10. Major cause of impaired wound healing is:
B. Thorax A. Smoking
C. Ilieocecal B. Diabetes mellitus
D. Liver C. Anemia
E. Spleen D. Local tissue infection
E. Malnutrition
4. Which of the following is most frequent site of Bed
sores (decubitus ulcers) 11. Regarding evaluation of spinal cord disorders, which
A. Sacrum of the following clinical findings are characteristics of
B. Greater trochanter upper motor neuron lesion
C. Ischium A. Decreased tone - flaccid
D. Heal B. Down going plantar response
E. Occiput C. Hyperreflexia
D. Hyporeflexia
5. What is the most common cause of lymphoedema E. Sensory loss
(non-pitting oedema) worldwide?
A. Iatrogenic -radiotherapy 12. Which of the following is the most common variety
B. Filariasis of thyroid cancer?
C. Iatrogenic - lymph node resection surgery A. Papillary carcinoma
D. Leptospirosis B. Follicular carcinoma
E. Chagas disease C. Medullary carcinoma
D. Anaplastic carcinoma
6. In Amoebiasis the majority of abscesses occur in E. Lymphoma
which of the following organs?
A. Spleen
B. Right lobe of liver
C. Lungs
D. Sub diaphragmatic
E. Left lobe of liver
13. Following surgery or major trauma, which of the 19. Complications of near total thyroidectomy includes:
following metabolic changes occur in the body? A. Hyperthyroidism
A. Decreased adrenal level B. Esophageal injury
B. Decreased cortisol level C. Recurrent laryngeal nerve palsy
C. Hyperglycaemia D. Pneumothorax
D. Hypoglycaemia E. Hyperparathyroidism
E. Increased plasma albumin
20. Neoplasia of the larynx is most common in:
14. For burn patients, which of the following fluids is A. Postcricoid region.
appropriate for resuscitation? B. Lateral wall.
A. 5% Dextrose water C. Piriform fossa.
B. 10% Dextrose water D. Aryepiglottic fold.
C. Fresh frozen plasma E. Medial wall.
D. Hypertonic saline
E. Ringer’s lactate 21. Which of the following is the mainstay of
investigation for the management of varicose veins?
15. Regarding pre-operative preparation before giving A. Magnetic resonance venography
general anaesthesia, patients are advised not to take B. Duplex ultrasound imaging
solids: C. Tourniquet test
A. Within 2 hours D. Vericography
B. Within 6 hours E. Descending intravenous venography
C. Within 12 hours
D. Within 24 hours 22. In a dehydrated patient, which of the following is the
E. Within 48 hours best parameter for assessing hydration after fluid
replacement?
16. Regarding intermittent claudication: A. B.P
A. It is present on taking first step B. Pulse
B. It is usually relieved only when resting in the sitting C. Respiratory rate
position D. Temperature
C. It is typically relieved when resting for more than 5 E. Urine output
minutes
D. It is caused by anaerobic muscle metabolism 23. The most common fracture of face is that of:
E. It is due to chronic venous disease A. Mandible
B. Maxilla
17. Intermittent claudication is a cramp-like pain felt in C. Zygoma
the limb muscles, which of the following statements is D. Nasal bone
appropriate? E. Orbital bone
A. The pain felt in the lower limb at rest
B. The pain felt on taking the first step 24. 45 years female C/0 painless lump in Rt breast with
C. The pain brought on by walking and relieved by low grade fever and weight loss O/E Pt: is thin built,
standing still having palpable man tender lump with multiple
D. The foot is often cold and paralyzed discharging sinuses and bilateral axillary lymph nodes
E. Ulceration and gangrene of toes are usually are enlarged with matting: What is most probable
associated with even mild cases diagnosis:
A. Mondor’s disease
18. Branchial cyst is best differentiated from cold B. Antibioma
abscess by: C. Hidradenitis suppurativa
A. Fluctuant. D. Ca breast
B. Trans illumination . E. TB mastitis
C. Contains cholesterol crystal
D. Contains sulphur granules
E. Contains sulphur granules
25. Haemorrhage must be recognized and managed 33. Antibiotic prophylactic
aggressively to reduce the severity and duration of Ans Clean + Implant
shock and multiple organ failure. The best treatment of
haemorrhage is: 34. Burn ?
A. Arrest the bleeding
B. Blood transfusion 35…
C. Fluid resuscitation by colloids
D. Fluid resuscitation by Crystalloids (SURGERY 2)
E. Intravenous tranexamic (tranexamic acid ) 1000-
2000 mg stat 1. Tumor marker of Yolk Sac Tumor
Ans AFP
26. During submandibular salivary gland surgery, which
of the following nerves may likely be at risk? 2. Colorectal carcinoma is the second most common
A. Facial nerve malignancy in western countries. Which is the most
B. Glossopharyngeal nerve common histological type?
C. Lingual nerve A. Squamous cell carcinoma
D. Trigeminal nerve B. Malignant melanoma
E. Vagus nerve C. Lymphoma
D. Sarcoma
27. A 20 years old man receives multiple blood E. Adenocarcinoma
transfusions for abdominal gunshot wound. He
complains of numbness around his mouth and displays 3. Staging Of rectal cancer is best done by:
carpopedal spasm and the positive Chvostek sign. The A Digital rectal examination (DRE)
treatment required is: B. Sigmoidoscopy
A. Intravenous bicarbonate C. Colonoscopy
B. Intravenous potassium D. C.T scan pelvis
C. Intravenous calcium E. Endoluminal ultrasound
D. Intravenous digoxin
E. Intravenous parathormone 4. Colorectal staging MRI

28. The most common direction of dislocation of the 5. The most accurate procedure for the diagnosis of
shoulder joint is? gastritis is:
A. Antero-inferior A. X ray abdomen
B. Anterosuperior B. Ultrasound abdomen
C. Inferioposterior C. CT scan abdomen
D. Posterior D. Endoscopy
E. Superior E. Barium meal

29. Energy requirement 6. 25 year old male suffered blunt trauma to the chest,
Ans 20%-30% in the ER he presented with shortness of breath and
cyanosis On Examination no breath sounds in the right
30. Regarding development of postoperative DVT, side and percussion note resonant The most likely
which of the following surgical procedures is associated diagnosis is
with high risk of DVT A. Cardiac contusion
Ans Pelvic bone B. Spontaneous pneumothorax
C. Cardiac tamponade
31. Snuff box D. Pericardial effusion
Ans Scaphoid bone E. Tension pneumothorax

32. 80% sialolithiasis


Ans Submandibular
7. Hernia is a protrusion of viscus or part of a viscus 14. The most common site of intussusception in children
through an abnormal opening. Which of the following is?
has high risk of strangulation A. Ileocolic
A. Epigastric hernia B. Colocolic
D. Direct inguinal hernia C. Ileoileal
C. Paraumbilical hernia D. Coloanal
D. Femoral hernia E. Colorectal
E. Indirect inguinal hernia
15. Which of the following is a characteristic feature of
8. Regarding Wilms tumor which one is correct? intussusceptions ?
Ans. On examination stromal and epithelial appearance A. Constipation
B. Diarrhea
9. Pancreatic tumor C. Passage of Mucus in stool
Ans Head D. Passage of Blood and mucus in stool
E. Red currant jelly stool
10. A middle aged patient c/o severe abdominal pain
with past history of dyspepsia. His x-ray chest 16. Colorectal cancer is the second most common
demonstrates free air under right dome of diaphragm. cause of cancer death in the west. Regarding
Which of the following is the most likely possibility distribution of colorectal cancer, which is the most
A. Acute cholecystitis frequent site of involvement
B. Acute pancreatitis A. Ascending colon
C. Perforated appendicitis B. Caecum
D. Perforated duodenal ulcer C. Descending colon
E. Ruptured liver abscess D. Rectosigmoid
E. Transverse colon
11. Good Sall's rule is a clinical assessment for certain
diseases. In which of the following conditions this rule is 17. A 50 year female presented with a mass in right iliac
applied ? fossa, he was diagnosed as carcinoma caecum, which
A. Biliary fistula of the following surgical procedure is appropriate for this
B. Fistula in ano condition?
C. Genitourinary fistula A. Right hemicolectomy
D. Perianal sinus B. Total colectomy
E. Pilonidal sinus C. Hartmann's procedure
D. Paul mikulicz procedure
12. The best conservative treatment of anal fissure is E. Extended right hemicolectomy
A. High fiber diet
B. Stool softness 18. Charcot’s triad of symptoms is present in:
C. 5% xylocaine cream tropically A. Bile duct stone
D. Chemical sphincterotomy B. Cholelithiasis
E. lateral internal sphincterotomy C. Hepatitis
D. Portal pyemia
13. Appendix is a blind muscular tube its base is E. Porcelain Gall bladder
attached to the caecum near the terminal What is the
common anatomical position of the tip of the appendix?
A. Pelvic
B. Rectocaecal
C. Paracaecal
D. Post ileal
E. Pre ileal
19. A 50 years old patient having uncontrolled diabetes 25. A glandular metaplasia of the lower oesophageal
presented in emergency with right upper quadrant pain mucosa resulting from chronic gastroesophageal reflux
& jaundice. U/S suggestive of acute cholecystitis. What is known as
will be the best treatment option? A. Tylosis
A. Cholecystectomy B. Leukoplakia
B. Conservative treatment C. Achalasia
C. Laparoscopic cholecystectomy D. Barrett s oesophagus
E. Plummer vinson syndrome
20. A 40 years old school teacher presents with flatulent
dyspepsia for last 2 years. Ultrasonography shows thick 26. Inguinal hernia is the most common surgical
walled gall bladder with a solitary 1 cm stone in the problem. Which is the most appropriate for direct
fundus. The best treatment for her would be: inguinal hernia
A. ESWL. A. It is more likely in younger patients
B. Bile salts. B. It is likely to strangulate earlier
C. Open cholecystectomy. C. It usually comes from deep inguinal ring
D. Mini-cholecystectomy. D. It usually comes from Hasselbach’s triangle
E. Laparoscopic cholecystectomy. E. Should be repaired surgically in emergency

21. In Barium swallow, which radiologic finding is 27. Regarding peritonitis


diagnostic in case of Achalasia? Ans whole abdomen is rigid
A. Coffee Bean sign
B. Bird Beak appearance 28. Chronic gastritis
C. Claw sign Ans least gastric malignancy
D. Double Bubble
E. Air-fluid level 29. A 22 year old lady presented in OPD with the
complain of bleeding per rectum after defecation and
22. A 23 years male is having splenomegaly and something coming out of anus which reduced
tendency to bleed upon minor injury. What could be the spontaneously. Determine the degree of hemorrhoids?
cause of this condition which needs splenectomy? A. 1st Degree
A. Acute leukaemia B. 2nd Degree
B. Chronic myeloid laukaemia C. 3rd Degree
C. Idiopathic thrombocytopenic purpura D. 4lh Degree
D. Polycythaemia E. 5th Degree
E. Pernicious anaemia
30. Intestinal tuberculosis can affect any part of the
23. The most common site of occurrence of peptic ulcer gastrointestinal tract from mouth to the anus. The most
is in the: common site affected much often is:
A. 1st part of the duodenum A. Duodenum
B. 2nd part of the duodenum B. Ileum
C. 3rd part of the duodenum C. Jejunum
D. 4th part of the duodenum D. Rectum
E. Greater curvature of the stomach E. Sigmoid colon

24. Laparotomy in trauma 31..


Ans CVS decompression 32..
33..
34..
35..
43 Preprof 6. The best suture material used for vascular
anastomosis is:
1. During ‘the catabolic phase of the stress response A. Catgut
muscle wasting occurs as a result of an increase in B. Polypropylene (prolene)
muscle protein degradation. The major site of protein C. PDS (polydioxanone)
loss occurs in: D. Silk
A. Cardiac muscles E. Vicryl (polyglycolic acid)
B. Central truncal skeletal muscles
C. Peripheral skeletal muscles 7. Surgical drains are inserted to allow fluid or air
D. Respiratory muscles that might collect at an operation site. Under-water
E. Smooth muscles of gut seal drain is used to collect fluid after which surgical
procedure?
2. The main labile energy reserve in the body is: A. Laparotomy
A. Brain B. Mastectomy
B. Bone C. Hysterectomy
C. Fat D. Thoracotomy
D. Liver E. Thyroidectomy
E. Skeletal muscle
8. The best time of prophylactic antibiotics to be given
3. In shock as tissue perfusion is reduced, cells are in surgery is:
deprived of oxygen and must switch from aerobic to A. within 4 hours
anaerobic metabolism. The product of anaerobic B. After 4 hours
respiration is: C. After 5 hours
A. Ammonia D. After 12 hours
B. Carbon dioxide (co2) E. Before 12 hours
C. Lactic acid
D. Uric acid 9. Which of the following organisms are responsible for
E. urea gas gangrene:
A. Clostridium perfringens
4. The wounds caused by explosions or bullets are B. Clostridium tetani
usually managed by: C. Clostridium welchii
A. Primary closure of the wound by sutures D. Streptococcus pyogenes
B. Primary repair of all damaged structures (bone, E. Staphylococcus
tender, nerve) may be attempted.
C Skin cover by flap or graft may be required if 10. Hydatid disease is caused by echinococcus
there is skin loss granulosus. The most commonly involved organ is:
D. A thorough debridement of the wound is essential A. Bone
to make it a tidy.wound, later require closure B. Brain
E. Only irrigate the wound with copious saline C. Liver
solution and advised daily dressing D. Lung
E. Spleen
5. Compartment syndromes typically occur In closed
lower limb Injuries, it Is best managed by: 11. During Laparoscopy the most common reason for
A. Elevation of the affected limb & close observation conversion of laparoscopic cholecystectomy to open
B. Intra venous antibiotics procedure is:
C. intra venous diuretics A. Difficult dissection of calot's triangle
D. Early fasciotomy B. Adhesion of gallbladder to liver bed
E. Late fasciotomy C. Perforation of the gallbladder
D. Spillage of stones
E. Uncontrolled bleeding
12. Testicular torsion is most common in adolescents 17. The prevalence of postoperative deep venous
but may occur at any age. The best approach of thrombosis is (DVT) is high in hospitalized patients.
treatment Is: Which of the following surgical procedure has high risk
A. Antibiotics & analgesic of DVT?
B. Scrotal support A. Cardiothoracic surgery
C. Wait for 2-3 days till diagnosis is obvious B. Maxillofacial surgery
D. Advise MRI to confirm the diagnosis C. Neurosurgery
E. The scrotum must be explored urgently if torsion D. Urological surgery
cannot be excluded E. Total knee and hip replacement surgery

13. In older infants & children, the most common cause 18. A 50 year old man brought to emergency with
of rectal bleeding is: Injuries in Rt: chest and femoral fracture. He is
A. Internal haemorrhoids conscious, BP b 80/60 Pulse is 104/min and heart
B. Fissure in ano sounds normal Respiratory rate Is 22/min. What does
C. Juvenile polyps he have?
D. Necrotizing enterocolitis A. Fat embolism
E. Rectal cancer B. Cardiac tamponade
C. Hypovolaemia
14. The inheritance and environment are two most D. Left heart failure
important determinants of cancer development. Which E. Pneumothorax
of the following malignancies is associated with
chemicals used in industry? 19. Which of the following pattern of head injury is
A. Hepatocellular carcinoma labeled as neurosurgical emergency require
B. Head and neck cancer immediate surgical intervention?
C. Stomach cancer A. Cerebral contusion
D. Thyroid cancer B. Diffuse axonal injury
E. Urinary bladder cancer C. Extradural haematoma
D. Subdural haematoma
15. While selecting imaging method for the diagnosis of E. Subarachnoid haematoma
certain diseases. Which of the following technique has
high radiation dose hazards: 20. In abdominal trauma, during laparotomy liver
A. CT scan bleeding can be controlled by Pringle's Maneuver. This
B. MRI Maneuver is described as:
C. Intravenous pyelography (IVP) A. Direct compression of the liver
D. Radioisotope scanning B Direct compression of the portal triad digitally or by
E. X-ray KUB using a soft clamp
C. Any hole due to penetrating injury can be plugged
16. Following surgical resection of terminal ileum more D. Four quadrant packing of the liver
than 100cm causing short bowel syndrome. Which of E. Direct suturing of the liver tear
the following complication may be associated with this
condition: 21. The term Escharotomy is the surgical incision in
A. Ca colon burn patients. This is indicated in:
B. Cholelithiasis A. Superficial partial thickness bums
C. Hyperglycaemia B. Deep partial thickness bums
D. Iron-deficiency anemia C. Full thickness bums to the trunk
E. Liver cirrhosis D. Circumferential full thickness bums to the limbs
E. Electric bums
22. During disaster, Triage has been the cornerstone of 26. A 35 years female operated for Ca breast, and
the management of mass casualties in emergency. conservative breast surgery was performed with
The aim of Triage is: placement of implant. Which investigation is useful to
A- An accurate assessment of the damage and the differentiate scar from local recurrence?
number of casualties A. FNAC
B. Early co-ordination of the rescue effort allows optimal B. Mammogram
use of limited resources. C. MRI
C. Coordination with relief agencies D. U/S
D. Identify the patients who will benefit the most by E. X-ray chest
being treated the earliest
E. Mobilization Of human and material resources 27. A 35 years female non lactating, smoker comes to
appropriate to the size and nature of the disaster. OPD with painless lump in her left breast beneath
areola associated with slit like nipple retraction and
23 Bilateral ovarian masses removed from a 55 year serous discharge from the nipple. Wilat is the most
female show poorly differentiated mucinous probable diagnosis of this condition?
adenocarcinoma. If ovaries are secondarily involved, A. Breast abscess
which of the following is the most likely primary site of B. Ca breast
patients disease. C. Fibrocystic disease
A. Breast D. Periductal mastitis
B. Lung E. Tuberculosis
C. Liver
D. Stomach 28 A 40 years non lactating female comes to OPD with
D. Kidney painless lump in her breast. O/E lump is 7x7cm, mobile,
having variable consistency, overlying skin is thin with
24 A 79 year old retired singer presents with dysphagia, prominent veins. Axillary lymph nodes are not palpable.
which has progressively worse during the last 5 years. FNAC shows spindle cells What is the likely diagnosis?
He stales that frequently he notices a lump on left side A. Antibioma
of his neck & he hears gurgling sounds during B. Ca of breast
swallowing, and regurgitate food during eating What is C. Fibroadenoma
likely diagnosis? D. Phyllodes tumor
A. Ca esophagus E. TB mastitis
B. Foreign body in esophagus
C. Peterson-Brown Kelly syndrome 29. Which of the following clinical test can differentiate
D. Scleroderma between direct and indirect inguinal hernia?
E. Zenker's diverticulum A. Cough impulse test
B. Fluctuation test
25. A 54 year old clerk c/o of dysphagia for 15 years. C. Ring occlusion test
Diagnosis of achalasia is confirmed by Barium study. D. Ring invagination test
What is the most appropriate statement about this E. Transillumination test
condition?
A Endoscopic dilatation should be avoided. 30 The surgical procedure Lichtenstein tension free
B. It In early stages, dysphagia is more pronounced for repair is commonly performed for
solids than liquids A. Diaphragmatic hernia
C. It is caused by trypanosomia cruzi B. Epigastric hernia
D. Recurrent pulmonary infection is rare C. Inguinal hernia
E. The most common symptom is dysphagia for liquids D. Paraumbilical hernia
E. Umbilical hernia
31. Bilateral hydronephrosis Is caused by: 36. A 62 year old man is admitted with a two day history
A. Benign prostatic enlargement or carcinoma of the of bloody diarrhea and abdominal cramps. He has
prostate. recently started on Clindamycin for septic arthritis as he
B. Left sided mega ureter. is allergic to penicillin, Which one of the following is the
C. Retrocaval ureter. most likely diagnosis?.
D. Ureterocele. A. Campylobacter jejuni diarrhea
E. Vesicoureteric reflux in lower moiety. B. Pseudomembranous colitis
C. Heamorrhoids
32. Which of the following renal stones are radiolucent D. Rectal cancer
on X-Ray KUB: E. Ulcerative colitis
A. Calcium oxalate.
B. Calcium phosphate. 37. After splenectomy, patients are more at risk of
C. Cystine. severe infection by encapsulated organisms, and
D. Mixed. indeed all kinds of infection. It is particularly important
E. Uric acid. that patients are aware of which of the following
infections?
33. Nausea and vomiting are commonly associated with A. Human immunodeficiency virus
ureteric colic due to: B. Tuberculosis
A. Back flow of urine. C. Malaria
B. Somatic nervous system transmits pain. D. Pertussis
C. Stone is impacted in kidney or ureter. E. Herpes Simplex Virus
D. The coeliac ganglion serves both kidneys and
stomach. 38. A 48 year old woman presented with right
E. Urinary tract infection is associated with hyperacidity. abdominal pain, nausea & vomiting. On examination,
she had tenderness in the right hypochondrial area.
34. A 60 year male complaining of painless haematuria. Investigations showed high WBC count, high alkaline
Cystoscopy and biopsy reveals Carcinoma of the phosphatase & high bilirubin level. The most likely
urinary bladder. Which of the following statement be diagnosis is:
most appropriate for the condition A. Acute cholecystitis
A. Over 90% are transitional cell in origin B. Acute appendicitis
B. Simple cystectomy is the choke of treatment for C. Perforated peptic ulcer
muscle invasive cancer D. Acute pancreatitis
C. Stage T2 means extension to the lamina propria E. Acute cholangitis
D. Superficial bladder cancer have extension to the
detrusor muscle Il 39. A 55 year old lady underwent a left sided modified
E. Ultrasonography is a confirmatory test. radical mastectomy (MRM) and axillary clearance for
breast cancer. After the operation she was found to
35. A 69 years old lady complaining of profused mucus have a winging scapula. She is otherwise well. What is
discharge per anum. On DRE a large pedunculated the most likely cause of this condition?
frond like mass palpable 4 cm from anal verge. Her A. Patient has suffered a TIA
blood report show hypokalaemia. The most likely B. Damage to the thoracodorsal nerve during the
diagnosis is: surgery
A. Haemangioma C. Damage to the long thoracic nerve during the surgery
B. Hyperplastic polyp D. Damage to the serratus anterior muscle during the
C. Serrated adenoma surgery
D. Tubular adenoma E. Dislocated scapula
E. Villous adenoma
40. The most precise diagnostic screening procedure 45. A 48 years female presented with obstructive
for differentiating benign thyroid nodules from malignant jaundice. U/s abdomen revealed dilated CBD with
ones is: presence of single stone in it. What is the most
A. Fine-needle-aspiration cytology FNAC preferred technique for Its removal
B. Thyroid ultrasonography A. ERCP & removal of stone
C. Thyroid scintiscan B. Laparoscopic CBD exploration
D. Thyroid antibodies C. Percutaneous choledochoscopy
E. Thyroid profile D. Percutaneous transhepatic cholangiography
E. Surgery and choledochotomy
41. A 25 years old university student undergoes total
thyroidectomy for multinodular goiter. During this 46. Intestinal tuberculosis can affect any part of the
operation inferior thyroid artery is ligated. The nerve gastrointestinal tract from mouth to the anus. The most
intimately related to this artery is: common site affected much often is:
A. Internal laryngeal nerve. A. Duodenum
B. External laryngeal nerve. B. Ileum
C. Superior laryngeal nerve. C. Jejunum
D. Recurrent laryngeal nerve D. Rectum
E. Non-recurrent laryngeal nerve E. Sigmoid colon

42. A 50 years old diabetic house maid presents with 47. Colorectal cancer is the second most common
right upper quadrant discomfort, accompanied by fever, cause of cancer death in the west. Regarding
anorexia and malaise for last 1 week. Examination distribution of colorectal cancer, which is the most
reveals tender enlarged liver. Ultrasonography reveals a frequent site of involvement
cystic lesion in the liver. The diagnosis in this case A. Ascending colon
would be: B. Caecum
A. Hydatid liver disease C. Descending colon
B. Haemangioma of liver D. Rectosigmoid
C. Amoebic liver abscess E. Transverse colon
D. Pyogenic liver abscess T
E. Polycystic liver disease 48. Careful clinical examination will be diagnostic in the
vast majority Of patients complaining of anal symptoms.
43. A 25 years old political worker was operated upon Which is the most common position for examination of
for perforated appendix 5 days ago and was put on anal pathology?
antibiotics. Now he has developed pyrexia, and A. Jack-knife position
diarrhoea with passage of mucus in stools. Rectal B. Knee-elbow position
examination reveals bulging of anterior rectal wall. The C. Lithotomy position
diagnosis in this case would be: D. Left lateral (sims) position
A. Amoebiasis. E. Right lateral position
B. Pelvic abscess
C. Gastroenteritis 49: Thyrotoxicosis differs from hyperthyroidism due to
D. Bacillary dysentery presence of:
E. Antibiotics induced diarrhoea A. Myopathy and eye signs
B. Palpitation
44. A middle aged patient c/o severe abdominal pain C. Tachycardia
with past history of dyspepsia. His x-ray chest D. Thyroid swelling
demonstrates free air under right dome of diaphragm. E. Weight loss despite good appetite
Which of the following Is the most likely possibility
A. Acute cholecystitis
B. Acute pancreatitis
C. Perforated appendicitis
D. Perforated duodenal ulcer
E. Ruptured liver abscess
50. A 70 year old man presents with 6 years history of 55. A 20 years old man receives multiple blood
pruritus ani, the Itching and discomfort has Increased transfusions for abdominal gunshot wound. He
during last few months. There is neither rectal bleeding complains of numbness around his mouth and displays
nor pain Physical signs suggest chronic dermatitis, and carpopedal spasm and the positive Chvostek sign. The
an irregular ulcer on the left anal margin.what is the best treatment required is:
next step In management: A. Intravenous bicarbonate
A. Wide local excision B. Intravenous potassium
B. Steroid ointment C. Intravenous calcium
C. Nitroglycerine ointment D. Intravenous digoxin
D. Internal sphincterotomy E. Intravenous parathormone
E. Biopsy
56. Which of the following is the immediate post-
51. Which of the following is most commonly associated operative complication after abdominal surgery?
with sclerosing cholangitis: A. Anastomotic leakage
A. Crohn's disease B. Intra-abdominal sepsis
B. Ulcerative colitis C. Paralytic ileus
C. Riedel thyroiditis D. Wound dehiscence
D. Sarcoidosis E. Wound infection
E. Retroperitoneal fibrosis
57. A 25 years male brought to casualty with history of
52. The commonest clinical feature of carcinoma head blunt abdominal trauma, C/O severe chest pain O/E
of pancreas is: pulse 104 per minute, chest x-ray revealed air fluid level
A. Epigastric pain in the left side of chest. What is the most probable
B. Anorexia and malaise diagnosis?
C. Thrombophlebitis. A. Diaphragmatic hernia
D. Obstructive jaundice B. Empyema thoracic
E. Mass in the abdomen C. Lung Abscess
D. Pleural effusion
53. 25 years female C/O Abdominal pain, vomiting, E. Pneumothorax
distention & absolute constipation since 5 days, O/E she
has tachycardia, whole abdomen is rigid & tender bowel 58. A patient underwent open reduction and internal
sounds are absent. What is the most probable fixation of fracture femur, complaining of severe limb
diagnosis? pain that is felt even worse on passive stretching of
A. Cholecystitis muscles. Limb found swollen and tense with altered
B. Intestinal obstruction sensations distally. Most likely diagnosis is:
C. Pancreatitis A. Femoral artery occlusion
D. Peritonitis B. Compartment syndrome
E. Pyelonephritis C. Development of localized hematoma
D. Missed muscle injuries
54. In hypothyroidism, which of the following is the most E. Prosthesis displacement
appropriate clinical sign of this condition
A. Clubbing 59 Obesity b defined us:
B. Delayed relaxation phase of ankle jerks A. BMI>25
C. Tachycardia B. BMI>30
D. Tremors C. BMI>35
E. Warm extremities D. BMI>40
E. BMI > 45
60. Which Of the following is the best surgical
approach for performing prostatic surgery:
A. Retropubic
B. Transperineal
C. Transperitoneal
D. Transvesical
E. Transurethral
43 Prof (SURGERY 1) 7. Intramedullary nailing is a method of open reduction
and internal fixation, used in fracture surgery. This
1. A patient with localized wound Infection after surgery procedure is best used for:
should be treated with? A. Clavicle fracture
A. Antibiotics and warm soaks to wound B. Digital fracture
B. Antibiotics alone C. Long bone fracture
C. Antibiotics and opening the wound D. Mandibular fracture
D. Antibiotics and closing the wound E. Fibula fracture
E. Incision and drainage alone
8. High risk lesion associated with malignant
2. In Amoebiasis the majority of abscesses occur in transformation to oropharyngeal cancers includes:
which of the following organs? A. Discoid lupus erythematosus
A. Spleen B. Erythroplakia
B. Right lobe of liver C. Oral sub mucous fibrosis
C. Lungs D. Oral lichen planus
D. Sub diaphragmatic E. Syphilitic Glossitis
E. Left lobe of liver
9. A 25-year-old male suffered blunt trauma to the
3. Regarding pre-operative preparation before giving chest. In the ER, he presented with shortness of breath
general anaesthesia, patients are advised not to take and cyanosis. On examination, no breath sounds were
solids: heard on the right side, and the percussion note was
A. Within 2 hours resonant. The most likely diagnosis is:
B. Within 6 hours A. Spontaneous pneumothorax.
C. Within 12 hours B. Cardiac tamponade.
D. Within 24 hours C. Cardiac contusion.
E. Within 48 hours D. Tension pneumothorax.
E. Pericardial effusion.
4. Which of the following is the mainstay of investigation
for the management of varicose veins? 10. A 50 years female having painless lump in Rt
A. Magnetic resonance venography breast with blood stained nipple discharge. O/E a single
B. Duplex ultrasound imaging 3 x 3 cm non tender, irregular hard, lump is palpable in
C. Tourniquet test central part of Rt breast, ipsilateral axillary lymph nodes
D. Vericography palpable and mobile. What investigations will be helpful
E. Descending intravenous venography to reach final diagnosis:
A. Tru cut needle biopsy
5. For burn patients, which of the following fluids is B. Doppler ultrasound
appropriate for resuscitation? C. Ultrasound breast
A. 5% Dextrose water D. FNAC
B. 10% Dextrose water E. Mammography
C. Fresh frozen plasma
D. Hypertonic saline 11. A 30-year-old man presents to the Emergency
E. Ringer’s lactate Department following a high speed motor vehicle
accident. He has marked abdominal distension, a pulse
6. During submandibular salivary gland surgery, which rate of 130 and a blood pressure of 80/50 mmHg. The
of the following nerves may likely be at risk? most appropriate initial investigation would be
A. Facial nerve A. Abdominal angiogram
B. Glossopharyngeal nerve B. Abdominal paracentesis
C. Lingual nerve C. CT scan of the abdomen
D. Trigeminal nerve D. plain X-ray of the abdomen
E. Vagus nerve E. FAST (focused abdominal sonography for trauma)
scan
12. A 7-year-old boy is brought to the emergency 17. Regarding management of arterial stenosis or
department by his parents several minutes after he fell occlusion which of the following is the best among
through a window. He is bleeding profusely from a 6-cm non-surgical options:
wound of his medial right thigh. Immediate management A. Stop smoking
of the wound should consist of B. Transluminal angioplasty and stenting
A. Application of a tourniquet. C. Peripheral vasodilators
B. Direct pressure on the wound. D. Reduce weight
C. Packing the wound with gauze. E. Beta blockers
D. Direct pressure on the femoral artery at the groin.
E. Debridement of devitalized tissue. 18. In carcinoma of the breast, the most frequent site of
skeletal metastasis is:
13. Which of the following pattern of head injury is A. Femur
labeled as neurosurgical emergency require B. Lumbar vertebrae
immediate surgical intervention? C. Ribs
A. Cerebral contusion D. Skull
B. Diffuse axonal injury E. Pelvis
C. Extradural haematoma
D. Subdural haematoma 19. What is the most common cause of lymphoedema
E. Subarachnoid haematoma (non-pitting oedema) worldwide?
A. Iatrogenic -radiotherapy
14. A 20 year old male presented with small swelling in B. Filariasis
front of neck in midline, swelling moves upward on C. Iatrogenic - lymph node resection surgery
protrusion of tongue. What is diagnosis D. Leptospirosis
A. Thyroglossal cyst E. Chagas disease
B. Branchial cyst
C. Thyroid nodule 20. "Bones, stones, groans, and moans" relate to:
D. Submental lymph node A. Hyperparathyroidism.
E. Cystic hygroma B. Hypoparathyroidism.
C. Hyperthyroidism.
15. The most precise diagnostic screening procedure D. Hypothyroidism.
for differentiating benign from malignant thyroid nodules E. Pheochromocytoma.
is:
A. Fine needle aspiration cytology (FNAC) 21. A patient after parotidectomy complains of swelling
B. Thyroid scan on eating food, most likely diagnosis is
C. Thyroid ultrasound A. Wound infection.
D. Thyroid antibodies B. Frey's syndrome.
E. Thyroid profile C. Hypertrophied scar.
D. Reactionary bleeding.
16. The wounds caused by explosions or bullets are E. Altered sensation in front of tragus.
usually managed by:
A. Primary closure of the wound by sutures 22. A sinus is a blind tract lined by granulation tissue,
B. Primary repair of all damaged structures (bone, which of the following is considered to be congenital
tender, nerve) may be attempted. sinus?
C Skin cover by flap or graft may be required if A. Actinomycosis
there is skin loss B. Hidradenitis suppurativa
D. A thorough debridement of the wound is essential C. Perianal sinus
to make it a tidy.wound, later require closure D. Pilonidal sinus
E. Only irrigate the wound with copious saline E. Preauricular sinus
solution and advised daily dressing
23. 50 years female having swelling in front of neck for 32. A patient after multiple blood transfusions develops
five years associated with dyspnea, dysphagia for six carpopedal spasm, chvostek sign is positive
months, O/E swelling moving with deglutition, veins of Ans Hypocalcemia
face, neck and anterior chest wall are engorged,
cervical lymph nodes are not palpable. What is the most 33. Regarding rodent ulcer
likely diagnosis? Ans Rolled Edges
A. Simple multi nodular goiter
B. Ca thyroid 34. ?
C. Simple diffuse goiter
D. Retrosternal goiter 35. ?
E. Primary toxic goiter
(SURGERY 2)
24. While discussing with your trainees about
management of patients presenting with breast lumps, 1. Upper Gl- Endoscopy:
you attempt to test the baseline knowledge of trainees A. Is used for treatment of oesophageal varices
in this context, by asking about term "Triple B. H : pylori should be seen in the stomach by
Assessment". What is the most accurate explanation of endoscopy
triple assessment? C. Is a gold standard investigation in GERD
A. Clinical assessment, U/S breast & mammography D. Has therapeutic role in colonic polyp
B. Clinical assessment, U/S breast & CT scan chest E. Eradicates H pylori
C Clinical assessment, U/S breast & tissue sampling
taken for either cytological or histological analysis 2. A 65 years male presented with progressive
D. U/S breast, CT scan chest & FNAC dysphagia and weight loss since 3 years On
E. U/S breast, MRI & Trucut needle biopsy esophagoscopy and biopsy he is diagnosed as
esophageal cancer in middle 3rd of oesophagus What
25. Swelling in anterior triangle of neck with -ve is the most common histopathological variety of
transillumination test oesophageal cancer?
Ans Brachial Cyst A. Carcinoid tumour
B. Adenocarcinoma
26. Correct regarding response to surgery C. Mucoepidermoid carcinoma
Ans decrease insulin, increase glucagon, increase D. Anaplastic carcinoma
cortisol E. Squamous cell carcinoma

27. Type of shock in which extremities are warm due to 3. Commonest site for colorectal cancer.
vasodilation of skin vessels A. Caecum
Ans Septic Shock B. Descending colon
C. Transverse colon
28. Regarding burn's criteria, following should be D. Recto sigmoid junction
transferred to burn centre E. Rectum
Ans Adult having 25% burns
4. Post Splenectomy sepsis syndrome:
29. Most common site of tongue Cancer A. Commonly occurs in children after splenorrhaphy in
Ans Lateral site traumatic rupture of spleen
B. Caused by viruses
30. Most common cause of DVT is C. The risk is greater in old age
Ans Immobilization D. Could be avoided by appropriate and timely
immunization and antibiotic prophylaxis
31. A patient develops fever on 10th day, E. The risk is greater after traumatic Splenectomy
post-operatively
Ans DVT
5. Staging Of rectal cancer is best done by: 10. Carcinoid tumour is one of the variety of
A Digital rectal examination (DRE) neuroendocrine tumour occurs throughout the
B. Sigmoidoscopy gastrointestinal tract The most common site of its
C. Colonoscopy occurrence is?
D. C.T scan pelvis A. Duodenum
E. Endoluminal ultrasound B. Ileum
C. Colon
6. A 52 years obese woman presents with colicky with D. Appendix
abdominal pain and vomiting. She has recurrent history E. Rectum
of pain in RHC and dyspepsia. X-ray abdomen reveals
multiple air-fluid levels and air in biliary tree. What is the 11. A 25 years old political worker was operated upon
most likely diagnosis? for perforated appendix 5 days ago and was put on
A. Abdominal adhesions antibiotics. Now he has developed pyrexia, and
B. Abdominal lymphosarcoma diarrhoea with passage of mucus in stools. Rectal
C. Carcinoma of the right colon examination reveals bulging of anterior rectal wall. The
D. Gallstone ileus diagnosis in this case would be:
E. Ileo-caecal tuberculosis A. Amoebiasis.
B. Pelvic abscess
7. Ascending cholangitis is an infection of the biliary C. Gastroenteritis
tract usually associated with obstruction, and patient D. Bacillary dysentery
presents with pain, fever and jaundice. The most E. Antibiotics induced diarrhoea
probable cause of this condition is:
A. As a complication of ERCP 12. Rectal prolapse
B. CBD stone A. In children it is associated with constipation
C. Cholangiocarcinoma B. In children it is usually full thickness
D. Primary biliary cirrhosis C. Surgery is necessary for full thickness
E. Primary sclerosing cholangitis D. In children conservative treatment has no role
E. Abdominal rectopexy may lead to severe diarrhea
8. A 40 years old school teacher presents with flatulent
dyspepsia for last 2 years. Ultrasonography shows thick 13. The diagnostic feature of urethral injury is
walled gall bladder with a solitary 1 cm stone in the A. Palpable urinary bladder
fundus. The best treatment for her would be: B. Urinary Incontinence
A. ESWL. C. Urethral bleeding
B. Bile salts. D. Perianal haematoma
C. Open cholecystectomy. E. Acute urinary retention
D. Mini-cholecystectomy.
E. Laparoscopic cholecystectomy. 14. What is the most common serious complication of
an end colostomy?
9. Which one of the following Is the commonest site of A. Bleeding
ulcerative colitis: B. Skin breakdown
A. Ascending colon C. Parastomal hernia
B. Descending colon D. Colonic perforation during irrigation
C. Transverse colon E. Stomal prolapse
D. Sigmoid colon
E. Rectum 15. The best surgical approach to the prostate for BPH
is:
A. Perineal prostatectomy
B. Retropubic prostatectomy
C. Transvesical prostatectomy
D. Transperitoneal Prostatectomy
E. Transurethral prostatectomy
16. The most common site of occurrence of peptic ulcer 28. Most Gravid Complication of Ulcerative Colitis
is in the: Ans Intestinal Perforation
A. 1st part of the duodenum
B. 2nd part of the duodenum 29. Most common cause of dynamic intestinal
C. 3rd part of the duodenum obstruction
D. 4th part of the duodenum Ans Adhesions & bands
E. Greater curvature of the stomach
30. Regarding Laparoscopic Appendectomy
17. The best conservative treatment of anal fissure is Ans Performed in obese woman with unknown etiology
A. High fiber diet of lower abdominal pain
B. Stool softness
C. 5% xylocaine cream tropically T 31. Regarding hemorrhoids
D. Chemical sphincterotomy Ans Most commonly occurs in pregnancy and obesity
E. lateral internal sphincterotomy
32. A patient with 4 month history of distension and
18. A patient develops perforation of whole abdomen difficulty in defecation with no rectal bleeding, patient
after post-cholecystectomy, most likely treatment option was operated for hemorrhoidectomy 4 months back,
is most likely it is
Ans Laparotomy Ans Postoperative Stricture

19. Following Sign and symptom carries 2 points in 33. A patient was operated for inguinal hernia, he
Alvarado score develops ipsilateral testicular swelling and tenderness
AnsRight Iliac Fossa tenderness Ans Testicular Artery Compression

20. Most common hernia in men 34. 17 years old develops sudden vomiting, nausea and
Ans Indirect inguinal hernia severe pain, most likely diagnosis is
Ans Testicular Torsion
21. In elderly patient sudden onset of hernia
Ans Direct Hernia is more common than indirect 35 ???

22. 25 years old presents with right upper abdominal


discomfort for last one year with non tender enlarged
liver ultrasound shows multiloculated cysts with floating
membrane inside, diagnosis
Ans Hydatid Cyst Disease

23. Most common indication for splenectomy is


Ans Severe Splenic Trauma

24. Regarding Acute acalculous gall stone


Ans Conservative management only

25. Child with impalpable testis


Ans Laparoscopy

26. Most common cause of acute pancreatitis in


Pakistan
Ans Biliary Calculus

27. A patient with ileal perforation


Ans depends on age, site of perforation and degree of
contamination
43 supply (SURGERY 1) 8. The criteria for acute hospital admission to a burn unit
include:
1. Roll No A. Any burn likely to require fluid resuscitation
2. University name B. Burn caused by hydrofluoric acid
C. Any accidental burn abdomen
3. Shock is the most common and most important cause D. Any burn in middle aged patient
of death in surgical patients. Type of the shock in which E. Larger burn on abdomen
there is reduction in pre load due to mechanical.
obstruction of cardiac filling is called: 9. 18 years female having painless swelling below left
A. Distributive jaws for 6 months O/E a dumb bell shaped swelling in
B. Endocrine left submandibular region that is non tender soft and
C. Obstructive fluctuant. What is probable diagnosis:
D. Cardiogenic A. Cystic degeneration of sub mandibular tumor
E. Hypovolaemic B. Stapne bone cyst
C. Inflammation of sub mandibular gland
4. 30 years female C/O episodes of of pain in fingers on D. Ranula
exposure to cold, fingers turn white and she is unable to E. Plunging Ranula
carry fine hand movements O/E characteristic sequence
of blanching, dusky cyanosis and red engorgement: 10. All nerves are at risk during sub mandibular gland
What is probable diagnosis? excision except:
A. Buerger's disease A. Nerve to mylohyoid muscle
B. Raynaud's syndrome B. Hypoglossal nerve
C. Acrocyanosis C. Lingual nerve
D. Cystic myxomatous degeneration D. Marginal mandibular nerve
E. Raynaud's disease E. Accessory nerve

5. What is the most important investigations in varicose 11. 45 years female C/0 painless lump in Rt breast with
veins: low grade fever and weight loss O/E Pt: is thin built,
A. Varicography having palpable man tender lump with multiple
B. Descending venography discharging sinuses and bilateral axillary lymph nodes
C. Duplex ultrasound are enlarged with matting: What is most probable
D. M.R.I diagnosis:
E. C.T scan A. Mondor’s disease
B. Antibioma
6. A middle aged male having bilateral lymphatic C. Hidradenitis suppurativa
oedema, hydrocele and inguinal lymphadenitis. Blood D. Ca breast
smear shows micro filarae and eosinophilia: What is E. TB mastitis
diagnosis:
A. Filariasis 12. A 50 years female having painless lump in Rt breast
B. Lymphatic malignancy with blood stained nipple discharge. O/E a single 3 x 3
C. Deep vein thrombosis cm non tender, irregular hard, lump is palpable in
D. Superficial thrombo phlebitis central part of Rt breast, ipsilateral axillary lymph nodes
E. Fungal infection palpable and mobile. What investigations will be helpful
to reach final diagnosis:
7. What is common site of bone metastasis: A. Tru cut needle biopsy
A. Iliac bone B. Doppler ultrasound
B. Ribs C. Ultrasound breast
C. Spine D. FNAC
D. Distal humerus E. Mammography
E. Radius
13. Indications for parathyroidectomy include all except: 18. A 30 years old female attends surgical clinic
A. Urinary tract calculi complaining of 3x2 cms firm swelling in front of neck
B. Decreased bone density which moves on deglutination. She is otherwise healthy
C. Age below 50 years and has normal thyroid function tests. Which of the
D. Symptomatic hypocalcemia following is the most important diagnostic test for such
E. High serum Calcium dominant thyroid nodule:
A. MRI neck
14. In the event of blood transfusion reaction, B. Fine needle aspiration cytology
measures to be taken are: C. Ct scan
A. A and D are correct D. Thyroid scan
B. Transfusion may be continued after giving antibiotics E. True cut biopsy
C. Patient will be transfused with fresh blood of different
blood group 19. A 30 years old female presents at surgical clinic,
D. Antihistamine, frusemide and adrenaline are given complaining of flue like illness and low grade fever for
E. Antidiuretic hormone should be given one week and difficulty in swallowing. On examination
tender diffuse swelling in front of neck moving with
15. Fluid containing highest amount of K(Potassium) is: deglutination is noted. Most likely condition accounting
A. Ringer lactate for this presentation is:
B. Plabolyte M A. De quervain thyroiditis.
C. 10% D/water B. Chronic lymphocytic thyroiditis
D. N/Saline C. Reterostemal thyroid extension
E. 5% D/saline D. Graves disease
E. Simple diffuse goiter
16. Regarding different causes of post- operative fever:
A. Antibiotics used always work well, without needing 20. A 22 years old anxious female presents with highly
any other treatment mobile discrete but well circumscribed lump in outer
B. On the 7th post-operative day signifies atelectasis. quadrant of her left Breast. She denies history of trauma
C. UTI is the most common cause or fever. Most likely diagnosis is:
D. May reflect a normal response to surgical trauma A. Fibro adenoma
E. Fever on the first day is because of deep seated B. Fibrocystic disease
abscess C. Duct ectasia
(Verify) D. Breast carcinoma
E. Phyllodes tumour
17. Regarding different types of bone fractures:
A. Colle’s fracture is fracture of distal radius 21. Following massive blood transfusion, which of the
B. Close fractures do not cause break in continuity of following complications may occur?
bone A. Hypocalcaemia
C. Green stick fractures commonly occur in elderly B. Hypercalcaemia
D. Spiral fracture results from direct impact C. Hypoglycaemia
E. In fracture dislocation fractured segments are fixed D. Hypoglycaemia
and do not move relative to each other E. Hyperlipidaemia

22. Regarding Amoebic liver abscess


A. Medical treatment is not effective
B. Quinolones are the drug of choice
C. Repeated aspiration is combined with drug treatment
D. The treatment of choice is always surgery
E. U/S guided aspiration is contraindicated
23. Regarding Hydatid disease, the most common 29. Regarding hypothermia in a patient on operating
affected organ is table:
A. Brain A. Children become hypothermic latter than the adult
B. Kidney B. It is not a significant problem in the patients operated
C. Liver C. None of the above
D. Lungs D. Emergency surgery on a shocked patient does not
E. Spleen predispose to it
E. Long preoperative fasting. predisposes to it
24. Which of the following is appropriate for metabolic
response to starvation 30. Regarding management of traumatic spinal cord
A. High plasma insulin injury:
B. Hepatic gluconeogenesis A. Meteorism should be allowed to occur
C. Increased resting energy expenditure B. Holocaust immobilization increases risk of developing
D. Low plasma Glucagon pressure sores
E. Protein anabolism C. Log-rolling does not have any role in the
25. Regarding burns, the key to monitoring resuscitation management
is D. Passive limb movements should not be allowed
A. Acid-base balance E. Position of patient should be changed after every 6
B. Blood pressure hours
C. Pulse rate
D. Respiratory rate 31. The fast focused abdominal sonogram for trauma
E. Urine output focuses in:
A. All of above
26. In tongue cancer, the site least affected is: B. Hepatic area
A. Posterior portion C. Splenic area
B. Lateral margin D. Pericardial area
C. Dorsal surface E. Pelvic area
D. Tip
E. Ventral surface 32. Regarding CT scan in trauma patient:
A. It has tremendous value and is investigation of
27. Regarding burns, which of the following fluids is choice in unstable patient
appropriate for respiration B. In duodenal injury, oral contrast should be avoided
A. 5% Dextrose water C. All of above
B. 10% Dextrose water D. None of above
C. Fresh frozen plasma E. If rectal injury is suspected, rectal contrast should be
D. Hypertonic saline given
E. Ringer's lactate (Verify)

28. Major cause of impaired wound healing is: 33. Breast conservative surgery (BCS) includes all BUT:
A. Smoking A. TART
B. Diabetes mellitus B. Excisional biopsy only
C. Anemia C. Wide local excision.
D. Local tissue infection D. QUART
E. Malnutrition E. Wide local excision and radiotherapy

34. Indications of surgery for the primary


hyperparathyroidism include:
A. All of the above
B. Symptomatic hypercalcemia
C. Reduced bone density
D. Urinary tract calculi, often recurrent
E. Deteriorating renal function
35. The malignant bone tumours are all but not: 41. The most common fracture of face is that of:
A. Ewing's sarcoma A. Mandible
B. Osteoid osteoma B. Maxilla
C. Osteogenic sarcoma C. Zygoma
D. Chondrosarcoma D. Nasal bone
E. Synovial sarcoma E. Orbital bone

36. What is not true regarding supracondylar humerus 42. The danger area of the face where infection can
fracture in children: spread directly to the cavernous sinus includes:
A. Is usually caused by fall on outstretched hand A. From below the eyes up to the chin.
B. Can cause compartment syndrome B. Area around the lips.
C. Elbow gets swollen C. Area around lips including the lower part of the nose.
D. Can cause nerve injury D. Whole of the face.
E. Intervention can be delayed E. From the maxillary sinus.

37. Regarding sialolithiasis: 43. Neoplasia of the larynx is most common in:
A. The stone is radiolucent on x-rays A. Postcricoid region.
B. Submandibular gland is more prone to develop the B. Lateral wall.
sialolithiasis C. Piriform fossa.
C. None of above D. Aryepiglottic fold.
D. Parotid gland has inheritant property to develop the E. Medial wall.
stones
E. All of above 44. The suture that maintains strength for longest time
is:
38. Pulmonary thrombo-embolism is caused by all A. Chromic catgut
except: B. Dexon
A. Saddle thrombus C. Plain catgut
B. Detachment of pelvic vein thrombus D. PDS
C. Superficial pylephlebitis E. Vicryl
D. Wet gangrene
E. Deep vein thrombosis of lower extremity 45. Trendelenburg position is useful for management in
which type of shock?
39. In surgical profession, a patient has been infected A. Hypovolemic shock.
by HIV positive doctor during which procedure: B. Cardiogenic shock.
A. Dental extraction C. Septic shock.
B. Cardiac transplant D. Neurogenic shock.
C. Abdominal surgery E. Electric shock.
D. Renal transplant
E. Endoscopy 46. A 20 years old woman is taking high doses of
vitamin D to supplement a defect in bones due to
40. Ideal steam sterilization of 121° C with pressure of abnormalities of vitamin D metabolism. The disease she
151 lb/inch should have hold time: is most likely suffering from is called:
A. 0 min A. Osteitis deformans
B. 45 min B. Osteoporosis
C. 15 min C. Osteomalacia
D. 30 min D. Osteitis fibrosa cystica
E. 55 min E. Osteogenesis imperfecta
47. Following surgery or major trauma, which of the (SURGERY 2)
following metabolic changes occur in the body?
A. Decreased adrenal level 1. Roll No
B. Decreased cortisol level 2. University name
C. Hyperglycaemia
D. Hypoglycaemia 3. Characteristic findings on peripheral film of blood
E. Increased plasma albumin after splenectomy is:
A. Decreased number of leukocytes
48. Intermittent claudication is a cramp-like pain felt in B. None of the above
the limb muscles, which of the following statements is C. Increased number of target and helmet RBCs
appropriate? D. Decreased number of platelets
A. The pain felt in the lower limb at rest E. All of the above
B. The pain felt on taking the first step
C. The pain brought on by walking and relieved by 4. Nissen fundoplication:
standing still A. May over treat the GERD and cause dysphagia
D. The foot is often cold and paralyzed B. Main disadvantage of this operation is that it cannot
E. Ulceration and gangrene of toes are usually be performed laparoscopically
associated with even mild cases C. Is always complication free
D. Is indicated in every case of GERD
49. Regarding management of arterial stenosis or E. Is indicated in esophageal atresia
occlusion which of the following is the best among
non-surgical options: 5. Gold standard test for the diagnosis of H.pylori
A. Stop smoking infection is:
B. Transluminal angioplasty and stenting A. Rapid urease test
C. Peripheral vasodilators B. Tissue culture and histology
D. Reduce weight C. Urea breathe test
E. Beta blockers D. Serological tests
E. Fecal antigen test
50. Gynaecomastia is associated with:
A. Multiple endocrine neoplasia type II (MEN-II) 6. Clinical features of carcinoma of stomach include:
B. Klinefelter's syndrome A. Vomiting
C. Nephrotic syndrome B. Metabolic alkalosis with paradoxical aciduria
D. Carcinoid syndrome C. Melana
E. Multiple endocrine neoplasia type I (MEN-I) D. Abdominal mass
E. All of above
51. Which of the following complications of
thyroidectomy may be avoided by prophylaxis is: 7. Regarding the metastatic carcinoma of liver:
A. Recurrent laryngeal nerve palsy A. None of above
B. Hypocalcemia B. Is associated with very high levels of alpha-fete
C. Postoperative haemorrhage protein
D. Wound infection C. All of above
E. Thyroid storm D. Colorectal carcinoma
E. primarily produce this It is essentially single
52. A patient developed an acute blood transfusion
reaction. The immediate step in management is: 8. Indications of CBD exploration while performing
A. Recheck the cross-match report. cholecystectomy include all accept:
B. Rehydrate the patient. A. Dilated CBD on exploration
C. Stop blood transfusion. B. Palpable stones in CBD
D. Catheterize the patient. C. Every elective cholecystectomy as standard practice
E. Give oxygen by mask. D. Recent history of jaundice
E. Preoperative concomitant choledocholithiasis
9. Regarding gastric outlet obstruction in a new born 15. Tuberculous mesenteric lymphadenitis. may present
baby: with:
A. Needs immediate surgery without optimization A. None of above
B. Hypertrophic pyloric stenosis is most common cause B. All of above
C. It is manifested by bilious vomiting C. Abdominal pain
D. Seromyotomy is rarely required to treat the condition D. As a cause of acute intestinal obstruction
E. It is common in female babies E. Mimicking acute appendicitis

10. PCNL is a suitable modality of managing kidney 16. Charcot's triad of symptoms is present in:
stones which are: A. Portal pyemia
A. Stones which can be managed easily by ESWL B. Cholelithiasis
B. More than 1 cm but less than 2cm in size C. Bile duct stone
C. Less than 1cm in size D. Porcelain
D. More than 2.5cm in size E. Gall bladder Hepatitis
E. Any size of stone which is impacted at upper ureter
CSC 17. Which of the following is major cause of acute
pancreatitis, which occurs in 50 to 70% of patients:
11. Drugs for chemical sphincterotomy for the treatment A. Abdominal trauma
of fissure in ano include: B. Gall stones
A. All of above C. Idiopathic
B. Glyceryl Tri Nitrate D. Alcoholism
C. Diltiazem E. Following biliary surgery
D. Botoux injection
E. None of above 18. Which artery is called artery of midgut:
A. Superior mesenteric artery
12. Carcinoma of rectum is present within 4 cm of anal B. Common iliac artery
verge, and the best management option is: C. Inferior mesenteric artery.
A. Abdominoperineal resection of recturn D. Celiac artery
B. Radiotherapy E. Pancreatico duodenal artery
C. Extended left hemicolectomy
D. Anterior resection 19. A 50 years female presents with recurrent
E. Chemotherapy abdominal pain. Vomiting, distention and absolute
constipation, x-ray abdomen reveals air in biliary tree
13. Consequences of acute appendicitis are: and radio opaque shadows on rt side of abdomen. What
A. Perforation is diagnosis:
B. Recurrent appendicitis. A. Trichobezoars
C. Resolution B. Stercolithe
D. Appendicular mass formation C. Phytobezoars
E. All of above D. Gallstone ileus
E. Rt.renal stones with intestinal obstruction
14. Regarding appendicular mass formation:
A. Once the mass has been formed, patient is bound to 20. A 45 years female comes to urologist with swinging
face serious complications pyrexia, left lumber pain and swelling O/E tachycardia,
B. Surgery must not be performed ever if mass is not temp: 100 F, tender swelling and left lumber region, TLC
regressing with antibiotics 12000 per cm, no pus cells or organisms in urine. What
C. Immediate surgery should be performed to remove it is probable diagnosis:
D. It can be managed safely by non- operative A. Perinephric abscess
management B. Psoas abscess
E.The palpable phlegmon consists grossly distended C. Urinary TB
appendix. D. Cold abscess
E. Hydatid cyst kidney
21. The safest site for insertion of a chest drain is in the: 26. A 30 years male patient presented with painful
A. 4th Intercostal space defecation, bleeding per rectum and constipation from
B. 5th Intercostal space two months DRE revealed, a sentinel pile at 6 o clock,
C. 3rd Intercostal space and increased anal tone. What is a possible likely
D. 2nd Intercostal space diagnosis:
E. 6th Intercostal space A. Perianal abscess
B. Fissure in ano
22. Hernia is a protrusion of viscus or part of a viscus C. Perianal haematoma
through an abnormal opening. Which of the following D. Malignant melanoma
has high risk of strangulation E. Ca anal canal
A. Epigastric hernia
B. Direct inguinal hernia 27. What is most common cause of intestinal
C. Paraumbilical hernia obstruction on global scale:
D. Femoral hernia A. Inflammatory
E. Indirect inguinal hernia B. Fecal impaction
C. Adhesions
23. Peritonitis is an acute surgical emergency. Which is D. Obstructed hernia
the most appropriate for this condition: E. Carcinoma
A. Abdominal tenderness and rigidity is present on
palpation 28. What is the most common type of testicular germ
B. Surgery is rarely indicated cell tumor:
C. A Patient usually rolling over the bed A. Yolk sac tumor
D. Pulse and temperature are commonly found normal B. Teratoma
E. Bowel sounds are usually exaggerated C. Embryonal cell tumor
D. Choriocarcinoma
24. Which is the most appropriate for carcinoma of the E. Seminoma.
oesophagus:
A. Barrets oesophagus maybe linked with development 29. Which of the following statements regarding primary
of adenocarcinoma in lower-third sclerosing cholangitis is true
B. Adenocarcinoma usually affects the upper-third A. It has a strong association with ulcerative colitis
C. It is a disease of young adult B. Only the intrahepatic ducts are involved
D. Squamous cell carcinoma usually affects the C. It occurs most commonly in elderly females
lower-third D. It is caused by prior surgical trauma to the bile duct.
E. Tobacco and alcohol are major factors in the E. There is no association with other fibrosis tissue
occurrence of adenocarcinoma disorders

25. Colorectal cancer is the second most common 30. Which of the following condition probably is the most
cause of cancer death in the west. Regarding common cause of recurrent urinary tract infections in
distribution of colorectal cancer, which is the most males:
frequent site of Involvement: A. Kidney stone
A. Transverse colon B. Immunodeficiency disorder
B. Rectum C. Syphilis
C. Descending colon D. Chronic prostatitis
D. Sigmoid colon E. Malakoplakia
E. Ascending colon
31. Diverticulosis occurs most frequently in the:
A. Descending colon
B. Ascending colon
C. Transverse colon
D. Sigmoid colon
E. Caecum
32. The best surgical option for annular pancreas is: 38. A 50 year female presented with a mass in right iliac
A. Partial pencreatectorny fossa, he was diagnosed as carcinoma caecum, which
B. Choledoco-duodenostomy of the following surgical procedure is appropriate for this
C. Partial Gastrectomy condition:
D. Ducdeno-duodenostorny A. Total colectomy
E. Castro jejunostomy B. Right hemicolectomy
C. Hartmann's procedure
33. In Barium swallow, which radiologic finding is found D. Extended right hemicolectomy
in case of Achalasia: E. Paul mikulicz procedure
A. Double Bubble
B. Bird Beak appearance 39. Which gas is used for the creation of
C. Air-fluid level pneumoperitoneum in laparoscopic cholecystectomy:
D. Coffee Bean sign A. Halothane
E. Claw sign B. NO2
C. O2
34. A 22 year old lady presented in OPD with the D. Carbon Monoxide
complain of bleeding per rectum after defecation and E. CO2
something coming out of anus which reduced
spontaneously. Determine the degree of hemorrhoids: 40. The most accurate procedure for the diagnosis of
A. 5th Degree gastritis is:
B. 1st Degree A. X ray abdomen
C. 3rd Degree B. Ultrasound abdomen.
D. 4th Degree C. CT scan abdomen
E. 2nd Degree D. Endoscopy
E. Barium meal
35. Which of the following is a characteristic feature of
intussusception: 41. Good sall's rule is to be applied for which disease:
A. Red currant jelly stool A. Genitourinary fistula
B. Diarrhea B. Pilonidal sinus
C. Passage of Blood and mucus in stool C. Biliary fistula
D. Passage of Mucus in stool D. Fistula in ano
E. Constipation E. Perianal sinus

36. What is the best surgical treatment of achalasia: 42. The most appropriate surgical management of
A. Esophagectomy bleeding ulcer in the antral part of stomach is
B. Nissen procedure A. Wedge resection of the gastric ulcer
C. Hill procedure B. Distal gastrectomy
D. Heller's procedure C. Oversewing the vessel at the ulcer base
E. Stent placement D. Distal gastrectomy along with truncal vagotomy
E. Truncal vagotomy
37. A 30 year farmer by occupation has hydatid cyst
liver, what is the most likely. causative organism: 43. Spontaneous closure in which of the following
A. Echinococcus granulosus congenital abnormalities of the abdominal wall generally
B. Enterobius vermicularis occurs by the age of 4 year:
C. Tinea saginata A. Patent urachus
D. Entamoeba histolytica B. Omphalocele
E. Ascaris lumbricoides C. Umbilical hernia
D. Patent omphalomesenteric duct
E. Gastroschisis
44. Upper Gl- Endoscopy: 50. Tuberculous peritonitis:
A. Is used for treatment of oesophageal varices A. Does not possess specific forms
B. H : pylori should be seen in the stomach by B. Antituberculous drugs are not used
endoscopy C. Is managed surgically
C. Is a gold standard investigation in GERD D. Is always associated with pulmonary tuberculosis
D. Has therapeutic role in colonic polyp E. May arise primarily as abdominal tuberculosis
E. Eradicates H pylori
51. Alvarado stone is important to support a in which
45. Recent advances in CBD stone treatment is: disease;
A. Choledocho dudenostomy A. Cholecystitis
B. MRCP B. Pyelonephritis
C. CBD exploration & removal of stone C. Acute pancreatitis
D. Burhenne technique D. Acute appendicitis
E. ERCP retrieval of stone E. Pelvic inflammatory disease

46. Colorectal carcinoma is the second most common 52. A 30 year female presented with severe. epigastric
malignancy in western countries. Which is the most pain, was treated conservatively after few weeks she
common histological type? noticed pain and epigastric distension, the most likely
A. Squamous cell carcinoma diagnosis is:
B. Malignant melanoma A. Acute pancreatitis
C. Lymphoma B. Cholecystitis
D. Sarcoma C. Acute appendicitis
E. Adenocarcinoma D. Pyelonephritis
E. Pelvic inflammatory disease
47. A volvulus is a twisting or axial rotation of a portion
of a bowel about its mesentery What is the most
common site of its occurrence?
A. Caecum
B. Descending colon
C. Sigmoid colon
D. Transverse colon
E. Ascending colon

48. Post Splenectomy sepsis syndrome:


A. Commonly occurs in children after splenorrhaphy in
traumatic rupture of spleen
B. Caused by viruses
C. The risk is greater in old age
D. Could be avoided by appropriate and timely
immunization and antibiotic prophylaxis
E. The risk is greater after traumatic Splenectomy

49. Regarding cholangiocarcinoma:


A. Squamous squamous cell carcinoma is commonest
type
B. Tends to metastasize widely
C. Gall-Stone is commonest cause
D. Commonly found in females over
E. Has close association with parasitic infestation
42 Preprof 5. A 35 female complaining of painless swelling in front
of neck for a year, no pressure symptoms. O/E discrete
1. A 6 month boy having cystic swelling in the posterior swelling in rt lobe of thyroid. Thyroid scan shows cold
triangle of neck, compressible and transilluminant.What nodule in rt lobe of thyroid. FNAC revealed nodular
is the probable diagnosis? goiter. How will you treat this condition?
A. Branchial cyst A. Wait and watch
B. Cold abscess B. Rt lobectomy and isthmusectomy
C. Carotid body tumor C. Radioiodine therapy
D. Cystic hygroma D. Sub total thyroidectomy
E. Lipoma E. Total thyroidectomy

2. A 42 years female having non healing ulcerated 6. A sinus is a blind tract lined by granulation tissue,
wound over her left leg O/E there is a vertical scar of which of the following is considered to be congenital
previous surgery and irregular ulcer with raised and sinus?
indurated base. What is the most likely diagnosis? A. Actinomycosis
A. Basal cell carcinoma B. Hidradenitis suppurativa
B. Marjolin's ulcer C. Perianal sinus
C. Malignant Melanoma D. Pilonidal sinus
D. Varicose ulcer E. Preauricular sinus
E. Tubercular ulcer
7. A sinus with discharging sulfur granules is a feature
3. A 65 years male C/0 painless swelling below right of?
eye for two years, O/E rounded, non tender, ulcerated A. Actinomycosis
nodule with rolled edges and indurated base. What is B. Hidradenitis suppurativa
the probable diagnosis? C. Osteomyelitis
A. Basal cell carcinoma D. Pilonidal sinus
B. Cutaneous horn E. Tuberculosis
C. Malignant Melanoma
D. Squamous cell carcinoma 8. A keloid scar is defined as excessive scar tissue that
E. Solar keratosis extends beyond the boundaries of the original incision
or wound. Which of the following treatment options have
4. 50 years female having swelling in front of neck for high rates of recurrence?
five years associated with dyspnea, dysphagia for six A. Surgical excision
months, O/E swelling moving with deglutition, veins of B. Intralesional steroid injection
face, neck and anterior chest wall are engorged, C. Laser
cervical lymph nodes are not palpable. What is the most D. Pressure elasticated garments
likely diagnosis? E. Silicone gel sheeting
A. Simple multi nodular goiter
B. Ca thyroid 9. Shock is a systemic state of low perfusion which is
C. Simple diffuse goiter inadequate for normal cellular respiration. Which of the
D. Retrosternal goiter following is the most common form of shock?
E. Primary toxic goiter A. Anaphylactic shock
B. Cardiogenic shock
C. Endocrine shock
D. Hypovolumic shock
E. Septic shock
10. In which form of shock the peripheries will be warm 16. The commonest surgical procedure for the
rather than cold and capillary refill will be brisk, despite treatment of pleomorphic adenoma of the parotid gland
profound shock? is:
A. Cardiogenic shock A. Enucleated of tumor
B. Endocrine shock B. Excision of tumor
C. Hypovolemic shock C. Open surgical biopsy
D. Haemorrhagic shock D. Radical parotidectomy
E. Septic shock E. Superficial parotidectomy

11. In hemorrhagic shock the ideal replacement fluid is? 17. A 60 year old male patient complained of urgency,
A. Blood frequency and dribbling of urine. What is the most likely
B. 10% dextrose water diagnosis?
C. Hartmann's solution A. BPH
D. Normal saline B. Bladder stone
E. Ringer’s lactate C. Cystitis
D. Ca Bladder
12. Compartment syndrome typically occurs in closed E. Urethral Stricture
lower limb injuries. This condition is best managed by?
A. Amputation of the limb 18. A final year MBBS student was asked to read an
B. Early fasciotomy X.ray abdomen, which revealed air under the right dome
C. Late fasciotomy of the diaphragm. What is the likely diagnosis?
D. Intravenous antibiotics and analgesics A. Intestinal obstruction
E. Wait till distal pulses found absent B. Intestinal perforation
C. Uterine perforation
13. A 19 year old woman is admitted unconscious to the D. Ruptured liver abscess
emergency department following a car accident, she E. Intestinal volvulus
has facial lacerations and a displaced tibia fracture.
Which one of the following is the first priority in the 19. What is the best screening test for breast cancer?
management? A. Mammography
A. Insert an intravenous line B. Ultrasound Breast
B. Insert a foley catheter C. Tumor Marker BRCA1 and BRCA2
C. Perform a through neurologic examination D. MRI
D. Maintain her air way E. CT scan
E. Check her blood pressure
20. In Barium swallow, which radiologic finding is
14. Which of the following problems can be encountered diagnostic in case of Achalasia?
in obese patients postoperatively, after prolonged A. Coffee Bean sign
abdominal surgery lasting for 4 hours? B. Bird Beak appearance
A. Deep venous thrombosis C. Claw sign
B. Hypothermia D. Double Bubble
C. Intestinal obstruction E. Air-fluid level
D. Poor wound healing
E. Risk of renal failure 21. A 22 year old lady presented in OPD with the
complain of bleeding per rectum after defecation and
15. The best investigation of choice for assessment of something coming out of anus which reduced
blunt abdominal trauma in unstable patient is: spontaneously. Determine the degree of hemorrhoids?
A. CT scan abdomen A. 1st Degree
B. Diagnostic peritoneal lavage (DPL) B. 2nd Degree
C. MRI C. 3rd Degree
D. Plain X-Ray abdomen D. 4lh Degree
E. U/S abdomen (focused assessment with sonography E. 5th Degree
in trauma FAST)
22. Which of the following malignant conditions of 27. A 30 year female presented with severe epigastric
thyroid be treated prophylactically by total thyroidectomy pain, was treated conservatively, after few weeks she
having strong family history? noticed pain and epigastric distension, the most likely
A. Anaplastic carcinoma diagnosis is?
B. Follicular carcinoma A. Acute gastric dilation
C. Lymphoma B. Intestinal obstruction
D. Medullary carcinoma C. Pseudopancreatic cyst
E. Papillary carcinoma D. Epigastric hernia
E. Hydatid cyst liver
23. Which of the following is a characteristic feature of
intussusceptions ? 28. A 30 year farmer by occupation has hydatid cyst
A. Constipation liver, what is the most likely causative organism?
B. Diarrhea A. Echinococcus granulosus
C. Passage of Mucus in stool B. Entamoeba histolytica
D. Passage of Blood and mucus in stool C. Ascaris lumbricoides
E. Red currant jelly stool D. Tinea saginata
E. Enterobius vermicularis
24. What is the best surgical treatment of achalasia?
A. Hill procedure 29. What is the pathognomonic sign of Grave's
B. Nissen procedure disease?
C. Heller’s procedure A. Exophthalmos
D. Oesophagectomy B. Tremors
E. Stent placement C. Tachycardia
D. Weight loss
25. Which of the following structures may likely be E. Delayed ankle jerk
damaged in parotidectomy?
A. Facial nerve 30. A 50 year female presented with a mass in right iliac
B. Greater auricular nerve fossa, he was diagnosed as carcinoma caecum, which
C. Trigeminal nerve of the following surgical procedure is appropriate for this
D. External laryngeal nerve condition?
E. Marginal mandibular nerve A. Right hemicolectomy
B. Total colectomy
26. In Modified Radical Mastectomy for Ca Breast, the C. Hartmann's procedure
following structures are usually excised? D. Paul mikulicz procedure
A. Whole breast tissue with nipple and areola. E. Extended right hemicolectomy
B. Whole breast tissue with axillary lymph nodes and
pectoralis major muscle. 31. A 30 year old man presented in OPD, which the
C. Whole breast tissue with axillary lymph nodes, complain of painful micturition, followed by few drops of
pectoralis major and minor blood from urethra, which of the following pathology is
muscles. suspected?
D. Whole breast tissue, nipple and areola with axillary A. Benign prostatic hyperplasia
lymph nodes . B. Urinary bladder stone
E. Whole breast tissue with axillary lymph nodes and C. Urinary bladder carcinoma
serratus anterior muscle. D. Urinary bladder tuberculosis
E. Ureteric stone
32. Which gas is used for the creation of 37. In colonic cancer, double contrast barium enema
pneumoperitoneum in laparoscopic cholecystectomy? shows a constant irregular filling defect is described as?
A. CO2 A. Apple core appearance
B. O2 B. Bird beak appearance
C. NO2 C. Claw sign
D. Halothane D. Double bubble sign
E. Carbon Monoxide E. Rat tail appearance

33. The most accurate procedure for the diagnosis of 38. Hydronephrosis is an aseptic dilatation of the kidney
gastritis is? caused by obstruction. The most common cause of
A. barium meal bilateral hydronephrosis in old age is usually occurs in:
B. CT scan abdomen A. Benign prostate hyperplasia (BPH)
C. Endoscopy B. Postoperative bladder neck scarring
D. ultrasound abdomen C. Posterior urethral valves
E. x ray abdomen D. Urethral atresia
E. Urethral stricture
34. Good Sall's rule is a clinical assessment for certain
diseases. In which of the following conditions this rule is 39. Renal calculi are the most frequent cause of renal
applied ? pain in our country. A staghorn calculus is smooth
A. Biliary fistula walled and present in renal pelvis. It is usually
B. Fistula in ano composed of:
C. Genitourinary fistula A. Blood pigment
D. Perianal sinus B. calcium oxalate monohydrate
E. Pilonidal sinus C. Calcium, ammonium and magnesium phosphate
(Trippie phosphate)
35. Carcinoma of the caecum or ascending colon is D. Cystine
most commonly treated when resectable by surgery. E. Uric acid
Which of the following surgical procedure is suitable for
this condition? 40. The malignant neoplasm of the kidney in adult is
A. Rt. hemicolectomy adenocarcinoma (Grawitz Tumor). It is the most
B. Extended right hemicolectomy common neoplasm of the kidney in men (75%
C. Limited right hemicolectomy incidence). Which organ is involved early by its distant
D. Hartman’s procedure metastasis:
E. Paul-Mikuliz procedure A. Bones
B. Brain
36. A 50 year old female presented with altered bowel C. Liver
habits since 3 years. Sigmoidoscopy and biopsy D. Lungs
revealed colorectal carcinoma at rectosigmoid junction. E. Vertebrae
What is the most frequent organ presumed to be
involved in distant metastasis? 41. The most common cause of acute retention of urine
A. Brain in male is:
B. Kidney A. Acute urethritis
C. liver B. Acute prostatitis
D. Lungs C. Bladder outlet obstruction
E. Ovary D. Phimosis
E. Urethral stricture
42. The best surgical approach to the prostate for BPH 46. 70 year old male who is known case of peptic ulcer
is: disease for past 30 years, presented with H/O anorexia,
A. Perineal prostatectomy progressive vomiting, with significant weight loss. He
B. Retropubic prostatectomy looks grossly anaemic. Abdominal examination reveals
C. Transvesical prostatectomy a mass in epigastric region. Best investigation to
D. Transperitoneal Prostatectomy confirm the diagnosis will be:
E. Transurethral prostatectomy A. Barium meal
B. CT scan abdomen
43. A 60 years old lady having multi nodular swelling C. Endoluminal ultrasound
infront of neck since 20 years. Her swelling remarkably D. Endoscopy and biopsy
increased in size during last 2 months , associated with E. PET scan
weight loss and reduced appetite. What is most
probable diagnosis? 47. A 50 years male patient underwent surgery for
A. Grave's disease common bile duct stone. Choledocholithotomy was
B. Lymphoma performed and T-tube placed. On 10th postoperative
C. Thyroid carcinoma day T-tube Cholangiogram performed that shows free
D. Simple multi nodular goiter flow of dye into the duodenum. What will be your next
E. Toxic multi nodular goiter step of management?
A. Block T-tubc and observe for few days
44.A 35 years old male patient presented with painful, B. Keep T-tube in place for 1 week more
irreducible swelling in right inguinal region for 2 days C. Perform CT scan followed by T-tube removal
associated with acute intestinal obstruction. Surgery D. Remove T-tube
performed in emergency and after exploration of the E. Repeat T-tube cholangiogram
swelling, gut was found bluish-black in color. Most likely
diagnosis is: 48. Intralesional excision followed by radiotherapy is
A. Incarcerated hernia usually the treatment of choice for which of the following
B. Ischemic bowel disease conditions?
C. Obstructed inguinal hernia A. Carcinoma in situ
D. Obstructed femoral hernia B. Keloid
E. Strangulated inguinal hernia C. Lip ulcer
D. Melanoma
45. A 30 years old lady complains about painless E. Papilloma
bleeding per rectum for past one month. On
examination she is severely anaemic with third degree 49. A young patient admitted in ward with history of pain
haemorrhoids. in Rt iliac fossa for 4 days. On examination a tender
What will be the first step in management in this lady: mass is palpable in right iliac fossa associated with
A. Band ligation temperature 100 F. U/S suggests appendicular mass.
B. Correction of anaemia followed by hemorrhoidectomy What is the management option?
C. Discharge and follow up A. Appendicectomy on next available OT list
D. Emergency surgery B. Colonoscopy and biopsy from mass
E. T Bandage And bedrest C. Discharge and advise interval appendicectomy
D. Laparotomy and Rt hemicolectomy
E. Admit and start Ochsner Sherren Regimen
50. A 45 years female presented with a large mass in
her Rt breast, diagnosed after tru cut biopsy as
carcinoma breast. The main aim of neoadjuvant therapy
in this case will be:
A. To down stage the disease
B. To cure the disease
C. To prevent metastasis
D. To reduce postoperative complications
E. To improve patient compliance
42 Prof (SURGERY 1) 7. Which of the following complications of thyroidectomy
may be avoided by prophylaxis is:
1. Which of the following is most frequent site of Bed A. Recurrent laryngeal nerve palsy
sores (decubitus ulcers) B. Hypocalcemia
A. Sacrum C. Postoperative haemorrhage
B. Greater trochanter D. Wound infection
C. Ischium E. Thyroid storm
D. Heal
E. Occiput 8. What the most common organ injured in blunt
abdominal trauma:
2. Regarding Buerger's disease A. Diaphragm
A. Thromboangiitis obliterans B. Liver
B. Thrombophlebitis C. Mesentery
C. Disease of large veins D. Spleen
D. More common in middle aged females E. Stomach
E. Common in diabetics
9. 20 years old boy presented to the emergency
3. Varicose veins department, he was hit against the motorcycle handle.
A. Are the tortuous dilated pampiniform plexus of veins He was in shock and urgent Investigation of choice for
B. Commonest site is scrotum blunt trauma at in unstable patient is:
C. Pregnancy & pelvic tumors are common factors A. CT scan
D. Stockings are contraindicated in old patients with B. Diagnostic peritoneal lavage
varicose veins C. MRI
E. Trendelenburg’s operation has no role D. USG
E. X-ray abdomen
4. Common cause of oral cancer
A. Tobacco smoking 10. The most common cause of increased serum
B. Bettie nut chewing calcium levels in hospitalized patients is:
C. Alcohol A. Primary hyperparathyroidism.
D. Dental ulcer B. Immobilization.
E- Gingivitis C. Malignancy.
D. Vitamin D excess.
5. The diagnosis of primary hyperparathyroidism is most E. Myeloma.
strongly suggest
A. Increased serum acid phosphatase 11. Concealed haemorrhage may become revealed in:
B. Increased serum calcium A. Ruptured spleen
C. Decreased urinary calcium B. Ruptured liver
D. Increased alkaline phosphatase C. Ruptured Kidney
E. Increased parathyroid hormone level D. Ruptured ectopic gestation/pregnancy
E. Cerebral haemorrhage
6. A young boy of 23 year age sustained head injury by
road traffic accident by bike he should be evaluated by
A. X-ray skull
B. Lumbar puncture
C. Observation by Glasgow coma scale
D. C.T scan brain
E. Carotid angiography
12. The most likely life threatening complication of 23. A girl present tender swelling periareolar
simple multi nodular goiter is: erythematosus
A. Secondary thyrotoxicosis Ans Acute mastitis
B. Carcinoma
C. Acute respiratory obstruction 24. A female lady prevents with 2x2cm cystic mass ,
D. Change of voice wich No lymph node palpable How will your manage
E. Cosmetic disfigurement Ans Mammograpy

13. Which one of the following methods of sterilization Is 25. Regarding mimmel case Surgery
Ideal for surgical instruments Ans Cosmetic
A. Boiling
B. Spirit 26. Regarding abscess
C. Pyodine pollution Ans Incision and drainage
D. Sterilization by autoclave
E. Hot air oven 27. A patient with heavy trauma, he is unconscious and
unable to take nutrition orally, favorable prolong time.
14. Regarding echinococcus granulosus rout for Nutrition
Ans Rupture of cyst cause anaphylactic shock Ans Parenteral

15. Late Sign of Compartment Syndrome 28. IV fluid for Hypovolemic Shock
Ans Absence of Distal pulse Ans IV Ringers Lactate

16. Necrotizing fasciitis 29. Delayed primary wound healing done in


Ans Its polymicrobial and synergetic infection Ans Perforated Appendix

17. Superior vena cava syndrome Commonest Cause 30. A boy present with 3 cm solitary Nodule
Ans Ca lung investigation of choice
Ans FNAC
18. Regarding Frey’s syndrome
Ans Starch iodine test 31. Post operative fever most likely
Ans ??
19. Which of following is not cause of acute suppurative
sialadenitis 32. Intertrochanteric fracture of femur in elderly
Ans Mumps Ans Reduction and internal fixation

20. Following massive blood transfusion, which of the 33. Regarding surgical drain
following complications may occur? Ans Source of infection
A. Hypocalcaemia
B. Hypercalcaemia 34. Risk of breast cancer
C. Hypoglycaemia Ans Increase with age
D. Hypoglycaemia
E. Hyperlipidaemia 35. Regarding burns, the key to monitoring resuscitation
is
21. 20 year lady presents with unilateral Bleeding A. Acid-base balance
disease B. Blood pressure
Ans Ductal papilloma C. Pulse rate
D. Respiratory rate
22. Reserving primary survey E. Urine output
Ans Maintain airway asses Breathing and circulation
(SURGERY 2) 7. The diagnostic feature of urethral injury is
A. Palpable urinary bladder
1. A 45 years female presented with severe epigastric B. Urinary Incontinence
pain, radiating to the back since 24 hours. She has past C. Urethral bleeding
H/O dyspepsia. Her serum amylase level 2200 U/lit. D. Perianal haematoma
She is diagnosed with acute pancreatitis Which of the E. Acute urinary retention
following is the most common of acute pancreatitis in
Asian Countries? 8. upto Para-reetel Stage Colorectal Carcinoma is
A. Alcohol Ans Stage B
B. Autoimmune
C. Gallstones 9. The most frequent site for blood borne metastases in
D. Steroids colorectal cancer is
E. Trauma A. Lungs
B. Liver
2. Tuberculous peritonitis C. Adrenals
Ans Abdominal pain, sweeting, weight loss are D. Bones
distinguish features E. Brain

3. A 10 years old primary school boy presents with 10. A volvulus is a twisting or axial rotation of a portion
attacks of central abdominal pain with vomiting. On of the bowel about its mesentery. What is the most
examination his temperature is 38°C. His right iliac common site of its occurrence?
fossa is tender, but tenderness moves medially when he A. Ascending colon.
lies on left side. The diagnosis is: B. Caecum.
A. Acute nonspecific mesenteric lymphadenitis. C. Descending colon.
B. Tuberculous mesenteric lymphadenitis. D. Sigmoid colon.
C. Henoch-Schonlein purpura. E. Transverse colon.
D. Meckel’s diverticulitis.
E. Acute appendicitis. 11. 25 years male brought to casualty with H/O blunt
abdominal trauma, C/O severe chest pain O/E pulse
4. Management of acute Intestinal obstruction include: 104 per minute, chest x-ray revealed air fluid level in the
A. Immediate surgical Intervention always. left side of chest. What is probable diagnosis
B. Never a surgical intervention. A. Diaphragmatic hernia
C. Resuscitation followed by surgery if un-relieved B. Empyema thoracic
D. No need of performing BUN and creatinine levels. C. Lung abscess
E. Resuscitation followed by kleen enema. D. Pleural effusion
E. Pneumothorax
5. A 45 years female C/O pain in rt upper abdomen,
flatulent dyspepsia, fever and jaundice. O/E pulse 96 b 12. Which of the following is a characteristic feature of
/min and tenderness in rt hypochondrium. ultrasound intussusceptions ?
shows multiple gall stones and dilated CBD. What is A. Constipation
diagnosis: B. Diarrhea
A. Cholangitis C. Passage of Mucus in stool
B. Cholecystitis D. Passage of Blood and mucus in stool
C. Hepatitis E. Red currant jelly stool
D. Pancreatitis
E. Portal pyemia 13. Staging Of colon carcinoma is best done by:
A Digital rectal examination (DRE)
6. Cholangitis result from B. Sigmoidoscopy
Ans CBD calculus C. Colonoscopy
D. C.T scan pelvis
E. Endoluminal ultrasound
14. A patient presents with severe epigastric pain, 20. Regarding anatomy and physiology of anal canal
radiates to back, O/E Show Parauntoliced petechial which one is true except
rash more Ans dentate line divide anal canal into right and left
likely diagnosis halves
Ans Acute pancreatitis
21. Upper Gl- Endoscopy:
15. A middle aged obese male complaining of A. Is used for treatment of oesophageal varices
retrosternal burning, epigastric pain radiating to back B. H : pylori should be seen in the stomach by
and regurgitation. Which of the fowling is gold standard endoscopy
to diagnose the problem. C. Is a gold standard investigation in GERD
A. Barium swallow D. Has therapeutic role in colonic polyp
B. Chest X-ray E. Eradicates H pylori
C. Manometry
D. 24 hour ph monitoring 22. Hernia is a protrusion of a viscus or part of a viscus
E. Upper G-I endoscopy through an abnormal opening. Which of the following
has a high risk of strangulation?
16. The best conservative treatment of anal fissure is A. Direct inguinal hernia
A. High fiber diet B. Epigastric hernia
B. Stool softness C. Femoral hernia
C. 5% xylocaine cream tropically D. Indirect inguinal hernia
D. Chemical sphincterotomy E. Paraumbilical hernia
E. lateral internal sphincterotomy
23. A middle-aged patient complains of severe
17. Intestinal tuberculosis can affect any part of the abdominal pain with a past history of dyspepsia. His
gastrointestinal tract from mouth to anus. The sites x-ray chest demonstrates free air under the right dome
affected most often are: of the diaphragm. Which of the following is the most
A. Duodenum. likely possibility?
B. Ileum. A. Acute cholecystitis
C. Jejunum. B. Acute pancreatitis
D. Rectum. C. Perforated appendicitis
E. Sigmoid colon. D. Perforated duodenal ulcer
E. Ruptured liver abscess
18. Post Splenectomy sepsis syndrome:
A. Commonly occurs in children after splenorrhaphy in 24. Hydatid cyst commonly caused by
traumatic rupture of spleen A. Entamoeba histolytica
B. Caused by viruses B. Ascaris lumbricoides
C. The risk is greater in old age C. Enterobius vermicularis
D. Could be avoided by appropriate and timely D. Liver fluke
immunization and antibiotic prophylaxis E. Echinococcus granulosus
E. The risk is greater after traumatic Splenectomy
25. Hemorrhoids:
19. 60 year male presented to emergency Department A. Are more common in obesity & pregnancy
with painful irreducible swelling in Rt inguino scrotal B. Are the dilated rectal cushions
region associated with vomiting and constipation since C. Classically they occur in 3, 6 & 12 0* clock positions
five day,O/E swelling is tender and cough impulse D. 1’ degree hemorrhoids protrude but not blood
negative. What is diagnosis? E. Presented e black coloured stool
A. Acute intestinal obstruction
B. Epididymo Orchitis 26. Charcot triad
C. Hydrocele Ans Acute cholangitis
D. Strangulated inguinal hernia
E. Torsion of testis
27. Commonest site for colorectal cancer. 35. The position of the base of the appendix is constant,
A. Caecum while the tip is variable. What is the most common
B. Descending colon position of the appendix
C. Transverse colon A. Paracaecal
D. Recto sigmoid junction B. Pelvic
E. Rectum C. Preileal
D. Postileal
28. Gastric Carcinoma Metastasis to ovary E. Retrocaecal
Ans Krukenberg tumor

29. Regarding Acute pancreatitis


Ans characterized by abdominal pain rigidity tenderness

30. A 45 years female comes to urologist with swinging


pyrexia, left lumber pain and swelling O/E tachycardia,
temp: 100 F, tender swelling and left lumber region, TLC
12000 per cm, no pus cells or organisms in urine. What
is probable diagnosis:
A. Perinephric abscess
B. Psoas abscess
C. Urinary TB
D. Cold abscess
E. Hydatid cyst kidney

31. Alvarado stone is important to support a in which


disease.
A. Acute appendicitis
B. Cholecystitis
C. Acute pancreatitis
D. Pyelonephritis
E. Pelvic inflammatory disease

32. Which of following pathology is premalignant


condition, appears as calcified gallbladder on plain x-ray
and it is an indication for cholecystectomy
A. Cholesterosis
B. Empyema gallbladder
C. Porcelain gallbladder
D. Pharyngian cap
E. Mucocele

33. Which is the most appropriate for gastric ulcer


A. Chronic gastric ulcer is associated with malignancy
B. Gastric ulcer is more common than duodenal ulcer
C. It is more prevalent in high socioeconomic groups
D. It is more common in females
E. It is more common in the west

34. Regarding gastric ulcer


Ans Managed by medications endoscopy and surgery
41 Preprof 7. Indications of surgery for primary
hyperparathyroidism include:
1. Regarding major wound infection: A. Urinary tract calculi, often recurrent.
A. Source of infection is always endogenous. B. Reduced bone density.
B. Leads to multiple organ dysfunction syndrome. C. Deteriorating renal function.
C. Discharges significant quantity of pus. D. Symptomatic hypercalcemia.
D. Systemic signs are marked. E. All of the above.
E. Patient's return to home is not delayed.
(Verify) 8. Malunion of a fracture is defined as:
A. Union of fracture in a position of deformity.
2. The use of prophylactic antibiotics are usually B. Non-union.
recommended: C. Delayed union.
A. Two hours before surgery. D. Fracture through a union.
B. One hour before surgery. E. Healing of a pathological fracture.
C. At the time of induction of anesthesia.
D. One hour after surgery. 9. A patient developed an acute blood transfusion
E. Two hours after surgery. reaction. The immediate step in management is:
A. Recheck the cross-match report.
3. The most accurate method of investigation to access B. Rehydrate the patient.
spinal injury is: C. Stop blood transfusion.
A. CT scan. D. Catheterize the patient.
B. MRI spine. E. Give oxygen by mask.
C. EMG (electromyography).
D. EEG (electroencephalography). 10. Which statement is best appropriate regarding
E. X-ray spines. keloid?
A. It is formed and regressed spontaneously.
4. Extradural hematoma is a neurosurgical emergency. B. It is best managed by surgical excision.
Typically, this condition occurs due to damage to the: C. It is best managed by wait and watch.
A. Anterior cerebral artery. D. Keloid tissue contains less collagen than the scar.
B. Cortical vessels. E. A keloid extends beyond the boundaries of the
C. Middle meningeal artery. original scar.
D. Posterior cerebral artery.
E. Posterior cerebellar artery. 11. Split skin grafting is one of the methods to cover the
raw area. Which statement is most appropriate?
5. Regarding metastasis of breast cancer: A. Epidermis and dermis are taken for grafting.
A. It occurs by the bloodstream most commonly. B. Can be done over joints and bare bones.
B. It occurs to the axillary lymph nodes commonly. C. Should be avoided in presence of infection.
C. Occurs to the lungs commonly. D. Donor area is usually the lower leg.
D. Occurs as skin nodules commonly. E. It is suitable over large and deep skin defects.
E. None of the above. (Verify)

6. Regarding clinically discrete thyroid swelling, what is 12. What is the percentage of burns in a patient who
most appropriate? suffers burns over the whole left lower limb
A. It is called dominant if present in an otherwise (circumferential) plus anterior trunk?
impalpable gland. A. 18%.
B. It is called solitary if present in an enlarged gland. B. 27%.
C. It is associated with increased risk of malignancy if it C. 36%.
does not take radioiodine. D. 45%.
D. FNAC has no role in its management. E. 54%.
E. Thyroid scan should be avoided.
13. "CYSTIC HYGROMA" is: 19. "Bones, stones, groans, and moans" relate to:
A. Lymphangiectasia. A. Hyperparathyroidism.
B. Cavernous hemangioma. B. Hypoparathyroidism.
C. Sebaceous cyst. C. Hyperthyroidism.
D. Dermoid cyst. D. Hypothyroidism.
E. Hemangioma. E. Pheochromocytoma.

14. Usual fluid requirement in a burnt patient per kg 20. Trendelenburg position is useful for management in
percent burn during the first 24 hours is: which type of shock?
A. 3-4 mL. A. Hypovolemic shock.
B. 5-6 mL. B. Cardiogenic shock.
C. 7-8 mL. C. Septic shock.
D. >10 mL. D. Neurogenic shock.
E. >20 mL. E. Electric shock.

15. Cleft lip is ideally repaired at: 21. A 25-year-old male suffered blunt trauma to the
A. Soon after birth. chest. In the ER, he presented with shortness of breath
B. 6 weeks - 12 weeks. and cyanosis. On examination, no breath sounds were
C. 10 weeks - 24 weeks. heard on the right side, and the percussion note was
D. 24 weeks - 36 weeks. resonant. The most likely diagnosis is:
E. 1year A. Spontaneous pneumothorax.
B. Cardiac tamponade.
16. The danger area of the face where infection can C. Cardiac contusion.
spread directly to the cavernous sinus includes: D. Tension pneumothorax.
A. From below the eyes up to the chin. E. Pericardial effusion.
B. Area around the lips.
C. Area around lips including the lower part of the nose. 22. Which one of the following is a metabolic feature of
D. Whole of the face. starvation?
E. From the maxillary sinus. A. High plasma insulin concentration.
B. Low plasma glucagon concentration.
17. Neoplasia of the larynx is most common in: C. Hepatic glycogenolysis.
A. Postcricoid region. D. Adaptive ketoacidosis.
B. Lateral wall. E. Increase in energy requirement.
C. Piriform fossa. (In key its C)
D. Aryepiglottic fold.
E. Medial wall. 23. Which of the following is an indication of parenteral
nutrition?
18. A 65-year-old female complains of regurgitation of A. Proximal intestinal fistula.
undigested food a few hours after meals, progressive B. Vesico-vaginal fistula.
difficulty in swallowing, weight loss, and swelling in the C. Rectovaginal fistula.
neck. What is the most likely diagnosis? D. Fistula in ano.
A. Achalasia. E. Thyroglossal fistula.
B. Carcinoma esophagus.
C. Diffuse esophageal spasm. 24. Regarding medullary carcinoma of the thyroid:
D. Pharyngeal pouch. A. Tumor takes up radioactive iodine.
E. Plummer-Vinson syndrome. B. No association with other endocrine abnormalities.
C. No screening test is available.
D. Prophylactic surgery is recommended in RET
gene-positive children.
E. It is associated with low calcitonin levels in the blood.
25. Triple assessment of a breast lump includes all 31. True statement regarding carcinoma of the
except: esophagus:
A. Clinical examination. A. Squamous cell carcinoma usually affects the lower
B. Fine needle aspiration cytology (FNAC). 2/3.
C. Mammogram. B. Adenocarcinoma usually appears in the upper 1/3.
D. Trucut biopsy. C. The incidence of adenocarcinoma is increasing.
E. Chest X-ray. D. Dysphagia is the most common early presenting
symptom.
26. The most common tumor of the small bowel is: E. Tobacco and alcohol are common etiological factors
A. Carcinoma. for adenocarcinoma.
B. Adenoma.
C. Fibroma. 32. Tumor marker in hepatocellular carcinoma is:
D. Hemangioma. A. Carcinoembryonic antigen (CEA).
E. Lipoma. B. Alkaline phosphatase.
C. Alpha-fetoprotein.
27. A 25-year-old male presented to the emergency D. B-HCG.
room after swallowing two open safety pins six hours E. PSA.
ago. X-rays show the pins located in the small intestine.
The most appropriate management at this point would 33. The most common postoperative complication of
be: appendicitis is:
A. Administration of a broad-spectrum antibiotic A. Intra-abdominal abscess.
intravenously. B. Ileus.
B. Administration of 250 mL of magnesium citrate to C. Wound infection.
induce catharsis and increase the rapidity of passage. D. Fecal fistula.
C. Follow-up with serial X-rays and abdominal E. Venous thromboembolism.
examinations.
D. Immediate surgery. 34. Perianal fistula is best demonstrated by:
E. Endoscopy. A. CT scan.
(Verify) B. Plain X-ray.
C. Tomography.
28. Which of the following increases the risk of D. Ultrasound.
developing colon cancer? E. MRI.
A. Diet high in saturated fat.
B. Diet high in sodium. 35. Which of the following hernias follow the path of the
C. Diet high in selenium. spermatic cord within the cremaster muscle?
D. Diet high in calcium. A. Femoral.
E. Diet high in magnesium. B. Direct inguinal.
C. Indirect inguinal.
29. Which of the following is most commonly confused D. Spigelian.
with appendicitis in children? E. Interparietal.
A. Meckel's diverticulitis.
B. Mesenteric adenitis. 36. Laparoscopic cholecystectomy is indicated for
C. Pelvic inflammatory disease. symptomatic gallstones in which of the following
D. Acute gastroenteritis. conditions?
E. Pyelonephritis. (Verify) A. Cirrhosis.
B. Prior upper abdominal surgery.
30. The treatment of choice for splenic abscess is: C. Suspected carcinoma of the gallbladder.
A. Antibiotics alone. D. Morbid obesity.
B. Percutaneous drainage. E. Coagulopathy.
C. Partial splenectomy.
D. Total splenectomy.
E. Wait and watch. (Verify)
37. Which of the following is the MOST common cause 43. The most precise diagnostic screening procedure
of painful rectal bleeding? for differentiating benign from malignant thyroid nodules
A. Internal hemorrhoids. is:
B. External hemorrhoids. A. Fine needle aspiration cytology (FNAC).
C. Diverticulitis. B. Thyroid scan.
D. Anal fissure. C. Thyroid ultrasound.
E. Rectal foreign body. D. Thyroid antibodies.
E. Thyroid profile.
38. A 55-year-old man who is extremely obese reports
weakness, sweating, tachycardia, confusion, and 44. The most common cause of increased serum
headache whenever he fasts for more than a few hours. calcium levels in hospitalized patients is:
He has prompt relief of symptoms when he eats. These A. Primary hyperparathyroidism.
symptoms are most suggestive of which of the following B. Immobilization.
disorders? C. Malignancy.
A. Diabetes mellitus. D. Vitamin D excess.
B. Insulinoma. E. Myeloma.
C. Zollinger-Ellison syndrome.
D. Carcinoid syndrome. 45. Intestinal tuberculosis can affect any part of the
E. Multiple endocrine neoplasia, type II. gastrointestinal tract from mouth to anus. The sites
affected most often are:
39. What is the most common site of breast cancer? A. Duodenum.
A. Lower outer quadrant. B. Ileum.
B. Lower inner quadrant. C. Jejunum.
C. Upper outer quadrant. D. Rectum.
D. Upper inner quadrant. E. Sigmoid colon.
E. Nipple and areola.
46. A volvulus is a twisting or axial rotation of a portion
40. The most common testicular tumor in prepubertal of the bowel about its mesentery. What is the most
adults is: common site of its occurrence?
A. Yolk sac tumor. A. Ascending colon.
B. Embryonal cell carcinoma. B. Caecum.
C. Seminoma. C. Descending colon.
D. Teratoma. D. Sigmoid colon.
E. Choriocarcinoma. E. Transverse colon.

41. On clinical examination of the breast, which of the 47. Which of the following is the best surgical approach
following is the most appropriate finding in breast for performing prostatic surgery?
cancer? A. Retropubic.
A. Breast tenderness. B. Transperineal.
B. Nipple discharge. C. Transperitoneal.
C. Breast lump. D. Transvesical.
D. Breast infection. E. Transurethral.
E. Hyperpigmentation of the nipple and areola.
48. Which of the following is the most appropriate
42. The 5-year survival rate for patients with esophageal regarding hydrocele?
adenocarcinoma is: A. It is dilation of veins within the spermatic cord.
A. 5%. B. Primary hydrocele is usually symptomatic.
B. 20%. C. Surgery is the mainstay of treatment.
C. 30%. D. Testicular malignancy is a common cause of
D. 50%. hydrocele.
E. 80%. E. It usually does not transilluminate.
(Verify)
49. The most common clinical feature of carcinoma of
the head of the pancreas is:
A. Epigastric pain.
B. Anorexia and malaise.
C. Thrombophlebitis.
D. Obstructive jaundice.
E. Mass in the abdomen.

50. Careful clinical examination will be diagnostic in the


vast majority of patients complaining of anal symptoms.
Which is the most common position for examination of
anal pathology?
A. Jack-knife position.
B. Knee-elbow position.
C. Lithotomy position.
D. Left lateral (Sims) position.
E. Right lateral position.
41 Prof (SURGERY 1) 9. The best investigation of choice for assessment of
blunt abdominal trauma in unstable patient is:
1. In Amoebiasis the majority of abscesses occur in A. CT scan abdomen
which of the following organs? B. Diagnostic peritoneal lavage (DPL)
A. Spleen C. MRI
B. Right lobe of liver D. Plain X-Ray abdomen
C. Lungs E. U/S abdomen (focused assessment with sonography
D. Sub diaphragmatic in trauma FAST)
E. Left lobe of liver
10. A patient brought to casualty with an obviously short
2. Which one of the following cases is considered a externally rotated leg after trauma, the possibility could
clean-contaminated wound? be:
A. Open cholecystectomy for cholelithiasis A. Intertrochanteric fracture
B. Herniorrhaphy with mesh repair B. Fracture head of femur
C. Lumpectomy with axillary node dissection C. Fracture neck of femur
D. Appendectomy with walled-off abscess D. Fracture shaft of femur
E. Gunshot wound to the abdomen with injuries to the E. Fracture lower end of tibia
small bowel and sigmoid colon
11. Deep vein thrombosis is commonly diagnosed by:
3. Subconjunctival bleed with no posterior border A. Ascending venography
indicates fracture of which bone? B. Duplex ultrasound image
A. Maxilla C. D-Dimer test
B. Mandible D. MRI
C. Zygoma E. Trendelenburg test
D. Nasal
E. Skull 12. Varicose surgery ind
Ans Lipodermo + Ulcer
4. Snuff box
Ans Scaphoid bone 13. The most precise diagnostic screening procedure
for differentiating benign thyroid nodules from malignant
5???? ones is:
A. Fine-needle-aspiration cytology FNAC
6. Regarding the evaluation of spinal cord disorders, B. Thyroid ultrasonography
which of the following clinical findings are characteristics C. Thyroid scintiscan
of upper motor neuron lesions? D. Thyroid antibodies
A. Decreased tone – flaccid E. Thyroid profile
B. Downgoing plantar response
C. Hyperreflexia 14. For burn patients, which of the following fluids is
D. Hyporeflexia appropriate for resuscitation?
E. Sensory loss A. 5% Dextrose water
B. 10% Dextrose water
7. Dehydration C. Fresh frozen plasma
Ans skin turgor D. Hypertonic saline
E. Ringer’s lactate
8. Pneumothorax + Collapse
Ans Chest incubation 15. Usual fluid requirement in a burnt patient per kg
percent burn during the first 24 hours is:
A. 3-4 mL.
B. 5-6 mL.
C. 7-8 mL.
D. >10 mL.
E. >20 mL.
16. Regarding pre-operative preparation before giving 21. Which of the following is the most common variety
general anaesthesia, patients are advised not to take of thyroid cancer?
solids: A. Papillary carcinoma
A. Within 2 hours B. Follicular carcinoma
B. Within 6 hours C. Medullary carcinoma
C. Within 12 hours D. Anaplastic carcinoma
D. Within 24 hours E. Lymphoma
E. Within 48 hours
22. In carcinoma of the breast, the most frequent site of
17. Regarding total energy requirement of a stable skeletal metastasis is:
patient, which of the following is approximately needed? A. Femur
A. 5-10 Kcal/kg per day B. Lumbar vertebrae
B. 10-15 Kcal/kg per day C. Ribs
C. 15-20 Kcal/kg per day D. Skull
D. 20-30 Kcal/kg per day E. Pelvis
E. 30-40 Kcal/kg per day
23. While discussing with your trainees about
18. A 30 years old laboratory technician suffering from management of patients presenting with breast lumps,
ulcerative colitis develops listlessness, slurred speech, you attempt to test the baseline knowledge of trainees
muscular hypotonia, depressed reflexes and abdominal in this context, by asking about term "Triple
distension. His ECG shows prolonged QT interval, Assessment". What is the most accurate explanation of
depressed ST segment and flattering of T wave. The triple assessment?
parenteral fluid most appropriate for this patient would A. Clinical assessment, U/S breast & mammography
be: B. Clinical assessment, U/S breast & CT scan chest
A. Dextrose 5°k. C Clinical assessment, U/S breast & tissue sampling
B. Normal saline 0.9%. taken for either cytological or histological analysis
C. Dextrose with saline. D. U/S breast, CT scan chest & FNAC
D. Saline with KCI. E. U/S breast, MRI & Trucut needle biopsy
E. Ringer's.
24. Regarding management of arterial stenosis or
19. A 20 years old man receives multiple blood occlusion which of the following is the best among
transfusions for abdominal gunshot wound. He non-surgical options:
complains of numbness around his mouth and displays A. Stop smoking
carpopedal spasm and the positive Chvostek sign. The B. Transluminal angioplasty and stenting
treatment required is: C. Peripheral vasodilators
A. Intravenous bicarbonate D. Reduce weight
B. Intravenous potassium E. Beta blockers
C. Intravenous calcium
D. Intravenous digoxin 25. "Bones, stones, groans, and moans" relate to:
E. Intravenous parathormone A. Hyperparathyroidism.
B. Hypoparathyroidism.
20. 80% of all salivary stones occur in: C. Hyperthyroidism.
A. Parotid gland D. Hypothyroidism.
B. Submandibular gland E. Pheochromocytoma.
C. Submaxillary gland
D. Minor salivary glands
E. Sublingual gland
26. A 25 years old university student undergoes total 33. What of the following is the most appropriate
thyroidectomy for multinodular goiter. During this regarding signs and symptoms of early sepsis?
operation inferior thyroid artery is ligated. The nerve A. Respiratory acidosis
intimately related to this artery is: B. Decreased cardiac output
A. Internal laryngeal nerve. C. Hypoglycemia
B. External laryngeal nerve. D. Increased arteriovenous oxygen difference
C. Superior laryngeal nerve. E. Cutaneous vasodilation
D. Recurrent laryngeal nerve (Verify)
E. Non-recurrent laryngeal nerve
34. RTA most
27. In recurrent laryngeal palsy, the position of affected Ans Brain injury
side of vocal cord is:
A. Full abduction 35. A 20 year old male presented with small swelling in
B. Full adduction front of neck in midline, swelling moves upward on
C. Paramedian protrusion of tongue. What is diagnosis
D. Lateral deviation A. Thyroglossal cyst
E. Medial deviation B. Branchial cyst
C. Thyroid nodule
28. Oropharynx common tumor D. Submental lymph node
Ans Squamous cell carcinoma E. Cystic hygroma

29. Frey’s syndrome (SURGERY 2)....


Ans Gustatory sweating

30. What is the most common cause of lymphoedema


(non-pitting oedema) worldwide?
A. Iatrogenic -radiotherapy
B. Filariasis
C. Iatrogenic - lymph node resection surgery
D. Leptospirosis
E. Chagas disease

31. In tongue cancer, the site least affected is:


A. Posterior portion
B. Lateral margin
C. Dorsal surface
D. Tip
E. Ventral surface

32. Healing by first intention means?


A. Using catgut.
B. Obtaining union between two edges of an incision
without subsequent breakdown.
C. Immediate use of protective dressing.
D. Using staples.
E. A method whereby an ulcer heals.
40 Preprof 7. For burn patients, which of the following fluids is
appropriate for resuscitation?
1. Following surgery or major trauma, which of the A. 5% Dextrose water
following metabolic changes occur in the body? B. 10% Dextrose water
A. Decreased adrenal level C. Fresh frozen plasma
B. Decreased cortisol level D. Hypertonic saline
C. Hyperglycaemia E. Ringer’s lactate
D. Hypoglycaemia
E. Increased plasma albumin 8. The most common direction of dislocation of the
shoulder joint is?
2. Which of the following is the causative factor of septic A. Antero-inferior
shock? B. Anterosuperior
A. Acute pancreatitis C. Inferioposterior
B. Excessive vomiting D. Posterior
C. Peritonitis following gut perforation E. Superior
D. Severe diarrhea
E. Severe haemorrhage 9. Regarding the evaluation of spinal cord disorders,
which of the following clinical findings are characteristics
3. Following massive blood transfusion, which of the of upper motor neuron lesions?
following complications may occur? A. Decreased tone – flaccid
A. Hypocalcaemia B. Downgoing plantar response
B. Hypercalcaemia C. Hyperreflexia
C. Hypoglycaemia D. Hyporeflexia
D. Hypoglycaemia E. Sensory loss
E. Hyperlipidaemia
10. Regarding Carpal Tunnel syndrome:
4. Regarding total energy requirement of a stable A. Occurs due to compression of the median nerve
patient, which of the following is approximately needed? B. Occurs due to compression of the ulnar nerve
A. 5-10 Kcal/kg per day C. Occurs due to compression of the radial nerve
B. 10-15 Kcal/kg per day D. Patient usually complains of tingling and numbness
C. 15-20 Kcal/kg per day in the middle one and a half fingers
D. 20-30 Kcal/kg per day E. Wasting of the hypothenar eminence is visible
E. 30-40 Kcal/kg per day
11. During submandibular salivary gland surgery, which
5. In a dehydrated patient, which of the following is the of the following nerves may likely be at risk?
best parameter for assessing hydration after fluid A. Facial nerve
replacement? B. Glossopharyngeal nerve
A. B.P C. Lingual nerve
B. Pulse D. Trigeminal nerve
C. Respiratory rate E. Vagus nerve
D. Temperature
E. Urine output 12. Regarding the parotid gland:
A. 80-90 percent of the tumors are benign
6. Regarding the use of antibiotic prophylaxis before B. Fine needle aspiration cytology (FNAC) may not help
surgery, what is the most appropriate time? in diagnosis
A. 2 hours before incision C. Most tumors arise from the deep lobe of the gland
B. 6 hours before incision D. Open surgical biopsy is usually indicated in all cases
C. 12 hours before incision E. Trigeminal nerve is usually at risk during its surgery
D. 24 hours before incision
E. 30 minutes before incision
13. Intermittent claudication is a cramp-like pain felt in 19. The common complication of deep vein thrombosis
the limb muscles, which of the following statements is is:
appropriate? A. Cerebral artery embolism
A. The pain felt in the lower limb at rest B. Mesenteric artery embolism
B. The pain felt on taking the first step C. Pulmonary artery embolism
C. The pain brought on by walking and relieved by D. Retinal artery embolism
standing still E. Renal artery embolism
D. The foot is often cold and paralyzed
E. Ulceration and gangrene of toes are usually 20. Which of the following gases is used in creating
associated with even mild cases pneumoperitoneum during laparoscopy?
A. Carbon dioxide
14. Deep vein thrombosis is commonly diagnosed by: B. Helium
A. Ascending venography C. Halothane
B. Duplex ultrasound image D. Nitrous oxide
C. D-Dimer test E. Oxygen
D. MRI
E. Trendelenburg test 21. During laparoscopic cholecystectomy, the most
common cause for conversion to an open procedure is:
15. The most important factor inhibiting wound healing A. Dilated common bile duct
is: B. Gallbladder adherent to the liver
A. Diabetes Mellitus C. Gallbladder perforation
B. Infection D. Thick-walled gallbladder Suspecting carcinoma
C. Ischemia E. Uncontrolled bleeding
D. Vitamin C deficiency
E. Zinc deficiency 22. During laparoscopic cholecystectomy, how many
ports are made for this procedure?
16. In a diagnosed case of gastric outlet obstruction, the A. Two
expected metabolic abnormality can be corrected by the B. Three
administration of? C. Four
A. Hypertonic saline or Glucose D. Five
B. Normal saline with added potassium E. Six
C. Plasma expanders
D. Ringer’s lactate/ Hartman solution 23. Urinary retention is the common postoperative
E. Total parenteral nutrition complication after which of the following procedures?
A. Appendectomy
17. Which of the following investigations does not have B. Breast surgery
radiation hazard? C. Thoracic surgery
A. CT scan D. Pelvic and perineal operations
B. X-ray E. Thyroid surgery
C. IVP
D. Thyroid scan 24. The most accurate method of investigation to
E. MRI assess spinal injury is:
A. CT scan
18. The most common cause of increased serum B. MRI spine
calcium levels in hospitalized patients is: C. EMG (electromyography)
A. Primary hyperparathyroidism D. EEG (electroencephalography)
B. Immobilization E. X-ray spine
C. Malignancy
D. Vitamin D excess
E. Myeloma
25. A patient brought to casualty with an obviously short 30. The most common site of occurrence of peptic ulcer
externally rotated leg after trauma, the possibility could is in the:
be: A. 1st part of the duodenum
A. Intertrochanteric fracture B. 2nd part of the duodenum
B. Fracture head of femur C. 3rd part of the duodenum
C. Fracture neck of femur D. 4th part of the duodenum
D. Fracture shaft of femur E. Greater curvature of the stomach
E. Fracture lower end of tibia
31. A 30-year-old male is having fever & tender
26. Severity of head injury is classified according to the hepatomegaly. U/S abdomen suggests a large
Glasgow Coma Score (GCS) and is the best predictor of (15×15cm) liver abscess. Which is the most likely
neurological outcome. What is to be considered the causative agent?
highest possible GCS SCORE? A. Echinococcus granulosus
A. 3/15 B. Entamoeba histolytica
B. 5/15 C. Pseudomonas aeruginosa
C. 10/15 D. Mycobacterium tuberculosis
D. 15/15 E. Staphylococcus aureus
E. 20/15
32. A 23 years male is having splenomegaly and
27. The typical finding of extradural hematoma on CT tendency to bleed upon minor injury. Which of the
scan after head injury is: following disorders are indicated for splenectomy
A. Hyperdense concave lesion A. Acute leukaemia
B. Hypodense concave lesion B. Chronic myeloid leukemia
C. Diffuse mixed density lesion C. Idiopathic thrombocytopenic purpura
D. Lentiform hyperdense lesion D. Polycythaemia
E. Heterogeneous contusion of the brain E. Pernicious anaemia

28. Hernia is a protrusion of a viscus or part of a viscus 33. A 45 years female is c/o pain RHC, U/s abdomen
through an abnormal opening. Which of the following revealed cholelithiasis. Which is the best procedure of
has a high risk of strangulation? choice for this condition
A. Direct inguinal hernia A. ERCP
B. Epigastric hernia B. Lithotripsy
C. Femoral hernia C. Laparoscopic cholecystectomy
D. Indirect inguinal hernia D. Open cholecystectomy
E. Paraumbilical hernia E. Use of stone dissolving agents

29. A middle-aged patient complains of severe 34. A 48 years female presented with obstructive
abdominal pain with a past history of dyspepsia. His jaundice. U/s abdomen revealed dilated CBD with
x-ray chest demonstrates free air under the right dome presence of single stone in it. Which is the most
of the diaphragm. Which of the following is the most preferred technique for its removal
likely possibility? A. ERCP & removal of stone
A. Acute cholecystitis B. Laparoscopic CBD exploration
B. Acute pancreatitis C. Percutaneous choledochoscopy
C. Perforated appendicitis D. Percutaneous transhepatic cholangiography
D. Perforated duodenal ulcer E. Surgery and choledochotomy
E. Ruptured liver abscess
35. A 58 years female presented with severe epigastric 41. Thyrotoxicosis differs from hyperthyroidism due to
pain radiated to the back. Serum amylase level is 72500 the presence of:
i.u. What is the most likely diagnosis A. Tachycardia
A. Acute pancreatitis B. Palpitation
B. Ectopic pregnancy C. Weight loss despite good appetite
C. Mesenteric infarction D. Thyroid swelling
D. Peptic ulcer E. Myopathy and eye signs
E. Retroperitoneal haematoma
42. The commonest cause of acute upper
36. Intestinal tuberculosis can affect any part of the gastrointestinal hemorrhage is:
gastrointestinal tract from mouth to the anus. The sites A. Vascular malformation
affected most often are: B. Mallory-Weiss tear
A. Duodenum C. Peptic ulcer
B. Ileum D. Oesophageal varices
C. Jejunum E. Gastro-duodenal erosion
D. Rectum
E. Sigmoid colon 43. A 70-year-old man presents with a 6-year history of
pruritus ani. The itching and discomfort have increased
37. A volvulus is a twisting or axial rotation of a portion during the last few months. There is neither rectal
of a bowel about its mesentery. What is the most bleeding nor pain. Physical signs suggest chronic
common site of its occurrence? dermatitis, and there is an irregular ulcer on the left anal
A. Ascending colon margin. What is the best next step in management?
B. Caecum A. Wide local excision
C. Descending colon B. Steroid ointment
D. Sigmoid colon C. Nitroglycerine ointment
E. Transverse colon D. Internal sphincterotomy
E. Biopsy
38. In hypothyroidism, which of the following is the most
appropriate clinical sign of this condition? 44. A 35-year-old female is having a simple solitary
A. Clubbing thyroid nodule in the right lobe of the thyroid. Which
B. Delayed relaxation phase of ankle jerks investigation is most appropriate to detect the cause of
C. Tachycardia this abnormality?
D. Tremors A. CT scan neck
E. Warm extremities B. FNAC
C. Thyroid profile
39. The commonest clinical feature of carcinoma of the D. Thyroid scan
head of the pancreas is: E. U/S thyroid
A. Epigastric pain
B. Anorexia and malaise 45. Careful clinical examination will be diagnostic in the
C. Thrombophlebitis vast majority of patients complaining of anal symptoms.
D. Obstructive jaundice Which is the most common position for examination of
E. Mass in the abdomen anal pathology?
A. Jack-knife position
40. Regarding medullary carcinoma of the thyroid: B. Knee-elbow position
A. Tumor takes up radioactive iodine C. Lithotomy position
B. No association with other endocrine abnormalities D. Left lateral (Sims) position
C. No screening test is available E. Right lateral position
D. Prophylactic surgery is recommended in RET
gene-positive children
E. It is associated with low calcitonin levels in blood
46. Which of the following is the best surgical approach 40 Prof…..
for performing prostatic surgery?
A. Retropubic
B. Transperineal
C. Transperitoneal
D. Transvesical
E. Transurethral

47. Which of the following is the most appropriate


regarding hydrocele?
A. It is dilatation of veins within the spermatic cord
B. Primary hydrocele is usually symptomatic
C. Surgery is the mainstay of treatment
D. Testicular malignancy is a common cause of
hydrocele
E. Usually not transilluminate

48. Regarding breast cancer, which pathological type is


the most common variety?
A. Ductal carcinoma
B. Lobular carcinoma
C. Mucinous carcinoma
D. Medullary carcinoma
E. Tubular carcinoma

49. In carcinoma of the breast, the most frequent site of


skeletal metastasis is:
A. Femur
B. Lumbar vertebrae
C. Ribs
D. Skull
E. Pelvis

50. Fine needle aspiration cytology (FNAC) is the


method of investigation for the diagnosis of most thyroid
lesions. Which pathological lesion is not detected by
FNAC?
A. Colloid goiter
B. Follicular carcinoma
C. Medullary carcinoma
D. Nodular goiter
E. Papillary carcinoma
39 Preprof 6. Regarding abnormal wound healing:
A. Hypertrophied scar and keloids are synonyms
1. The most common site of peripheral artery aneurysm B. Keloids are abnormal scars confined to the
is? borders of original scar.
A. Brachial C. Hypertrophied scars always extend beyond the
B. Radial original wound margins
C. Popliteal D. Hypertrophied scar can be stopped or reversed once
D. Tibial the stimulatory phenomenon is over.
E Ulnar E Keloid scars are reversible once the stimulatory
stimulus is over.
2. Which one of the following compartment is most
commonly affected in lower leg compartment 7. Neurogenic shock is characterized by?
syndrome? A. Cool and moist peripheries.
A. Anterior compartment B. Hot and moist peripheries.
B. Lateral compartment C. Decreased resting tone of peripheral vessels.
C. Deep posterior compartment D. Decreased cardiac output.
D. Superficial posterior compartment E. Decreased blood volume.
E. Medial compartment
8. A young male was driving his car when he sustained
3. In patients who have developed documented episode head on collision with a truck: when received in the ER,
of deep venous thrombosis (DVT), the most significant he was in a state of shock & having difficulty in
long- term sequel will be? breathing but his airway was intact. His neck veins were
A. Claudication. distended. The most probable diagnosis is?
B. Recurrent foot infections. A. Tension pneumothorax.
C. Development of stasis ulcer. B. Open pneumothorax
D. Pulmonary embolization C. Pericardial tamponade
E. Diminished arterial perfusion. D. Extradural bleeding.
E. Subdural hemorrhage
4. The fluid that passes through the lymphatic vessels?
A. Flows toward the lungs 9. In a diagnosed case of gastric outlet obstruction, the
B. Passes from the lymphatic vessels into the arteries expected metabolic abnormality can be corrected by
C. Enters the left ventricle of the heart through the right administration of?
thoracic duct A. Hypertonic saline or Glucose
D. Moves in a single direction towards root of the neck B. Normal saline with added potassium
E. Flows toward the kidneys C. Plasma expanders
D. Ringer's lactate/ Hartman solution
5. Most important factor inhibiting the wound healing is: E. Total parenteral nutrition
A. Diabetes Mellitus
B. Infection 10. Which one of the following is metabolic feature of
C. Ischemia starvation?
D. Vitamin C deficiency A. High plasma insulin concentration
E Zinc deficiency B. Low plasma glucagon concentration
C. Hepatic glycogenolysis
D. Adaptive ketoacidosis
E. Increase in energy requirement
(In key its C)
11. Which one of the following is an indication of 16. Regarding medullary carcinoma of thyroid:
parenteral nutrition? A. Tumor takes up radio-active iodine
A. Proximal intestinal fistula B. No association with the other endocrine
B. Vesico-vaginal fistula abnormalities.
C. Recto vaginal fistula C. No screening test is available.
D. Fistula in ano D. Prophylactic surgery is recommended in RET gene
E. Thyroglossal fistula positive children
E. It is associated with low calcitonin level in blood.
12. About the management of major trauma?
A. The peripheral circulation should be evaluated before 17. Triple assessment of breast lump includes all
the airway is assessed except:
B. The airway should always be assessed with the A. Clinical examination.
cervical spine B. Fine needle aspiration cytology.
C. CT scanning is a useful investigation for the C. Mammogram.
assessment of unexplained shock D. Trucut biopsy.
D. A jaw thrust to maintain the airway is indicated in the E. Chest X-ray.
presence of a suspected cervical spine injury
E. Venous access should usually be by two large 18. Specific post parotidectomy complication includes:
cannulas in the antecubital fossa A. Wound infection.
(Verify) B. Frey's syndrome.
C. Hypertrophied scar.
13. Which of the following metabolic responses occur in D. Reactionary bleeding.
response to trauma? E. Altered sensation in front of tragus.
A. Increased growth hormone
B. Decreased antidiuretic hormone 19. Which of the following investigation does not has
C. Decreased ACTH radiation hazard:
D. Decreased urine osmolality A. CT scan
E. Decreased glucagon B. X-ray
C. IVP
14. Thyrotoxicosis differs from hyperthyroidism due to D. Thyroid scan
presence of: E. MRI
A. Tachycardia
B. Palpitation 20. Advantage of full thickness over split thickness skin
C. Weight loss despite good appetite graft is:
D. Thyroid swelling A. Less wound contraction
E. Myopathy and eye signs B. Better take
C more resistance to infection
15. Regarding papillary carcinoma of thyroid: D. Better sensory function
A. Primary root of metastasis is haematogenous E. Can be done under general anesthesia
B. Primary root of metastasis is local invasion
C. Primary root of metastasis is through lymphatics 21. The most common cause of increased serum
D. It has bad prognosis if compared with follicular calcium level in hospitalized patient is:
carcinoma A. Primary hyperparathyroidism
E. Rate of recurrence is high if compared with follicular B. Immobilization
carcinoma C. Malignancy
D. Vitamin D Excess
E. Myeloma
22. which one of the following problem can be E.Emergent surgical repair with exploration of small
encountered in obese patients undergoing surgery. bowel.
A. Deep venous thrombosis 28. 60 years old male with 02 years history of passage
B. Obstruction & sepsis of urine from umbilicus best signifies?
C. poor wound healing A. Acute umbilical infection
D. Hypothermia B. Chronic umbilical infection
E Risk of renal failure C. Patent urachus
D. Umbilical malignancy
23. The most common complication of single blood E. Vitelo intestinal fistula
transfusion is:
A. Hemolytic reaction 29. 70 years old female with past history of open
B. Human immunodeficiency virus transmission cholecystectomy followed by postoperative wound
C. Allergic reaction infection has developed expansile swelling near
D. Volume overloaded operative site. Name most probable diagnosis?
E. Coagulopathy A. Recurrent hernia
(In key its A) B. Incisional hernia
C. Abdominal hernia
24. The commonest complication of chronic D. Dorsal hernia
osteomyelitis is: E. Spigellion hernia
A. Amyloidosis
B. Sinuses discharging pus or piece of bone 30. Identify the single operative choice suitable for
C. Acute exacerbation congenital hernia and hydrocele?
D. Pathological fracture A. Herniotomy
E. Deformity B. Hernioplasty
C. Lord's operation
25. Breast cancer is more commonly found in region of: D. Jaboulay's procedure
A. Axillary E. Truss
B. Central
C. Lower inner quadrant 31. 35 years male presents with midline pea size painful
D. Upper outer quadrant swelling+absent cough impulse most likely diagnosis is?
E. Upper inner quadrant A. Lipoma
B. Abdominal wall haematoma
26. 22 years college student notices bulge in right groin, C. Epigastric hernia
which is accentuated by coughing, but is easily D. Duodenal ulcer
reducible. Which of the following hernia follows the path E. Fibroma
of spermatic cord within the cremasteric muscle?
A. Femoral 32. 40 years female with strangulated femoral hernia
B. Direct inguinal needs to be operated in emergency. Best management
C. Indirect inguinal approach option is?
D. Spigellion A. Laparoscopic repair
E Para umbilical B. Inguinal/lotheissen
C. Low/lackwaad
27. 20 years old woman has reducible hernia below D. High/Mcevedy
inguinal ligament. Which of the following is appropriate E. Mesh repair
management?
A. Observation & follow-up in surgical clinic for 6 33. Investigation of choice for oral cancers?
months A. Radiography
B. Observation & follow-up in surgical clinic of she B. FNAC
develop symptoms C. Redioneucleotide studies
C. Elective surgical repair D. CT
D. Emergent surgical repair E. MRI
34. 25 years female C/O Abdominal pain, vomiting, 39. The commonest underlying cause of acute intestinal
distention & absolute constipation since 5 days, O/E she obstruction in Pakistan is:
has tachycardia, whole abdomen is rigid & tender, A. Obstructed external hernia
bowel sounds are absent.What is probable diagnosis? B. Intussusception
A. Cholecystitis C. Volvulus
B. Intestinal obstruction D. Adhesions
C. Pancreatitis E. Meckel's diverticulum
D. Peritonitis
E. Pyelonephritis 40. The commonest cause of acute upper
(Verify) gastrointestinal hemorrhage is:
A. Vascular malformation.
35. 30 years old male presented with gradual distension B. Mallory Weiss tear
of abdomen and weight loss. O/E abdomen is distended C. Peptic ulcer
and fluid thrill is present peritoneal tape reveales D. Esophageal Varices
exudative fluid.What is the most probable diagnosis? E Gastro duodenal erosion
A. Cirrhosis
B. Malnutrition 41. Strawberry gallbladder is gross description given to:
C. Nephrotic syndrome A. Cholesterosis
D. Congestive cardiac failure B. Adenomyomatosis
E. TB peritonitis C. Porcelain gallbladder
D. Acalculous cholecystitis
36. Pseudomyxoma peritonei is a rare condition in E. Gangrenous cholecystitis
which abdomen is filled with a yellow jelly like material.
The condition is mostly associated with mucinous cystic 42. The commonest clinical feature of carcinoma head
tumors of which organ: of pancreas is:
A. Ovary A. Epigastric pain
B. Intestine B. Anorexia and malaise
C. Kidney C. Thrombophlebitis
D. Uterus D. Obstructive jaundice
E. Stomach E. Mass in the abdomen

37. Chylous ascites is condition in which ascitic fluid 43. A 30 year old woman comes with hypovolemic
appears white, which of the following pathogenesis is shock after blunt trauma of abdomen An emergency
associated with this condition? USG of abdomen shows splenic tear. Which one of the
A. Fibroma following will be next step in management:
B. Lymphoma A CT scan of the abdomen
C. Leukemia B. Diagnostic lavage of peritoneum before proceeding
D. Mesothelioma C. Monitor patient to assess for progression
E. Sarcoma D. Immediate surgery
E. Chest X ray
38. Which of the following investigations for abdominal
tuberculosis have accuracy of around 100% 44. A 70 years old man presents with 6 years history of
A. Bowel wall thickness on UWS pruritus ani the itching and discomfort has increased
B. Enteroclysis (Small bowel enema) during last few months. There is neither rectal bleeding
C. CT Scan Abdomen nor pain. Physical signs suggest chronic dermatitis, and
D. Raised ESR more than 50 mm /1 hour an irregular ulcer on the left anal margin what is the best
E. Polymerase chain reaction next step in management:
A. Wide local excision
B. steroid ointment
C. Nitroglycerine ointment
D. Internal sphincterotomy
E. Biopsy
45. Which of the following is most commonly is
associated with sclerosing cholangitis:
A Crohn's disease
B. Ulcerative colitis
C. Riedel thyroiditis
D. Sarcoidosis
E. Retroperitoneal fibrosis

46. Laparoscopic surgery:


A. Has very limited role in general surgery
B. Is unsafe because surgeon cannot touch the
structures being operated
C. Is associated with greater postoperative pain and
immobility
D. can only be used as part of an operation
E Enables cholecystectomy to be performed as day
care procedure

47. The commonest cause of bladder stone is:


A. Upper urinary tract obstruction
B. Acute urethritis
C. Vesico ureteric reflux
D. Urethral stricture
E. BPH

48. Treatment of transitional cell carcinoma of the renal


pelvis is:
A. Excision of tumor
B. Nephrectomy
C. Nephroureterectomy
D. Radiotherapy alone
E. Chemotherapy

49. The most common complication of hydatid


cyst liver is:
A. Jaundice
B. Rupture into peritoneal cavity
C. Suppuration
D. Rupture into biliary channels
E. Rupture into chest

50. Indication for permanent urinary diversion Includes


all except:
A. Carcinoma prostate
B. Loss of neurological control over bladder
C. Incurable vesico-vaginal fistula
D. Extensive advanced pelvic malignancy
E. After total cystectomy
39 Prof (SURGERY 1) 7. In Amoebiasis the majority of abscesses occur in
which of the following organs?
1. Following surgery or major trauma, which of the A. Spleen
following metabolic changes occur in the body? B. Right lobe of liver
A. Decreased adrenal level C. Lungs
B. Decreased cortisol level D. Sub diaphragmatic
C. Hyperglycaemia E. Left lobe of liver
D. Hypoglycaemia
E. Increased plasma albumin 8. Regarding Amoebic liver abscess
A. Medical treatment is not effective
2. Which of the follow ing maybe clinical features of B. Quinolones are the drug of choice
severe shock C. Repeated aspiration is combined with drug treatment
A. Normal conscious level D. The treatment of choice is always surgery
B. Decreased Respiratory rate E. U/S guided aspiration is contraindicated
C. Decreased Pulse rate
D. Increased urine output 9. Regarding Hydatid disease, the most common
E. Decreased urine output affected organ is
A. Brain
3. Which of the following is the causative factor of B. Kidney
septic shock C. Liver
A. Acute pancreatitis D. Lungs
B. Severe haemorrhage E. Spleen
C. Severe diarrhea
D. Peritonitis following gut perforation 10. Regarding pre-operative preparation before giving
E. Excessive vomiting general anaesthesia, patients are advised not to take
solids:
4. Following massive blood transfusion which of the A. Within 2 hours
following complications may occur B. Within 6 hours
A. Hypoglycaemia C. Within 12 hours
B. Hypercalcaemia D. Within 24 hours
C. Hypoglycaemia E. Within 48 hours
D. Hyperlipidaemia
E. Hypocalcaemia 11. Which of the following is appropriate for metabolic
response to starvation
5. Which of the following is most frequent site of Bed A. High plasma insulin
sores (decubitus ulcers) B. Hepatic gluconeogenesis
A. Sacrum C. Increased resting energy expenditure
B. Greater trochanter D. Low plasma Glucagon
C. Ischium E. Protein anabolism
D. Heal
E. Occiput 12. Regarding total energy requirement of a stable
patient, which of the following is approximately needed?
6. Which of the following factors decrease the risk of A. 5-10 Kcal/kg per day
wound infection B. 10-15 Kcal/kg per day
A. Chemotherapy C. 15-20 Kcal/kg per day
B. Good surgical technique D. 20-30 Kcal/kg per day
C. Obesity E. 30-40 Kcal/kg per day
D. Radiotherapy
E. Steroids
13. Which or the following Kcal/g will be released by 19. The most common direction of dislocation of the
providing 1 gram of fat? shoulder joint is?
A. 10 Kcal/g A. Antero-inferior
B. 6 Kcal/g B. Anterosuperior
C. 8 Kcal/g C. Inferioposterior
D. 9 Kcal/g D. Posterior
E. 4 Kcal/g E. Superior

14. Which of the following is the best parameter for 20. Regarding evaluation of spinal cord disorders, which
assessing hydration in dehydrated patient after fluid of the following clinical findings are characteristics of
replacement upper motor neuron lesion
A. B.P A. Decreased tone - flaccid
B. Pulse B. Down going plantar response
C. Respiratory rate C. Hyperreflexia
D. Temperature D. Hyporeflexia
E. Urine output E. Sensory loss

15. Regarding the use of antibiotic prophylaxis before 21. Which of the following clinical findings are
surgery, what is the most appropriate time? characteristics of lower motor neuron lesion
A. 2 hours before incision A. Ankle clonus
B. 6 hours before incision B. Hyporeflexia
C. 12 hours before incision C. Increased tone-spasticity
D. 24 hours before incision D. Positive Hoffman's sign
E. 30 minutes before incision E. Up going plantar response

16. Which of the following surgical procedures is 22. Regarding Carpal Tunnel syndrome
associated with high risk of development of DVT A. Patient usually complaining of tingling and numbness
postoperatively? in the middle one and half fingers
A. Inguinal hernia surgery B. Occurs due to compression of median nerve
B. Neurosurgery C. Wasting of hypothenar eminence is visible
C. Maxillofacial surgery D. Occurs due to compression of radial nerve
D. Total knee and hip replacement E. Occurs due to compression of ulnar nerve
E. Urological surgery
23. Which of the following malignancies are usually
17. Regarding burns, which of the following fluids is associated with metastatic bone tumours
appropriate for respiration A. Colorectal carcinoma
A. 5% Dextrose water B. Carcinoma of the cervix
B. 10% Dextrose water C. Carcinoma of the stomach
C. Fresh frozen plasma D. Ovarian carcinoma
D. Hypertonic saline E. Prostatic carcinoma
E. Ringer's lactate
24. During submandibular salivary gland surgery, which
18. Regarding burns, the key to monitoring resuscitation of the following nerves may likely be at risk?
is A. Facial nerve
A. Acid-base balance B. Glossopharyngeal nerve
B. Blood pressure C. Lingual nerve
C. Pulse rate D. Trigeminal nerve
D. Respiratory rate E. Vagus nerve
E. Urine output
25. During parotid gland excision which of the following 31. The common complication of deep vein thrombosis
nerves may likely be at risk? is:
A. Trigeminal nerve A. Cerebral artery embolism
B. Accessory nerve B. Mesenteric artery embolism
C. Lingual nerve C. Pulmonary artery embolism
D. Hypoglossal nerve D. Retinal artery embolism
E. Facial nerve E. Renal artery embolism

26. Regarding the parotid gland: 32. The most common cause of lymphedema is
A. 80-90 percent of the tumors are benign A. Deep venous thrombosis
B. Fine needle aspiration cytology (FNAC) may not help B. Filariasis
in diagnosis C. Fungal infection
C. Most tumors arise from the deep lobe of the gland D. Primary lymphatic malignancy
D. Open surgical biopsy is usually indicated in all cases E. Superficial thrombophlebitis
E. Trigeminal nerve is usually at risk during its surgery
33. In hypothyroidism, which of the following is the most
27. Intermittent claudication is a cramp-like pain feel in appropriate clinical sign of this condition
the limb muscles, which of the following statement is A. Clubbing
appropriate B. Delayed relaxation phase of ankle jerks
A. The pain felt in lower limb at rest C. Tachycardia
B. The pain felt on taking the first step D. Tremors
C. The pain brought on by walking and relieved by E. Warm extremities
standing still
D. The foot is often cold and paralyzed 34. Regarding hyperthyroidism, which of the following
E. Ulceration and gangrene of toes are usually are the most appropriate clinical findings
associated with even mild cases A. Bradycardia
B. Bradykinesia, slow movements
28. Regarding management of arterial stenosis or C. Dry skin & hair
occlusion which of the following is the best among D. Hot, moist palms
non-surgical options: E. Hoarse voice
A. Stop smoking
B. Transluminal angioplasty and stenting 35. Regarding malignant thyroid tumours, which of the
C. Peripheral vasodilators following is having high incidence
D. Reduce weight A. Anaplastic carcinoma
E. Beta blockers B. Follicular carcinoma
C. Malignant lymphoma
29. Regarding assessment of Varicose veins legs, D. Medullary carcinoma
which of the following investigations is gold standard E. Papillary carcinoma
A. Duplex ultrasound imaging
B. Descending intravenous venography
C. Magnetic resonance venography (SURGERY 2) ….
D. Tourniquet test
E. Varicography

30. Deep vein thrombosis is commonly diagnosed by


A. Duplex ultrasound image
B. D Dimer test
C. MUI
D. Ascending venography
E. Trendelenburg test
https://drive.google.com/drive/folders/1622Cc8Bh8VOxh
D_2LLdth3OpxEn_aBdx

https://drive.google.com/drive/folders/1AC1yc7EPfVcuQ
TUevzfhMIvnQwmzsduy

You might also like